You are on page 1of 92

1.

परजीविता और परभक्षण के मध्य निम्नलिखित अं तरों पर विचार कीजिए:

1. एक परजीवी शिकार को मार दे ता है जबकि एक परभक्षी परपोषी की मृ त्यु का कारण नहीं बनता है ।

2. परजीविता में दुर्बल सबल से पोषण प्राप्त करता है , जबकि परभक्षण में सबल दुर्बल से पोषण
प्राप्त करता है ।

उपर्युक्त कथनों में से कौन-सा/से सही है /हैं ?

(a) केवल 1

(b) केवल 2

(c) 1 और 2 दोनों

(d) न तो 1, न ही 2

Q1.B

 परभक्षण और परजीविता पृ थ्वी पर विद्यमान जीवों के मध्य दो प्रकार की नकारात्मक जै विक


अं तःक्रियाएं हैं । परजीविता (Parasitism), एक परजीवी और एक परपोषी के मध्य
अं तःक्रिया है , जिसमें परजीवी परपोषी को हानि पहुंचा कर जीवित रहता है । एक परजीवी
परपोषी की मृ त्यु का कारण नहीं बनता है । जबकि परभक्षण (Predation) दो प्रजातियों के
मध्य ऐसा सं बंध है , जहां एक प्रजाति (परभक्षी) शिकार को आहार के लिए मार दे ती है ।
इसलिए कथन 1 सही नहीं है ।
 परजीविता में दुर्बल (जीव) सबल से पोषण प्राप्त करता है , जबकि परभक्षण में सबल दुर्बल
पोषण प्राप्त करता है । इसलिए कथन 2 सही है ।
 उदाहरण के लिए, मछली के मीनपक्षों से सं लग्न ग्लोचिडियम (Glochidium) लार्वा,
परजीविता का एक उदाहरण है , जबकि मछली से आहार प्राप्त करने वाले पक्षी परभक्षण के
उदाहरण हैं ।

2. किसी पारिस्थितिक तं तर् की उत्पादकता के सं दर्भ में , निम्नलिखित कथनों पर विचार कीजिए:

1. पारिस्थितिक तं तर् की सकल प्राथमिक उत्पादकता कार्बनिक और अकार्बनिक पदार्थों के उत्पादन


की दर है ।

2. ने ट प्राथमिक उत्पादकता परपोषितों के उपभोग के लिए उपलब्ध बायोमास है ।

उपर्युक्त कथनों में से कौन-सा/से सही है /हैं ?


(a) केवल 1

(b) केवल 2

(c) 1 और 2 दोनों

(d) न तो 1, न ही 2

Q2.B

 एक पारिस्थितिक तं तर् की सकल प्राथमिक उत्पादकता (Gross Primary Productivity:


GPP) कार्बनिक तत्व की उत्पादन दर होती है । इसलिए कथन 1 सही नहीं है ।
 सकल प्राथमिक उत्पादकता की एक महत्वपूर्ण मात्रा पादपों में श्वसन द्वारा उपयोग की
जाती है । यदि हम सकल प्राथमिक उत्पादकता से श्वसन (R) के दौरान हुई क्षति को घटा दे ते
हैं , तो हमें प्राथमिक उत्पादकता (NPP) प्राप्त होती है ।
 GPP-R= NPP
 ने ट प्राथमिक उत्पादकता परपोषितों के उपभोग या अपघटक के रूप में ) के लिए उपलब्ध
बायोमास होती है । इसलिए कथन 2 सही है ।
 द्वितीयक उत्पादकता को उपभोक्ताओं ने नए पदार्थों के निर्माण की दर के रूप में परिभाषित
किया है ।
 प्राथमिक उत्पादकता एक सु निश्चित क्षे तर् में पादप प्रजातियों के निवास पर निर्भर करती है ।
ये विभिन्न प्रकार के पर्यावरणीय कारकों, पोषकों की उपलब्धता तथा पादपों की प्रकाश
सं श्ले षण क्षमता पर भी निर्भर करती है । इसलिए ये विभिन्न प्रकार के पारिस्थितिक तं तर् ों में
भिन्न-भिन्न होती हैं ।
 सं पर्ण
ू जीव मं डल की वार्षिक ने ट प्राथमिक उत्पादकता का भार कार्बनिक तत्व (शु ष्क भार) के
रूप में लगभग 170 बिलियन टन आं का गया है । यद्यपि पृ थ्वी के धरातल का लगभग 70%
भाग समु दर् से ढका हुआ है , फिर भी इसके बावजूद इनकी उत्पादकता केवल 55 बिलियन टन
है । शे ष मात्रा भूमि पर उत्पन्न होती है ।

ू ण से निपटने के लिए निम्नलिखित में से कौन-से प्रभावी तरीके हैं ?


3. वायु प्रदष

1. कोयले की धु लाई करना।

2. खु ली-चूल्हा भट्टियों के स्थान पर नियं त्रित ऑक्सीजन भट्टियों की स्थापना करना।

3. पे ट्रोलियम रिफाइनरियों में फ्लोटिं ग रूफ कवर्स के साथ भं डारण टैं कों का उपयोग करना।
नीचे दिए गए कू ट का प्रयोग कर सही उत्तर चु निए।

(a) केवल 1 और 2

(b) केवल 1 और 3

(c) केवल 2 और 3

(d) 1, 2 और 3

Q3.B

 ू ण को कम करने के लिए उपयोग की जाने


विकल्प 1 सही है । प्रक्रिया में परिवर्तन, वायु प्रदष
वाली मूल सु धार विधियों में से एक है । उत्पादन की प्रक्रिया में परिवर्तन या सं शोधन करने से
ू कों को कम करने में मदद मिल सकती है । उदाहरण के लिए, फ्लाई ऐश
वायु मंडलीय प्रदष
उत्सर्जन को कम करने के लिए कोयले को चूर्ण बनाने से पूर्व धोया जाता है ।
 विकल्प 2 सही नहीं है और विकल्प 3 सही है । अवां छित उत्सर्जन को कम/नियं त्रित करने के
लिए मौजूदा उपकरणों में सु धार में किया जा सकता है ।
 यदि खु ली चूल्हा भट्टियों (Open-Hearth Furnaces) को नियं त्रित मूलभूत ऑक्सीजन/विद्यु त
भट्टियों (Basic Oxygen/Electric Furnaces) से प्रतिस्थापित कर दिया जाए तो कार्बन
मोनोऑक्साइड, धु एं और गं धयु क्त धु एं (Fumes) के उत्सर्जन को कम किया जा सकता है ।
 पे ट्रोलियम रिफाइनरियों में , भं डारण टैं कों से हाइड्रोकार्बन वाष्प की हानि (वाष्पीकरण या
तापमान में परिवर्तन के कारण) को फ्लोटिं ग रूफ कवर के साथ भं डारण टैं कों को तै यार करके
कम किया जा सकता है ।

4. पारिस्थितिकी के सं दर्भ में , एं टीबायोसिस से तात्पर्य है :

(a) एक ही प्रजाति के जीवों के बीच प्रतिस्पर्धा।

ू री प्रजाति को खाने के लिए मारती है ।


(b) दो प्रजातियों के मध्य साहचर्य, जहां एक प्रजाति दस

ू रे के प्रत्यक्ष उपयोग से लाभान्वित होता है ।


(c) एक सं बंध जिसमें एक जीव दस

ू रों के लिए हानिकारक होते हैं ।


(d) एक जीव द्वारा उत्पन्न स्राव जो दस

Q4.D
 इस धरती पर रहने वाले जीव किसी न किसी रूप में एक-दुसरे से जु ड़े हुए हैं । जीवों के बीच
अं तःक्रिया समग्र रूप से पारिस्थितिक तं तर् के अस्तित्व एवं कार्यप्रणाली के लिए
आधारभूत है ।
 एं टीबायोसिस एक नकारात्मक अं तःक्रिया है जिसमें एक जीव हानिकारक स्राव उत्पन्न करता
है । इस प्रकार के सं बंध में , किसी भी समष्टि को लाभ नहीं होता है । इसलिए विकल्प (d) सही
है ।
 उदाहरण के लिए, तालाबों में उगने वाले नील-हरित शै वाल की कुछ प्रजातियां विषाक्त
पदार्थों का उत्सर्जन करती हैं जिससे मछलियों की मृ त्यु हो जाती है । समु दर् ी जल में , कुछ
रोगाणु ओं की समष्टि, जिन्हें लाल ज्वार (Red tide) के नाम से जाना जाता है , मछली और
अन्य जीवों के विनाश का कारण बनते हैं ।
 इं ट्रास्पे सिफिक स्पर्धा एक ही प्रजाति के जीवों के बीच की स्पर्धा है ।
 ू री प्रजाति
परभक्षण (Predation) दो प्रजातियों के बीच का साहचर्य है , जहां एक प्रजाति दस
को खाने के लिए मारती है ।
 ू रे जीव के प्रत्यक्ष उपयोग से
दोहन (Exploitation) वह सं बंध है जिसमें एक जीव दस
लाभान्वित होता है ।

ू ण नियं तर् ण उपकरण, स्क् रबर के सं दर्भ में ,


5. वायु प्रदष

निम्नलिखित में से कौन-सा कथन सही है ?

(a) यह विद्यु त ऊर्जा सं यंतर् ों में गै सों से निलं बित कणों को अलग करता है ।

ू कों को एक गै स में स्थिरण की


(b) यह गु रुत्वाकर्षण के प्रभाव के कारण नीचे बै ठने के लिए प्रदष
अनु मति प्रदान करता है ।

ू कों को ग्रहण करने के लिए जल के फुहारे का उपयोग करता है ।


(c) यह उत्सर्जन के दौरान प्रदष

(d) यह कणों को विद्यु त आवे श प्रदान करता है जिससे वे धात्विक प्ले टों से चिपक जाते हैं ।

Q5.C

 एक फिल्टर विद्यु त ऊर्जा सं यंतर् ों में गै सों से निलंबित कणों को अलग करता है । धु आं कपड़े
की थै लियों की एक श्रखृं ला से होकर गु जरता है जिसमे निलंबित कण फंस जाते हैं ।
 गु रुत्वाकर्षण निपटान कक्ष (Gravitational Settling Chamber) में विशाल आयताकार कक्ष
ू त गै स प्रवाह प्रवे श करती है । गु रुत्वाकर्षण द्वारा कणों को
होते हैं । जिसमे एक तरफ से प्रदषि
आराम से नीचे बै ठने हे तु समय दे ने के लिए गै स प्रवाह के क्षै तिज वे ग को अत्यं त निम्न स्तर
पर रखा जाता है ।
 ू कों को ग्रहण करने के लिए जल के
स्क् रबर एक ऐसा उपकरण है जो उत्सर्जन के दौरान प्रदष
फुहारे का उपयोग करता है । एक सूखे स्क् रबर का उपयोग अम्लीय गै सों के निलं बन हे तु किया
जाता है । इस प्रक्रिया में गर्म निकास कक्ष में गीले चूने के पाउडर का छिड़काव करना
शामिल है । स्क् रबर इस चूने का उपयोग अम्लीय गै सों को निष्क्रिय करने के लिए करता है ।
इसलिए विकल्प (c) सही उत्तर है ।
 ू ण को
वै द्युत अवक्षे पित्र (Electrostatic Precipitator) अने क रसायनों को हटाकर प्रदष
नियं त्रित करने में सहयोग करता है । यह दहन होने के उपरांत ही धु एं से कणों को हटाता है ।
यह कणों को एक विद्यु त आवे श प्रदान करता है जिससे कण अवक्षे पक के अं दर धातु की प्ले टों
से चिपक जाते हैं ।

6. निम्नलिखित में से कौन-सी चारागाह पारिस्थितिकी तं तर् की विशे षता है /विशे षताएं हैं ?

1. चरागाहों में पशु ओं की एक समृ द्ध विविधता होती है ।

् के लिए अनु कूलतम परिस्थितिया होती हैं ।


2. वृ क्षों की वृ दधि

3. पतला पॉडजोल चरागाहों की मृ दा की विशे षता है ।

नीचे दिए गए कू ट का प्रयोग कर सही उत्तर चु निए।

(a) केवल 1

(b) केवल 2 और 3

(c) केवल 1 और 3

(d) केवल 2

Q 6.A

 अं टार्क टिका को छोड़कर प्रत्ये क महाद्वीप पर घास के मै दान/चारागाह पाए जाते हैं । ये
स्थलीय पारिस्थितिकी तं तर् पृ थ्वी की सतह के लगभग 19 प्रतिशत भाग को आच्छादित
करते हैं । घास की विस्तृ त विविधता के प्रभु त्व के साथ वृ क्षहीन जड़ी-बूटियों के पौधे इनकी
विशे षताएं हैं ।
 चरागाहों की विशे षताएं :
o चारागाह प्रतिवर्ष लगभग 25-75 से मी वर्षा वाले क्षे तर् ों में पाए जाते वर्षा वनों के विकास
के लिए पर्याप्त नहीं होती है , जबकि एक वास्तविक रे गिस्तान (true desert) की तु लन
अधिक है । विशिष्ट चारागाह वनस्पति सं रचनाएं वे हैं जो सामान्य तौर पर समशीतोष्ण
जलवायु में पायी जाती
o चरागाहों में घासों का प्रभु त्व होता है । बड़ी झाड़ियाँ या पाए जाते हैं । इसलिए कथन 2
सही नहीं है ।
o चरागाहों में जं तुओं की एक समृ द्ध विविधता होती कथन 1 सही है ।
o मृ दा हर जगह पायी जाती है , कहीं-कहीं चट् टानी अधिकां शतः स्थिर या मोबाइल टीलों
के साथ रे तीली। चारा केवल अल्पकालिक आर्द्र मौसम के दौरान उपलब्ध होता है । पतले
पॉडजोल, बोरियल वन मृ दा की विशे षता है । इसलिए कथन 3 सही नहीं है ।
o चरागाह लाखों वर्षों से चराई वाले पशु ओं के घर रहे हैं ।

7. निम्नलिखित में से कौन-से प्रकाश रासायनिक धूम कोहरे के घटक हैं ?

1. ओजोन

2. फार्मेल्डिहाइड

3. पे राक्सीएसीटिल नाइट् रेट (PAN)

नीचे दिए गए कू ट का प्रयोग कर सही उत्तर चु निए।

(a) केवल 1 और 2

(b) केवल 2 और 3

(c) केवल 1 और 3

(d) 1, 2 और 3

Q7.D

 ू कों का एक मिश्रण है जिसका निर्माण तब होता है जब


प्रकाश रासायनिक धूम कोहरा प्रदष
नाइट् रोजन ऑक्साइड और वाष्पशील कार्बनिक यौगिक (VOCs) सूर्य के प्रकाश में
अभिक्रिया करते हैं , जिससे शहरों के ऊपर एक भूरे रं ग की धुं ध निर्मित हो जाती है । यह उष्ण,
शु ष्क और साफ धूपमयी जलवायु में होता है । प्रकाश-रासायनिक धूम कोहरा के मु ख्य घटक
स्वचालित वाहनों और कारखानों द्वारा उत्पादित असं तृप्त हाइड्रोकार्बन एवं नाइट् रोजन
ऑक्साइड पर सूर्य के प्रकाश की क्रिया के परिणामस्वरूप होते हैं । प्रकाश-रासायनिक धूम
कोहरे में ऑक्सीकारक अभिकर्मकों की उच्च सांदर् ता होती है और इसलिए इसे ऑक्सीकारक
धूम कोहरा कहा जाता है ।
 प्रकाश रासायनिक धु म कोहरा का निर्माण:
o जब जीवाश्म ईंधनों का दहन होता है , तो पृ थ्वी के क्षोभमं डल में हाइड्रोकार्बन (अदहित
ू क उत्सर्जित होते हैं ।
ईंधन) और नाइट्रिक ऑक्साइड (NO) सहित कई प्रकार के प्रदष
ू कों का स्तर पर्याप्त रूप से ऊँचा हो जाता है , तब सूर्यप्रकाश से इनकी
जब इन प्रदष
अन्योन्य क्रिया के परिणामस्वरूप एक शृं खला अभिक्रिया होती है जिसमें NO
नाइट् रोजन डाइऑक्साइड (NO2) में परिवर्तित हो जाती है ।
o बदले में यह NO2 सूर्य के प्रकाश से ऊर्जा को अवशोषित करके नाइट्रिक ऑक्साइड और
मु क्त ऑक्सीजन परमाणु ओं में विघटित हो जाता है ।
 NO2(g)→ NO(g) + O(g)
o ऑक्सीजन परमाणु अत्यधिक क्रियाशील होने के कारण हवा में O2 के साथ सं युक्त होकर
ओजोन को उत्पादित करते हैं ।
 O(g) + O2 (g) → 03 (g)
o उपर्युक्त अभिक्रिया में निर्मित ओजोन NO के साथ शीघ्रतापूर्वक अभिक्रिया करके
NO2 को पु न: उत्पन्न करती है । NO2 एक भूरी गै स है और जिसका पर्याप्त उच्च स्तर धुं ध
का कारण हो सकता है ।
 NO(g)+O (g)-NO (g)+O (g)
 ू त
ओजोन एक जहरीली गै स है और NO2 तथा 03 दोनों ही प्रबल ऑक्सीकारक हैं और प्रदषि
वायु में अदहित हाइड्रोकार्बन के साथ अभिक्रिया करके फार्मेल्डिहाइड, एक् रोलिन और
परॉक्सीऐसीटिल नाइट् रेट (PAN) जै से रसायनों का निर्माण करते हैं ।
 प्रकाश रासायनिक धुं ध के प्रभाव:
 प्रकाश रासायनिक धूम कोहरे के सामान्य घटक ओजोन, नाइट्रिक ऑक्साइड, एक् रोलिन,
फार्मेल्डिहाइड और परॉक्सीऐसीटिल नाइट् रेट (PAN) हैं । प्रकाश रासायनिक धूम कोहरे के
कारण भी गं भीर स्वास्थ्य समस्याएं उत्पन्न होती हैं । ओजोन और PAN दोनों ही आं खों में
जलन उत्पन्न करने वाले शक्तिशाली कारक के रूप में कार्य करते हैं । ओजोन और नाइट्रिक
ऑक्साइड नाक और गले में जलन उत्पन्न करते हैं और इनकी अत्यधिक सांदर् ता के कारण
सिरदर्द, सीने में दर्द, गले का शु ष्क होना, खांसी और सांस ले ने में कठिनाई होती है । इसलिए
विकल्प (d) सही उत्तर है ।

8. ओजोन अवक्षय के सं दर्भ में , निम्नलिखित कथनों पर विचार कीजिए:


1. क्लोरोफ्लोरोकार्बन (CFC) विषहीन कार्बनिक अणु हैं जो ओजोन अवक्षय में सहायता करते हैं ।

2. ध्रुवीय समतापमं डलीय बादल ओजोन अवक्षय के कारण होने वाली अभिक्रियाओं के लिए एक
सतह प्रदान करते हैं ।

3. मु क्त क्लोरीन मूलक तब निर्मित होते हैं जब पराबैं गनी विकिरणों द्वारा CFCs को विघटित किया
जाता हैं ।

उपर्युक्त कथनों में से कौन-से सही हैं ?

(a) केवल 1 और 2

(b) केवल 2 और 3

(c) केवल 1 और 3

(d) 1, 2 और 3

Q8.D

 पराबैं गनी (UV) विकिरणों की डाइऑक्सीजन (O2) अणु ओं से उत्पाद समताप मं डल में
उपस्थित ओजोन पराबैं गनी (UV) विकिरण आणविक ऑक्सीजन को मु क्त परमाणु ओं में
विखं डित कर दे ते हैं । ओजोन ऊष्मागतिकी रूप से अस्थाई होते है एवं आण्विक ऑक्सीजन में
विध हो जाती है । इस प्रकार, ओजोन अणु ओं के निर्माण और विघटन में एक गतिकीय साम्य
स्थापित हो जाता है ।
 हाल के वर्षों में , समताप मं डल में कुछ रसायनों की के कारण ओजोन की इस सु रक्षा-परत में
अवक्षय की सूचनाएं प्राप्त हुई हैं । ओजोन परत में अवक्षय का मु ख्य कारण
क्लोरोफ्लोरोकार्बन यौगिकों (CFCs) का उत्सर्जन है , जिन्हें फ्रियोन (freons) भी कहा जाता
है । ये यौगिक अक्रिय, अज्वलनशील, विषहीन कार्बनिक अणु हैं । अतः इनका उपयोग
रे फ्रिजरे टर, एयरकंडीशनर, प्लास्टिक फोम के उत्पादन में और इले क्ट् रॉनिक उद्योग द्वारा
कंप्यूटर कल पु र्जो की सफाई आदि में होता है । CFCs के एक बार वायु मंडल में उत्सर्जित होने
पर ये वायु मंडल की अन्य गै सों से मिश्रित होकर सीधे समताप मं डल में पहुँच जाते हैं ।
समताप मं डल में , ये शक्तिशाली UV विकिरणों से विघटित होकर क्लोरीन मु क्त मूलक
उत्सर्जित करते हैं । क्लोरीन मु क्त मूलकतब समतापमं डलीय ओजोन के साथ अभिक्रिया करके
क्लोरीन मोनोऑक्साइड मूलक और आण्विक ऑक्सीजन बनाते हैं ।क्लोरीन मोनोऑक्साइड
मूलक परमाण्वीय ऑक्सीजन के साथ की अभिक्रिया करके अधिक क्लोरीन मूलक उत्पन्न
करते हैं । क्लोरीन मूलक लगातार पु नर्योजित होते रहते हैं एवं ओजोन को विखं डित करते हैं ।
इस प्रकार, CFCs समताप मं डल में लगातार क्लोरीन मूलकों को उत्पन्न करने वाले और
ओजोन-परत को हानि पहुँचाने वाले परिवहनीय कारक हैं । इसलिए कथन 1 और 3 सही हैं ।
 सर्दी के मौसम में , अं टार्क टिका के ऊपर एक विशे ष प्रकार के बादल बनते हैं । जिन्हें ध्रुवीय
समतापमं डलीय बादल कहा जाता है । ये ध्रुवीय समतापमं डलीय बादल एक सतह प्रदान
करते हैं जिस पर बना हुआ क्लोरीन नाइट् रेट जलयोजित होकर हाइपोक्लोरस अम्ल बनाता
हैं । यह अभिक्रिया में उत्पन्न हाइड्रोजन क्लोराइड के साथ भी अभिक्रिया करके आण्विक
क्लोरीन दे ता है । बसं त ऋतु में अं टार्क टिका पर जब सूर्य का प्रकाश लौटता है , तब सूर्य की
गर्मी बादलों को विखं डित कर दे ती है , और HOCL और Cl2 सूर्य के प्रकाश से अपघटित
(photolyzed) हो जाते हैं । जै सा पूर्व में बताया गया है , इस प्रकार निर्मित क्लोरीन मूलक
ओजोन अवक्षय के लिए शृं खला अभिक्रिया प्रारं भ कर दे ते हैं । इसलिए कथन 2 सही है ।

9. पारिस्थितिक तं तर् में नाइट् रोजन के निम्नलिखित में से कौन-से स्रोत हैं ?

1. कृषि भूमि

2. जीवाश्म ईंधन

3. पशु धन

4. शहरी अपशिष्ट

नीचे दिए गए कू ट का प्रयोग कर सही उत्तर चु निए।

(a) केवल 1 और 4

(b) केवल 2, 3 और 4

(c) केवल 1, 2 और 3

(d) 1, 2, 3 और 4

Q9.D

 जीवाश्म ईंधन के दहन, नाइट् रोजन आधारित उर्वरकों के उपयोग अन्य गतिविधियों से
पारिस्थितिकी तं तर् में जै विक रूप से उपलब्ध नाइट् रोजन की मात्रा में उल्ले खनीय रूप से
् है । अतः वातावरण में नाइट् रोजन के मु ख्य स्रोतों में सम्मिलित हैं :
वृ दधि
o वायु मंडलीय वर्षा,
o भूगर्भीय स्रोत,
o कृषि भूमि,
o पशु धन (गाय, भे ड़, बकरी आदि जै से जु गाली करने वाले पशु ) और कुक्कुट पालन
गतिविधियां तथा
o शहरी अपशिष्ट
 इसलिए विकल्प (d) सही उत्तर है ।

10. राष्ट् रीय तापविद्यु त निगम लिमिटे ड (NTPC) द्वारा भारत का सबसे बड़ा सौर पार्क किस राज्य में
स्थापित किया जाएगा?

(a) गु जरात

(b) राजस्थान

(c) तमिलनाडु

(d) उत्तर प्रदे श

Q 10.A

 हाल ही में , राष्ट् रीय तापविद्यु त निगम लिमिटे ड (NTPC) की 100 फीसदी सहायक कंपनी,
एनटीपीसी रिन्यूएबल एनर्जी लिमिटे ड को गु जरात के खवाड़ा में कच्छ के रण में 4,750
मे गावाट नवीकरणीय ऊर्जा पार्क स्थापित करने के लिए नवीन एवं नवीकरणीय ऊर्जा मं तर् ालय
(MNRE) से मं जरू ी मिल गई है । यह भारत का सबसे बड़ा सोलर पार्क होगा, जिसका निर्माण
दे श की सबसे बड़ी विद्यु त उत्पादक कंपनी करे गी।
 सौर ऊर्जा पार्क योजना के अल्ट् रा मे गा रिन्यूएबल एनर्जी पावर पार्क के बारे में "सौर पार्कों
और अल्ट् रा मे गा सौर ऊर्जा परियोजनाओं के विकास" की योजना 12-12-2014 को नवीन एवं
नवीकरणीय ऊर्जा मं तर् ालय द्वारा प्रारं भ की गई थी।
 इस योजना के तहत, वर्ष 2014-15 से प्रारं भ होने वाले 5 वर्षों की अवधि के भीतर 20,000
मे गावाट से अधिक सौर ऊर्जा स्थापित क्षमता को लक्षित करते हुए कम से कम 25 सौर पार्क
और अल्ट् रा मे गा सौर ऊर्जा परियोजनाओं को स्थापित करने का प्रस्ताव किया गया था।
 योजना की क्षमता वर्ष 2017 में 20,000 मे गावाट से बढ़ाकर 40,000 मे गावाट कर दी गई है ।
 NTPC REL की भी इस पार्क से व्यावसायिक स्तर पर हरित हाइड्रोजन उत्पन्न करने की
योजना है ।
 ् के एक भाग के रूप में , NTPC लिमिटे ड का लक्ष्य 2032
अपने हरित ऊर्जा पोर्टफोलियो वृ दधि
तक 60 गीगावॉट अक्षय ऊर्जा क्षमता का निर्माण करना है । वर्तमान में , राज्य के स्वामित्व
वाली विद्यु त कंपनी की 70 विद्यु त परियोजनाओं में 66 गीगावॉट की स्थापित क्षमता है , जिसमें
अतिरिक्त 18 गीगावॉट निर्माणाधीन है ।
 हाल ही में , NTPC द्वारा आं धर् प्रदे श के सिम्हाद्री तापीय विद्यु त सं यंतर् के जलाशय में
भारत का सबसे बड़ा 10 मे गावाट (ac) का फ्लोटिं ग (तै रता हुआ) सोलर भी प्रारं भ किया गया
है । इसलिए विकल्प (a) सही उत्तर है ।

11. निम्नलिखित में से कौन-सी ग्रीन हाउस गै स (GHG) है /हैं ?

1. जल वाष्प

2. कार्बन डाइऑक्साइड

3. नाइट् रस ऑक्साइड

नीचे दिए गए कू ट का प्रयोग कर सही उत्तर चु निए।

(a) केवल 1 और 2

(c) केवल 1 और 3

(d) 1, 2 और 3

Q 11.D

 हरित गृ ह प्रभाव (Greenhouse effect) एक प्राकृतिक प्रक्रिया है जो पृ थ्वी की सतह को


गर्म करती है । जब सूर्य की ऊर्जा पृ थ्वी के वायु मंडल में पहुंचती है , तो उसमें से कुछ वापस
अं तरिक्ष में परावर्तित हो जाती है और शे ष ग्रीन हाउस गै सों द्वारा अवशोषित और पु नः
विकिरणित हो जाती है । पृ थ्वी वायु के आवरण से घिरी हुई है जिसे वायु मंडल कहते हैं ।
वातावरण में ग्रीन हाउस गै स के अणु ऊष्मा को रोककर रखते हैं क्योंकि ये सूर्य के प्रकाश के
लिए पारदर्शी होते हैं किन्तु ऊष्मा के विकिरण के लिए पारदर्शी नहीं होते हैं । यदि कार्बन
डाइऑक्साइड की मात्रा 0.03 प्रतिशत के सं वेदनशील अनु पात को पार कर जाती है , तो
प्राकृतिक ग्रीन हाउस सं तुलन अव्यवस्थित हो सकता है । वै श्विक ऊष्मन में कार्बन
डाइऑक्साइड का प्रमु ख योगदान है ।
 कार्बन डाइऑक्साइड के अतिरिक्त, अन्य ग्रीन हाउस गै सें मीथे न, जल वाष्प, नाइट् रस
ऑक्साइड और ओजोन हैं । मीथे न प्राकृतिक रूप से तब उत्पन्न होती है जब ऑक्सीजन की
अनु पस्थिति में वनस्पति का दहन, पाचन अथवा विगलन किया जाता है । धान के खे तों,
कोयला खदानों, विगलित अपशिष्ट के ढे रों और जीवाश्म ईंधन से मात्रा में मीथे न निर्मुक्त
होती है । क्लोरोफ्लोरोकार्बन (CFCs) मानव निर्मित औद्योगिक रसायन हैं जिसका योग एयर
कंडीशनिं ग आदि में किया जाता है । CFC भी ओजोन परत को क्षति पहुंचा रही हैं । नाइट् रस
ऑक्साइड वात रण में प्राकृतिक रूप से पायी जाती है । हाल के वर्षों में , रासायनिक उर्वरकों के
् हुई है ।
उपयोग और जीवाश्म ईंधन के दहन के इनकी मात्रा में अधिक वृ दधि
 इसलिए विकल्प (d) सही उत्तर है ।

12. कार्बन के ऑक्साइड के सं दर्भ में , निम्नलिखित कथनों पर विचार कीजिए:

1. कार्बन के डाइऑक्साइड वायु मंडल के आयतन का 3% निर्मित करती है । कथनों में से कौन-सा/से
सही है /हैं ?

(a) केवल 1

(b) केवल 2

(c) 1 और 2 दोनों

(d) न तो 1, न ही 2

Q 12.A

 कार्बन ऑक्साइड या ऑक्सोकार्बन, कार्बनिक यौगिकों के एक वर्ग हैं जिसमें केवल कार्बन और
ऑक्सीजन होते हैं । सबसे मूलभूत ऑक्सोकार्बन कार्बन मोनोऑक्साइड और कार्बन
डाइऑक्साइड हैं ।
 कार्बन मोनोऑक्साइड: कार्बन मोनोऑक्साइड (CO) एक रं गहीन एवं गं धहीन गै स है जो जीवों
के लिए अत्यधिक जहरीली होती है , क्योंकि यह अं गों और ऊतकों के लिए ऑक्सीजन के
वितरण को अवरुद्ध करने की क्षमता रखती है । यह कार्बन के अपूर्ण दहन के परिणामस्वरूप
उत्पन्न होती है । CO जहरीली होती है क्योंकि यह हीमोग्लोबिन को कार्बोक्सीहीमोग्लोबिन
निर्मित करने के लिए बं ध बनाती है , जो ऑक्सीजन-हीमोग्लोबिन कॉम्प्ले क्स की तु लना में
लगभग 300 गु ना अधिक स्थायित्व यु क्त होता है । रक्त में , जब कार्बोक्सीहीमोग्लोबिन की
सांदर् ता लगभग 3-4 प्रतिशत तक पहुँच जाती है , तब रक्त की ऑक्सीजन-वहन क्षमता
अत्यन्त कम हो जाती है । इस ऑक्सीजन की कमी के परिणामस्वरूप सिरदर्द, कमजोर दृष्टि,
घबराहट और हृदय तथा रक्त वाहिकाओं सं बंधी विकार उत्पन्न होते हैं । गर्भवती महिलाओं
में , जिन्हें धूमर् पान की आदत है , के रक्त में CO का स्तर बढ़ जाने से समय से पहले बच्चे का
जन्म, स्वतः प्रसूत गर्भपात और विकृत बच्चे का जन्म हो सकता है । इसलिए कथन 1 सही
है ।
 कार्बन डाइऑक्साइड: कार्बन डाइऑक्साइड (CO2) को श्वसन, ऊर्जा के लिए जीवाश्म ईंधन
के दहन और सीमें ट के निर्माण के दौरान चूना पत्थर के अपघटन द्वारा वातावरण में निर्मुक्त
किया जाता है । यह ज्वालामु खी विस्फोट के दौरान भी उत्सर्जित होती है । सामान्यतः यह
वायु मंडल के आयतन का लगभग 0.03 प्रतिशत निर्मित करती है । इसलिए कथन 2 सही नहीं
है ।
 जीवाश्म ईंधन के बढ़ते उपयोग से बड़ी मात्रा में कार्बन डाइऑक्साइड वायु मंडल में निर्मुक्त
हो जाती है । हरित पादपों को प्रकाश सं श्ले षण के लिए CO2 की आवश्यकता होती है और
बदले में , वे ऑक्सीजन का उत्सर्जन करते हैं , इस प्रकार वे उत्कृष्ट सं तुलन बनाए रखते हैं ।
वनोन्मूलन और जीवाश्म ईंधन के दहन से CO2 का स्तर बढ़ जाता है और वायु मंडल में
सं तुलन बिगड़ जाता है । वायु में CO2 की बढ़ी हुई मात्रा मु ख्य रूप से वै श्विक तापन के लिए
उत्तरदायी है ।

13. ओजोन के सं दर्भ में , निम्नलिखित कथनों पर विचार कीजिए:

1. क्षोभमं डलीय ओजोन एक ग्रीनहाउस गै स है ।

2. समताप मं डल में ओजोन, पराबैं गनी (UV) विकिरणों की डाइऑक्सीजन (O2) अणु ओं से
प्रतिक्रिया का उत्पाद है ।

3. ओजोन ऑक्सीजन की तु लना में ऊष्मागतिकी रूप से स्थिर है । उपर्युक्त कथनों में से कौन-से सही
हैं ?

(a) केवल 1 और 3

(b) केवल 2 और 3

(c) केवल 1 और 2

(d) 1, 2 और 3

Q 13.C

 धरातलीय स्तर पर विद्यमान या क्षोभमं डलीय ओजोन नाइट् रोजन के ऑक्साइड (NOx गै सों)
और वाष्पशील कार्बनिक यौगिको (VOCs) के मध्य रासायनिक अभिक्रियाओं द्वारा निर्मित
होता है । क्षोभमं डल में विद्यमान ओजोन को ग्रीनहाउस गै स माना जाता है और यह वै श्विक
तापन में योगदान करती है । यह प्रकाश रासायनिक धूम कोहरा का भी एक सामान्य घटक है ।
इसलिए कथन 1 सही है ।
 ऊपरी समताप मं डल में ओजोन (O3) की प्रचु र मात्रा विद्यमान होती है , जो सूर्य से आने
वाले हानिकारक पराबैं गनी (UV) विकिरणों से सु रक्षा प्रदान करती है । ये विकिरणें मनु ष्यों में
त्वचा कैंसर (मे लेनोमा) का कारण बनती हैं । इसलिए ओजोन सु रक्षा परत को बनाए रखना
अत्यधिक महत्वपूर्ण है । समताप मं डल में ओजोन, पराबैं गनी (UV) विकिरणों की
डाइऑक्सीजन (O2) अणु ओं से प्रतिक्रिया का उत्पाद है । पराबैं गनी (UV) विकिरण आणविक
ऑक्सीजन को मु क्त ऑक्सीजन (O) परमाणु ओं में विखं डित कर दे ते हैं । ये ऑक्सीजन परमाणु
ऑक्सीजन से सं युक्त होकर ओजोन का निर्माण करते हैं । इसलिए कथन 2 सही है ।
 ओजोन ऊष्मागतिकी रूप से ऑक्सीजन की तु लना में अस्थायी है और आणविक ऑक्सीजन में
विघटित हो जाती है जिसके परिणामस्वरूप ऊष्मा निर्मुक्त होती है । ओजोन में ऑक्सीजन के
तीन अणु होते हैं और इस प्रकार यह अस्थिर अवस्था में होता है । अतः स्थिर होने के लिए,
यह द्विआणविक अवस्था की पु नर्घाप्ति हे तु ऑक्सीजन के एक अणु को निर्मुक्त कर दे ती है ।
इस प्रकार, ओजोन अणु ओं के निर्माण और विघटन में एक गतिकीय साम्य स्थापित हो जाता
है । हाल के वर्षों में , समतापमं डल में कुछ रसायनों की उपस्थिति के कारण ओजोन की इस
सु रक्षात्मक परत के अवक्षय होने की सूचना प्राप्त हुई है । ओजोन पर अवक्षय का मु ख्य कारण
क्लोरोफ्लोरोकार्बन यौगिकों (CFCs) का उत्सर्जन है , जिसे 'फ्रियोन' भी कहा जाता है ।
इसलिए कथन 3 सही नहीं है ।

14. बायोगै स के सं दर्भ में , निम्नलिखित कथनों पर विचार कीजिए:

1. यह वायु की उपस्थिति में जै वभार, जै विक खाद, नगरपालिका अपशिष्ट और हरित कचरे जै से जै व
निम्नीकरणीय पदार्थों के किण्वन से उत्पन्न होती है ।

2. इसमें मु ख्य रूप से मीथे न और कार्बन डाइऑक्साइड होते हैं ।

3. इसका ऊष्मीय मान बहुत कम होता है और इसे ईंधन के रूप में उपयोग नहीं किया जा सकता है ।

उपर्युक्त कथनों में से कौन-सा/से सही नहीं है /हैं ?

(a) केवल 1

(b) केवल 2 और 3

(c) केवल 1 और 3

(d) केवल 2

Q14.C
 बायोगै स का तात्पर्य ऑक्सीजन की अनु पस्थिति में कार्बनिक पदार्थों के जै विक अपघटन से
उत्पन्न होने वाली गै स से है । इसलिए कथन 1 सही नहीं है ।
 बायोगै स जै वभार, जै विक खाद या वाहित मल, नगरपालिका अपशिष्ट, हरित अपशिष्ट और
ऊर्जा फसलों (energy crops) जै से जै व निम्नीकरणीय पदार्थों के अवायवीय पाचन (anaerobic
digestion) या बायोगै स में मीथे न और कार्बन डाइऑक्साइड होते हैं । एक अन्य प्रकार काष्ठ
गै स gas) है जो काष्ठ या अन्य जै वभार के गै सीकरण द्वारा निर्मित की जाती है ।
 बायोगै स में मु ख्य रूप से मीथे न, कार्बन डाइऑक्साइड और अल्प मात्रा नाइट् रोजन,
हाइड्रोजन एवं कार्बन मोनोऑक्साइड होते हैं । इसलिए कथन 2 सही है ।
 गोबर गै स, जै विक खाद के अवायवीय पाचन (anaerobic से उत्पन्न होती है ।
 बायोगै स का महत्वः
o बायोगै स सं यंतर् से प्राप्त उत्पादों, अर्थात मीथे न उपयोग ईंधन के रूप में और तरल
ू स का उपयोग प्राकृतिक उर्वरक के रूप में किया जाता है ।
ह्यम
o बायोगै स को स्थानीय रूप से उपलब्ध पदार्थों जै से पशु ओं के गोबर, कृषि अपशिष्ट
आदि से उत्पन्न किया जा सकता है ।
o यह एक स्वच्छ ईंधन है ।
o इसका ऊष्मीय मान उच्च होता है । इसलिए कथन 3 सही नहीं है ।

ू क का प्रकार जनित रोग


15. निम्नलिखित यु ग्मों पर विचार कीजिए: प्रदष

1. आर्सेनिक है जा

2. पारा इटाई-इटाई

3. कैडमियम मिनामाता रोग

उपर्युक्त यु ग्मों में से कौन-सा/से सही सु मेलित है /हैं ?

(a) केवल 1

(b) केवल 1 और 2

(c) केवल 2 और 3

(d) उपर्युक्त में से कोई नहीं

Q15.D
 यु ग्म 1 सही सु मेलित नहीं है । अपशिष्ट जल, जिसमें वाहित मल शामिल है , में रोग उत्पन्न
ू त जल (न कि आर्सेनिक) कई रोगों जै से है जा,
करने वाले कई रोगाणु होते हैं । प्रदषि
टाइफाइड, अतिसार (डायरिया), पे चिश, पोलियो और पीलिया का कारण बन सकता है । है जा
(Cholera) एक अत्यधिक दस्त लगने का रोग है जो आं त में विब्रियो कॉले री (Vibrio
Cholerae) नामक बै क्टीरिया के सं क्रमण के कारण होता है ।
 ू ण इटाई-
यु ग्म 2 सही सु मेलित नहीं है । जापान में भारी धातु कैडमियम से होने वाला प्रदष
इटाई (Itai-ltai) रोग का कारण बना।
 यु ग्म 3 सही सु मेलित नहीं है । जल में क्षे पित पारे को जीवाणु ओं की क्रिया के द्वारा मिथाइल
मरकरी (Methylmercury) में परिवर्तित कर दिया जाता है जो जल में विले य होता है । यह
ू त
मिथाइल मरकरी मछलियों के शरीर में सं चित हो जाता है । इस मिथाइल मरकरी से सं दषि
मछलियों का से वन करने से बड़े आने पर माता रोग उत्पन्न हुआ।

16. निम्नलिखित पर विचार कीजिए:

1. CD और DVD के स्टायरोफोम के खोल

2. चमड़े के बै ग

3. प्लास्टिक के खिलौने

4. सं श्ले षित कीटनाशक

उपर्युक्त अपशिष्ट उत्पादों में से कौन-से अजै व निम्नीकरणीय अपशिष्ट की श्रेणी में आते हैं ?

(a) केवल 1, 2 और 4

(b) केवल 1, 3 और 4

(c) केवल 2 और 3

(d) 1, 2, 3 और 4

Q16.B

 अपशिष्टों को जै व-निम्नीकरणीय (biodegradable) और अजै व निम्नीकरणीय (non-


biodegradable) अपशिष्ट जै सी विभिन्न श्रेणियों में वर्गीकृत किया जा सकता है ।
 ऐसे अपशिष्ट जो कवक और जीवाणु ओं की सूक्ष्मजै विक क्रियाओं के द्वारा निम्नीकृत या
अपघटित हो सकते हैं , वे जै व निम्नीकरणीय अपशिष्ट कहलाते हैं । कृषि अपशिष्ट, मल
अवशे ष, मृ त पादप, चमड़े के जूते/बै ग, टिन के डिब्बे इस श्रेणी में आते हैं ।
 ऐसे अपशिष्ट जो सूक्ष्मजै विक क्रियाओं के द्वारा निम्नीकृत या अपघटित नहीं हो सकते हैं , वे
अजै व निम्नीकरणीय अपशिष्ट कहलाते हैं । इस तरह के अपशिष्टों में कच्चा पे ट्रोलियम,
प्लास्टिक, स्टायरोफोम उत्पाद, कांच, पॉलीमर, सं श्ले षित कीटनाशक, रे डियोधर्मी राख आदि
शामिल हैं ।
o स्टायरोफोम एक शु द्ध ठोस और कठोर पदार्थ है जो सीमित रूप से लचीला होता है ।
इसका उपयोग डिस्पोजे बल कटलरी,
o प्लास्टिक मॉडल, CD और DVD के खोल आदि बनाने के लिए किया जाता है ।
o इसलिए विकल्प (b) सही उत्तर है ।

17. नवीनतम वै श्विक साइबर सु रक्षा सूचकांक (GCl), 2020 में भारत को 10 वां स्थान प्रदान किया
गया है । GCI जारी किया जाता है :

(a) ग्लोबल साइबर अलायं स के द्वारा

(b) इं टरने शनल टे लीकम्यु निकेशन यूनियन के द्वारा

(c) इं टरने ट सोसायटी के द्वारा

(d) इं टरने शनल साइबर सिक्योरिटी प्रोटे क्शन अलायं स के द्वारा

Q17.B

 वै श्विक साइबर सु रक्षा सूचकांक (GCI) साइबर सु रक्षा जागरूकता बढ़ाने और साइबर सु रक्षा
के लिए दे शों की प्रतिबद्धता तथा उद्योगों एवं विभिन्न क्षे तर् कों में इसके अनु पर् योग की
व्यापकता के आकलन हे तु एक बहु-हितधारक पहल है ।
o इसे अं तर्राष्ट् रीय दरू सं चार सं घ (इं टरने शनल टे लीकम्यु निकेशन यूनियन) द्वारा जारी किया
जाता है जो सं युक्त राष्ट् र की एक विशे षीकृत एजें सी है । यह एजें सी सूचना एवं सं चार
प्रौद्योगिकियों से सं बंधित सभी मामलों के लिए उत्तरदायी है ।
 वै श्विक साइबर सु रक्षा सूचकांक (GCI) 2020 में भारत को 10 वां स्थान प्रदान किया गया है ।
एशिया प्रशांत क्षे तर् में भारत चौथे स्थान पर है ।
 सं युक्त राज्य अमे रिका प्रथम स्थान हासिल करके सूची में शीर्ष पर है , जिसके बाद ब्रिटे न
ू रे स्थान पर हैं ।
और सऊदी अरब एक साथ दस
 वै श्विक साइबर सु रक्षा सूचकांक की रैं किंग पांच स्तं भों पर आधारित है :
o कानूनी उपाय
o तकनीकी उपाय
o क्षमता निर्माण के उपाय
o सं गठनात्मक उपाय
o सहयोग
 इसलिए विकल्प (b) सही उत्तर है ।

18. "यह एक ग्रीनहाउस गै स है । यह धान के खे तों से , कोयले की खदानों से और कचरे के ढे रों के सड़ने
से बड़ी मात्रा में निकलती है । यह उन रासायनिक अभिक्रियाओं से सं बंधित है जिनके कारण ओजोन
का अवक्षय होता है । इसका उपयोग बायोगै स के रूप में भी किया जाता है ।" यह गै स है :

(a) नाइट् रस ऑक्साइड

(b) मीथे न

(c) क्लोरोफ्लोरोकार्बन

(d) कार्बन डाइऑक्साइड

Q18.B

 मीथे न एक ऐसी गै स है जो पृ थ्वी के वायु मंडल में अल्प मात्रा में पाई जाती है । मीथे न सबसे
सरल हाइड्रोकार्बन है जिसमें एक कार्बन परमाणु और चार हाइड्रोजन परमाणु होते हैं । मीथे न
एक शक्तिशाली ग्रीनहाउस गै स है । मीथे न एक ज्वलनशील गै स है और इसका उपयोग
दुनिया भर में ईंधन के रूप में किया जाता है ।
 यद्यपि पृ थ्वी के वायु मंडल में मीथे न की सांदर् ता कम है (लगभग 1.8 भाग प्रति मिलियन),
किंतु यह एक महत्वपूर्ण ग्रीनहाउस गै स है क्योंकि यह एक शक्तिशाली ऊष्मा अवशोषक है ।
जब ऑक्सीजन की अनु पस्थिति में वनस्पति को जलाया जाता है , उसका पाचन होता है या वह
सड़ती है तो मीथे न प्राकृतिक रूप से उत्पन्न होता है । मीथे न के अन्य प्रमु ख प्राकृतिक
स्रोतों में आर्द्रभूमियों और महासागरों से तथा दीमक की पाचन प्रक्रियाओं से होने वाले
उत्सर्जन शामिल हैं । इसके मानव गतिविधियों से सं बंधित स्रोतों में चावल उत्पादन, भूमि
भराव स्थल, मवे शी और अन्य जु गाली करने वाले जानवरों का पालन तथा ऊर्जा उत्पादन
शामिल हैं ।
 मीथे न और क्लोरीन परमाणु ओं की अभिक्रिया क्लोरीन (CI) परमाणु ओं के प्राथमिक सिं क के
रूप में कार्य करती है और समताप मं डल में हाइड्रोक्लोरिक अम्ल (HCI) का एक प्राथमिक
स्रोत है ।
o CH4 + Cl → CH3 + HCI
o इस अभिक्रिया में उत्पादित HCL समताप मं डल में ओजोन के अवक्षय के लिए उत्प्रेरक
की तरह कार्य करता है ।
 बायोगै स का उत्पादन तब होता है जब कार्बनिक पदार्थ (पादप और पशु उत्पाद) ऑक्सीजन
मु क्त वातावरण में जीवाणु ओं विघटित किए जाते हैं , इस प्रक्रिया को अवायवीय पाचन
(anaerobic digestion) कहा जाता है । बायोगै स प्रणालियों में इन कार्बनिक पदार्थों का
पु नर्चक् रण करने और उन्हें बायोगै स में बदलने के लिए अवायवीय पाचन का उपयोग किया
जाता है । इसके उत्पाद में ऊर्जा (गै स), और मूल्यवान मृ दा उत्पाद (तरल और ठोस) दोनों होते
हैं । बायोगै स में लगभग 50-70 प्रतिशत मीथे न, 30-40 प्रतिशत कार्बन डाइऑक्साइड और
अन्य गै सों की अल्प मात्रा होती है ।
 इसलिए विकल्प (b) सही उत्तर है ।

19. एक सां विधि अधिनियम, 1972 के तहत राष्ट् रीय वन्यजीव बोर्ड के सं दर्भ में , निम्नलिखित कथनों
पर विचार कीजिए:

1. यह वन्यजीव गठित निकाय है ।

2. अध्यक्षता पर्यावरण, वन और जलवायु वर्तन मं तर् ालय (MoEFCC) के प्रभारी मं तर् ी द्वारा की
जाती है ।

उपर्युक्त कथनों में से कौन-सा/से सही है /हैं ?

(a) केवल 1

(b) केवल 2

(c) 1 और 2 दोनों

(d) न तो 1, न ही 2

Q19.A

 राष्ट् रीय वन्यजीव बोर्ड (NBWL) वन्यजीव सं रक्षण अधिनियम, 1972 के तहत गठित एक
सां विधिक निकाय है । यह उल्ले ख करना महत्वपूर्ण है कि वन्यजीव सं रक्षण अधिनियम, जो
मूल रूप से 1972 में अधिनियमित किया गया था, में NBWL के लिए प्रावधान नहीं था। वर्ष
2002 में वन्यजीव सं रक्षण अधिनियम में सं शोधन के माध्यम से ही NBWL का गठन किया
गया था। यह एक 47 सदस्यीय समिति है । इसलिए कथन 1 सही है ।
o इस बोर्ड के अध्यक्ष प्रधानमं तर् ी होते हैं । इसलिए कथन 2 सही नहीं है ।
o पर्यावरण, वन और जलवायु परिवर्तन मं तर् ालय (MoEFCC) के प्रभारी मं तर् ी बोर्ड के
उपाध्यक्ष होते हैं ।
o NBWL की स्थायी समिति की अध्यक्षता पर्यावरण, वन और जलवायु परिवर्तन मं तर् ी द्वारा
की जाती है ।
 राष्ट् रीय वन्यजीव बोर्ड के कार्य:
o यह वन्यजीव सं रक्षण और वन्य जीवों तथा वनों के विकास के लिए विभिन्न योजनाओं के
कार्यान्वयन की, ऐसे उपायों के द्वारा जो वह उचित समझे , दे खरे ख करने वाला सर्वोच्च
ू ण के मामलों से निपटने का उत्तरदायित्व नहीं
सलाहकारी निकाय है । इसे पर्यावरण प्रदष
सौंपा गया है ।
o वन्यजीव सं रक्षण को बढ़ावा दे ने और वन्यजीवों के अवै ध शिकार तथा वन्यजीवों उनके
उत्पादों के अवै ध व्यापार को प्रभावी ढं ग से नियं त्रित करने के तरीकों एवं साधनों पर
नीतियां तै यार करना केंद्र सरकार एवं राज्य सरकारों को सलाह दे ना।
o राष्ट् रीय उद्यानों, अभयारण्यों और अन्य सं रक्षित क्षे तर् ों की स्थापना और । करने पर तथा
ऐसे क्षे तर् ों में गतिविधियों को प्रतिबं धित करने से सं बंधित मामलों पर सिफारिशें करना।
o वन्यजीवों या उसके पर्यावास पर विभिन्न परियोजनाओं और गा का प्रभाव मूल्यांकन
करना या करवाना।
o समय-समय पर, दे श में वन्यजीव सं रक्षण के क्षे तर् में हुई प्रगा समीक्षा करना और उसमें
सु धार के उपाय सु झाना।
o दे श में वन्यजीवों पर दो वर्ष में कम से कम एक बार रिपोर्ट (status report) तै यार करना
और प्रकाशित करना।

् को उत्प्रेरित करने में एक जै विक कर्मक


20. निम्नलिखित जीवों में से कौन-सा खे त के पौधों की वृ दधि
के रूप में कार्य करता है ?

1. बद्धहस्त कीट (प्रेइं ग मैं टिस)

2. मरुस्थलीय टिड्डी (डे जर्ट लोकस्ट)

3. कैल्सीड ततै या (कैल्सीड वास्प)


4. सै निक भृ ग (सोल्जर बीटल)

नीचे दिए गए कू ट का प्रयोग कर सही उत्तर चु निए।

(a) 1, 2, 3 और 4

(b) केवल 1, 3 और 4

(c) केवल 1 और 4

(d) केवल 2 और 4

Q20.B

 मरुस्थलीय टिड्डियों (Desert locusts) को खाद्य फसलों, हरी-भरी वनस्पतियों और पादपों को


बड़े पै माने पर नष्ट करने के लिए जाना जाता है । ये लाखों की सं ख्या में प्रजनन करती हैं और
झुंड बनाकर भोजन की तलाश करती हैं । ये अपने मजबूत पं खों और पै रों के कारण बिना रुके
सै कड़ों मील की यात्रा कर सकती हैं । खाद्य एवं कृषि सं गठन (FAO) के अनु सार, एक वर्ग
किलोमीटर में फैला टिड्डियों का झुंड जिसमें लगभग 40 मिलियन टिड्डियां हो सकती हैं , एक
दिन में लगभग 35,000 लोगों के भोजन के बराबर भोजन कर सकता है , यह मानते हुए कि
प्रत्ये क व्यक्ति प्रति दिन 2.3 किलोग्राम भोजन करता है । इसलिए केवल विकल्प 2 सही
नहीं है ।
 बद्धहस्त कीट या प्रेइं ग मैं टिस (Praying mantises) ऐसे किसी भी जीव का शिकार और भक्षण
कर ले ते हैं जिसे वे सफलतापूर्वक पकड़ सकते हैं और खा सकते हैं । शाकाहारी जीवों के साथ-
साथ मांसाहारी जीव भी इनका मु ख्य आहार होते हैं । वे गों, घु न, कीड़ों, पतं गों, तितलियों और
फसलों के बीच रहने वाले विभिन्न कीटों का भक्षण करते हैं । इस प्रकार ये पत्तियों को खाने
वाले विभिन्न कुंडलक एवं अर्द्ध कुंडलक कीटों (leaf-eating looper and semilooper), फल
खाने वाले बोलवर्म कीटों, पत्ती मोड़क और तना छे दक कीटों, कुख्यात मिलीबग, ग्रे वे विल
जै से घु न, चे फर और ब्राउन फ्लावर बीटल जै से भं ग और जै सिड, एफिड्स, हॉपर जै से
नाशीकीटों को नियं त्रित करने के लिए एक प्राकृतिक कीटनाशक के रूप में कार्य करते हैं ।
 कैल्सीड ततै या (Chalcid wasps), चाल्सीडोइडिया कुल के कीट हैं जो हाइमनोप्टे रा गण का
भाग है । इस कुल में लगभग 22,500 ज्ञात प्रजातियां हैं और अनु मानित समग्र विविधता
500,000 से अधिक प्रजातियों की है , जिसका तात्पर्य यह है कि प्रजातियों की विशाल सं ख्या
की खोज और वर्णन किया जाना अभी भी शे ष है । एक समूह के रूप में चाल्सीडोइड्स
सामान्यत: मनु ष्यों के लिए लाभदायक होते हैं और फसलों के विभिन्न कीटों को नियं त्रित
करने में मदद करते हैं । इनकी कई प्रजातियों का जै व नियं तर् ण कर्मकों के रूप में आयात किया
गया है ।
 सै निक भृ ग (Soldier beetles) सामान्य बाहरी कीट हैं जो लार्वा या वयस्क के रूप में प्रचु र
मात्रा में आकस्मिक आक् रमणकारी हो सकते हैं । सै निक ,गों को उनके नरम, कपड़े जै से पं खों
के आवरण जो चमकीले रं ग के होने पर वर्दी के समान प्रतीत होते हैं , के कारण 'ले दरविं ग्स'
(leatherwings) के उपनाम से भी जाता है । सै निक भं ग का जीवन चक् र एक लार्वा के रूप में
आरं भ होता है जो पतझड़ ऋतु के दौरान अं डे से निकलता है । ये लार्वा परभक्षी होते हैं और
कई उद्यान कीटों के अं डों का भक्षण कर ले ते हैं , साथ कीटों के लार्वा और कोमल शरीर वाले
कीटों को क्षति पहुंचाते हैं । तत्पश्चात वे वसं त ऋतु के आगमन तक मृ दा में या गिरे हुए पत्तों
के बीच शीतनिष्क्रियता की अवस्था में चले जाते हैं । इसलिए विकल्प (b) सही उत्तर है ।

21. इन्हें "महासागरीय वर्षा वनों" के रूप में वर्णित किया जाता है । ये जल स्तर से ऊपर जीवित नहीं
रह सकते हैं तथा अधिकां शतः पृ थ्वी के 25 डिग्री उत्तरी और दक्षिणी अक्षां शों के मध्य सीमित हैं । ये
सामान्यतः जलमग्न द्वीपों के अन्तःजलीय भागों से जु ड़े होते हैं । उपर्युक्त परिच्छे द द्वारा निम्नलिखित
में से किसका सर्वश्रेष्ठ वर्णन किया गया है ?

(a) पादप प्लवक

(b) मै न्ग्रोव

(c) प्रवाल भित्ति

(d) समु दर् ी घास

Q21.C

 प्रवाल भित्तियों को यहां विद्यमान जीवन की अत्यधिक प्रचु रता के कारण प्राय:
"महासागरीय वर्षावनों (Rainforests of the sea) के रूप में भी वर्णित किया जाता है । अपनी
सं रचनात्मक जटिलता के कारण, प्रवाल पृ थ्वी पर सबसे अधिक उत्पादक पारिस्थितिक तं तर् ों
में से एक हैं , जो मानव जाति को मत्स्य सं साधन, तटीय सं रक्षण, औषधियां , मनोरं जन और
पर्यटन सहित महत्वपूर्ण से वाएं प्रदान करते हैं ।
 प्रवाल समूहों में निवास करने वाले छोटे जीव होते हैं तथा जूजैन्थे ली (zooxanthellae)
शै वाल के साथ सहजीवी सं बंध से पोषण और ऊर्जा ग्रहण करते हैं । प्रवाल भित्तियां हजारों
वर्षों की अवधि के दौरान प्रवालों द्वारा बनी चूना पत्थर की सं रचनाओं के सं चित होने से
निर्मित होती हैं और जीवित प्रवालों के लिए एक सं रचनात्मक आधार प्रदान करती हैं ।
वै ज्ञानिकों का अनु मान है कि ये भित्तियां चटकीले रं गों वाली उष्णकटिबं धीय मछलियों से
ले कर कैंसर रोधी यौगिकों का उत्पादन करने वाले सी कुकुम्बर तक, लाखों प्रजातियों को
आवास स्थल प्रदान करती हैं । अत्यधिक सं गठित भित्तियों का निर्माण केवल तभी होता है
जब तापमान विस्तारित अवधि के लिए 18 डिग्री से ल्सियस से नीचे नहीं गिरता है । उच्च
प्रकाश के प्रवे श के लिए स्वच्छ जल होना अनिवार्य है । प्रवालों की उच्च प्रकाश सं बंधित
आवश्यकता यह भी वर्णित करती है कि अधिकां श भित्ति-निर्माण प्रजातियां यूफोटिक
(प्रकाश प्रवे श) जोन (लगभग 70 मीटर) तक ही सीमित क्यों हैं । इस प्रकार के कठोर
पर्यावरणीय प्रतिबं धों के कारण ही प्रवाल भित्तियां सामान्यतः उष्णकटिबं धीय और अर्द्ध-
उष्णकटिबं धीय जल तक ही सीमित होती हैं ।
 प्रवालों की विविधता, अर्थात प्रजातियों की सं ख्या, उच्च अक्षां शों की ओर लगभग 25°
उत्तर और दक्षिण तक घटती जाती है , जिसके आगे सामान्यतः प्रवाल नहीं पाए जाते हैं । 32°
उत्तरी अक्षां श पर स्थित बरमूडा, इस नियम का अपवाद है क्योंकि यह उष्ण जलधारा गल्फ
स्ट् रीम के मार्ग में अवस्थित है ।
 इसलिए विकल्प (c) सही उत्तर है

ू ण के उपशमन हे तु निम्नलिखित में से कौन-सा/से उपाय किया जा सकता है /किए जा


22. मृ दा प्रदष
सकते हैं ?

1. से नेटरी लैं डफिल


2. कीटनाशकों के उपयोग में वृ दधि

3. कम्पोस्टिं ग

नीचे दिए गए कू ट का प्रयोग कर सही उत्तर चु निए।

(a) केवल 1 और 2

(b) केवल 2

(c) केवल 1 और 3

(d) उपर्युक्त में से कोई नहीं

Q22.C

 विकल्प 1 सही है । एक सै निटरी लैं डफिल (Sanitary landfill) सार्वजनिक स्वास्थ्य के लिए
बाधा उत्पन्न किए बिना किसी स्थल पर कचरे का निपटान करने की एक पद्धति है । इसके
अं तर्गत अपशिष्ट को एक स्थल पर डाल दिया जाता है तथा इसमें कृन्तकों या कीड़ों/कीटों के
प्रवे श को अवरोधित करने हे तु मृ दा से ढक दिया जाता है । तत्पश्चात अपशिष्ट को
जीवाणु ओं द्वारा अपघटित किया जाता है ।
 ् मृ दा के लिए हानिकारक होती है और
विकल्प 2 सही नहीं है । कीटनाशकों के उपयोग में वृ दधि
ू ण होता है । इसके स्थान पर, कीटनाशकों की आवश्यकता को कम करने के
इससे मृ दा प्रदष
लिए कीट नियं तर् ण की जै विक पद्धतियों का उपयोग किया जा सकता है ।
 विकल्प 3 सही है । कम्पोस्टिं ग अपशिष्ट (Composting waste), ठोस अपशिष्ट को विघटित
करने की एक वायवीय विधि है । इसमें अपशिष्ट का ह्यम
ू स के रूप में अपघटन किया जाता है
जिसे कम्पोस्ट (खाद) कहा जाता है । यह पादपों के लिए एक उत्तम उर्वरक के रूप में कार्य
करता है । सूक्ष्मजीव कार्बनिक पदार्थों को स्थिर करने में सहायता करते हैं । उदाहरणस्वरुप
अपशिष्ट को डालने के पश्चात् कवक प्रथम सप्ताह में अपघटन कार्य प्रारं भ कर दे ते हैं ।
एक्टीनोमाइसीट् स (Actinomycetes) अपघटन के अं तिम चरण में सहायता करते हैं ।
बै क्टीरिया इस सं पर्ण
ू प्रक्रिया के दौरान उपस्थित रहते हैं ।

23. पारिस्थितिक तं तर् के सं दर्भ में , निम्नलिखित में से कौन- सा 'स्थायी अवस्था (standing state)' का
सही वर्णन करता है ?

(a) पारिस्थितिक तं तर् में किसी निश्चित समय पर उपस्थित शु ष्क बायोमास की मात्रा।

(b) पारिस्थितिक तं तर् में सूक्ष्मजीवों की प्रजातियों की कुल सं ख्या।

(c) पारिस्थितिक तं तर् में पाए जाने वाले अकार्बनिक पोषक तत्वों की मात्रा।

(d) पारिस्थितिक तं तर् की सीमाओं के भीतर प्राप्त वार्षिक वर्षा की मात्रा।

Q 23.C

 स्थायी अवस्था (Standing State): इसे पारिस्थितिक तं तर् में पाए जाने वाले अकार्बनिक
पोषक तत्वों जै से कार्बन, नाइट् रोजन, फास्फोरस, कैल्शियम आदि की मात्रा के रूप में
सं दर्भित किया जाता है । इसलिए कल्प (C) सही उत्तर है ।
 यह निर्जीव पदार्थ के भाग का प्रतिनिधित्व करती है ।
 यह विभिन्न प्रकार के पारिस्थितिक तं तर् ों में भिन्न होती तथा मौसम पर होती है ।
 ् के माध्यम में निर्मित होती है । यह तं तर् की उत्पादकता
यह सामान्यतः उत्पादकों की वृ दधि
को निर्धारित करती है ।
 यह पारिस्थितिकी तं तर् के सजीव और निर्जीव घटकों के बीच सं चारित है ।
 इसका नियमित रूप से ह्रास होता है तथा जीवित पदार्थों द्वारा पु नः पूर्ती की जाती है ।
 खड़ी फसल (Standing crop) किसी पारिस्थितिकी तं तर् में विद्यमान बायोमास की मात्रा
होती है । यह सं पर्ण
ू जीवित पदार्थों का प्रतिनिधित्व करती है ।
24. निम्नलिखित में से कौन-सा/से मरुस्थलीय पारिस्थितिक तं तर् के पौधों और जं तुओं में पाए जाने
वाले अनु कूलन गु ण हैं /हैं ?

1. मोम की तरह सख्त और स्थूल पत्ते

2. लं बा तथा विस्तृ त जड़ तं तर्

3. सं केंद्रित कायिक वसा

4. अतिसक्रिय स्वे द ग्रंथियां

नीचे दिए गए कू ट का प्रयोग कर सही उत्तर चु निए।

(a) केवल 2 और 4

(b) केवल 1 और 3

(c) केवल 1, 2 और 4

(d) केवल 1, 2 और 3

Q 24.D

 मरुस्थलीय पौधों को अपने पर्यावास में जीवित रहने के लिए जल ग्रहण करने के विभिन्न
पद्धतियों का विकास करना पड़ा। इन परिवर्तनों को अनु कूलन कहा जाता है ।
 जड़ों, तनों, पत्तियों या फलों में जल सं गर् हण करना एक सामान्य अनु कूलन है । इस प्रकार से
जल सं गर् हण करने वाले पौधे मांसलोद्भिद (succulents) कहलाते हैं , जिनमें से कैक्टस एक है ।
 कुछ पौधे अपनी जड़ों को अत्यधिक लम्बाई तक विकसित कर ले ते हैं , जो कि भूमिगत जल
तक पहुंचने के लिए धरातल में गहराई तक विस्तृ त हो जाती हैं । अन्य सतह के तत्काल नीचे
एक विस्तृ त जड़ प्रणाली विकसित करते है और दरू तक फैली होती हैं । इन पौधों में कई
छोटी-छोटी जड़ें होती हैं जो वर्षा के समय जल अवशोषित करने में सहायक होती हैं । इसलिए
विकल्प 2 सही है ।
 अन्य मरुस्थलीय अनु कूलन पत्तियों में दृष्टिगत होते है । मरुस्थलीय पौधे अपनी पत्तियों के
आकार या सं रचना के आधार पर अपनी पत्तियों की सतह के माध्यम से जल की क्षति को
सीमित करते हैं ।
 छोटे या कांटेदार पत्तियां शु ष्क गर्मी के सं पर्क में आने वाले सतही क्षे तर् को सीमित कर दे ते हैं ।
चिकनी सतह यु क्त पत्तियां सूर्य की किरणों को परावर्तित कर दे ती हैं , जिससे पत्तियों का
तापमान कम रहता है । मोम जै सी सतह वाली पत्तियां जल के निष्कासन को अवरोधित करती
हैं । इसलिए विकल्प 1 सही है ।
 मरुस्थलीय जीवों द्वारा जीवित रहने के लिए, अनु कूलन हे तु विभिन्न पद्धतियों का विकास
किया गया है । व्यवहारिक रूप में सबसे सामान्य अनु कूलन पौधों या चट् टानों की छाया में
रहना या दिन की गर्मी में धरातल के नीचे छिपना है । अने क मरुस्थलीय जीव निशाचर होते हैं
अर्थात, वे दिन में निष्क्रिय रहते हैं और रात में ठं ड होने पर शिकार करते हैं ।
 ् करती है , इसलिए कुछ मरुस्थलीय जीवों में वसा सं पर्ण
वसा शरीर की ऊष्मा में वृ दधि ू शरीर में
फैले होने के बजाए एक ही स्थान पर, जै से कू बड़ या पूंछ पर सं केंद्रित (अर्थात सं केंद्रित
कायिक वसा) होती है । इसलिए विकल्प 3 सही है ।
 स्वे द ग्रंथियों की अनु पस्थिति और मूतर् का सं केंद्रण मरुस्थलीय जीवों द्वारा किया गया
अन्य शारीरिक अनु कूलन है । इसलिए कथन 4 सही नहीं है ।

25. एक निश्चित क्षे तर् में पगमार्कों की गिनती करके निम्नलिखित में से क्या निर्धारित किया जा सकता
है ?

1. क्षे तर् में विभिन्न प्रजातियों की उपस्थिति

2. विभिन्न प्रजातियों की आयु

3. विभिन्न प्रजातियों का लिं गानु पात

4. विशिष्ट की पहचान

नीचे र कू ट का प्रयोग कर सही उत्तर चु निए।

(a) केवल 1,2 और 4

(b) केवल 3 और 4

(c) केवल 1 और 2

(d) 1, 2, 3 और 4

Q 25.D

 पगमार्क अधिकां श जानवरों के पदचिह्न को सं दर्भित करने के लिए प्रयोग किया जाने वाला
शब्द है । ये वे चिह्न हैं जो विभिन्न जानवरों की प्रजातियों द्वारा चलने , दौड़ने या एक स्थान से
ू रे स्थान पर जाते समय छोड़े जाते हैं । जानवरों की प्रत्ये क प्रजाति का पगमार्क भिन्न
दस
होता है । इसका उपयोग पहचान के उद्दे श्य से प्रयोग किया जाता है । इनका उपयोग किया
जाता है -
o जानवरों की ट् रैकिंग के लिए (विशे षकर बड़ी बिल्लियों के लिए)।
o विभिन्न प्रजातियों की उपस्थिति की सटीक पहचान करने के लिए।
o लिं ग का सटीक निर्धारण के लिए।
o जानवरों की आयु और शारीरिक स्थिति की सटीक पहचान के लिए।
 इसलिए विकल्प (d) सही उत्तर है ।

26. आनु वंशिक जै व विविधता के सं दर्भ में , निम्नलिखित कथनों पर विचार कीजिए:

1. यह विभिन्न प्रजातियों के बीच जीन की भिन्नता को सं दर्भित करता है ।

2. उच्च आनु वंशिक जै व विविधता पर्यावरणीय परिवर्तनों के अनु कूलन में कम लचीले पन की ओर
अग्रसरित होती है ।

उपर्युक्त कथनों में से कौन-सा/से सही है /हैं ?

(a) केवल 1

(b) केवल 2

(c) दोनों 1 और 2

(d) न तो 1, न ही 2

Q26.D

 आनु वंशिक विविधता एक प्रजाति के भीतर पाई जाने वाली विविधता (या आनु वंशिक
परिवर्तनशीलता) को सं दर्भित करती है । प्रत्ये क विशिष्ट प्रजाति में ऐसे जीन विद्यमान होते
हैं जो उसकी अनूठी विशे षताओं के स्रोत होते हैं : उदाहरण के लिए, मनु ष्यों में इनके चे हरे की
व्यापक विविधता प्रत्ये क मनु ष्य की आनु वंशिक विशिष्टता को दर्शाती है । आनु वंशिक
विविधता शब्द के अं तर्गत एक ही प्रजाति की भिन्न समष्टियां भी शामिल होती हैं , जै से
विभिन्न कुत्तों की नस्लें या गु लाब की विभिन्न किस्में । इसलिए कथन 1 सही नहीं है ।
 मनु ष्य आनु वंशिक रूप से होमो से पियन्स समूह से सं बंधित हैं और उनकी जै से ऊंचाई, रं ग,
शारीरिक सं रचना आदि में व्यापक भिन्नताएं होती हैं । ये भिन्नताएं आनु वंशिक विविधता के
कारण होती हैं ।
 विभिन्न जीन समूहों की व्यापक विविधता किसी व्यक्ति या पूरी की किसी भी पर्यावरणीय
कारक के दबाव को सहन करने की क्षमता को परिभाषित करती है । बदलती जलवायु
प्रजातियों की अनु कूलन क्षमता के सं बंध में भी क विविधता महत्वपूर्ण होती है । आनु वंशिक
विविधता जितनी अधिक के विशे ष सं दर्भ में विभिन्न वातावरणों के लिए होती है , पर्यावरणीय
परिवर्तनों के अनु कूल होने भावना उतनी ही अधिक होती है और इस प्रकार प्रजातियों की
उत्तरजीविता की उतनी ही अधिक पु ष्टि होती है । कथन 2 सही नहीं है ।

27. खाद्य शृं खला में ऊर्जा के प्रवाह के सं दर्भ में , निम्नलिखित कथनों पर विचार कीजिए:

1. खाद्य शृं खला में ऊर्जा का प्रवाह सदै व एकदिशीय होता है ।

2. प्रत्ये क पोषण स्तर पर ऊर्जा का ह्रास 10 से 15 प्रतिशत की सीमा में होता है ।

3. पोषण स्तर जितना अधिक होगा उपलब्ध ऊर्जा उतनी ही अधिक होगी। उपर्युक्त कथनों में से कौन
सा/से सही है /हैं ?

(a) केवल 1 और 2

(b) केवल 2 और 3

(c) केवल 1

(d) 1,2 और 3

Q 27.C

 ू रे से पोषण तं तर् या पोषण स्तरों के माध्यम से सं बद्ध होते


पारिस्थितिक तं तर् में जीव एक दस
ू रे जीव के लिए भोजन होता है । जीवों का एक दस
हैं , अर्थात एक जीव किसी दस ू रे को भक्षण
करने का क् रम, एक खाद्य शृं खला का निर्माण करता है ।
 ऊर्जा सभी चयापचय गतिविधियों के लिए उत्तरदायी मूल शक्ति होती है । उत्पादक से शीर्ष
उपभोक्ताओं तक ऊर्जा के प्रवाह को ऊर्जा प्रवाह कहा जाता है जो एकदिशीय होता है ।
इसलिए कथन 1 सही है ।
 बारम्बार भक्षण किए जाने और भक्षण करने वाले जीवों की एक शृं खला के माध्यम से पादपों
द्वारा अपने ऊर्जा स्रोत से खाद्य ऊर्जा का स्थानांतरण, खाद्य शृं खला के रूप में जाना जाता है ।
प्रत्ये क स्थानान्तरण पर भोजन से अत्यधिक ऊर्जा (80-90%) ऊष्मा के रूप में नष्ट हो जाती
है । अत: ऊर्जा का केवल शे ष 10 से 20% ही अगले पोषण स्तर पर स्थानांतरित हो पाता है ।
इसलिए कथन 2 सही नहीं है ।
 इस प्रकार प्रत्ये क पोषण स्तर में पिछले स्तर की तु लना में कम ऊर्जा होती है । उदाहरण के
लिए, बाघ (मांसाहारी) की तु लना में हिरण (शाकाहारी) प्रचु र मात्रा में होते हैं । इसलिए
किसी भी खाद्य शृं खला में चरणों की सं ख्या चार या पांच तक सीमित होती है । जो जीव खाद्य
शृं खला की शु रुआत के जितना करीब होता है , उसे उपलब्ध ऊर्जा उतनी ही अधिक प्राप्त
होती है । साथ ही पोषण स्तर जितना अधिक होगा, वहां उपलब्ध ऊर्जा उतनी ही कम होगी।
इसलिए कथन 3 सही नहीं है ।

28. निम्नलिखित में से कौन-सी विशे षताएं सु पोषी झील से सं बंधित हैं ?

1. पोषक तत्वों की निम्न मात्रा

2. जै व विविधता का ह्रास

3. घु लित ऑक्सीजन की उच्च मात्रा


4. शै वाल की अत्यधिक वृ दधि

नीचे दिए गए कू ट का प्रयोग कर सही उत्तर चु निए।

(a) केवल 2 और 4

(b) केवल 1 और 3

(c) केवल 1, 2 और 4

(d) केवल 2, 3 और 4

28.A

 सु पोषण वह प्रक्रिया है जिसमें एक जल निकाय पोषक तत्वों से अत्यधिक समृ द्ध हो जाता है ,
् होती है । किसी जल निकाय में शै वाल और
जिससे साधारण पादपों की प्रचु र मात्रा में वृ दधि
् (या प्रस्फुटन) इस प्रक्रिया के सं केतक हैं । सु पोषण, एक गं भीर
प्लवक की अत्यधिक वृ दधि
पर्यावरणीय चिं ता का कारण माना जाता है क्योंकि इसके परिणामस्वरूप प्रायः जल की
गु णवत्ता में गिरावट आ जाती है और जल निकायों में घु लित ऑक्सीजन का अवक्षय हो जाता
है । सु पोषित जल अं ततः "मृ त क्षे तर् " बन सकते हैं जो जीवन का समर्थन करने में असमर्थ होते
हैं ।
 ू क तथा सु पोषण प्रक्रिया:
जल में रासायनिक प्रदष
o चूंकि जल एक उत्कृष्ट विलायक होता है , अतः जल में घु लनशील अकार्बनिक रसायन,
ू कों का एक
जिनमें कैडमियम, पारा, निकेल आदि जै सी भारी धातु एं शामिल हैं , प्रदष
ू त जल में पाए जाने वाले प्रदष
महत्वपूर्ण वर्ग निर्मित करते हैं । प्रदषि ू कों में कार्बनिक
ू त
रसायन पदार्थ एक अन्य वर्ग हैं । पे ट्रोलियम उत्पाद जल के अने क स्रोतों को प्रदषि
करते हैं उदाहरण के लिए, महासागरों में प्रमु ख ते ल रिसाव की घटनाएं । गं भीर प्रभाव
वाले अन्य कार्बनिक पदार्थों में कीटनाशक आते हैं जो स्प्रे/फुहार या भूमि से अपवाहित
हो कर जल निकाय में प्रवाहित होते हैं । विभिन्न औद्योगिक रसायनों जै से
पॉलीक्लोरिने टेड बाई फिनाइल (PCBs), जो सफाई विलायकों, डिटर्जेंट और उर्वरकों के
ू कों की सूची में शामिल किया जाता है ।
रूप में उपयोग किए जाते हैं , को भी जल प्रदष
PCBs को कैंसर रोग के कारक के रूप में माना जाता है ।
o वर्तमान में उपलब्ध अधिकां श डिटर्जेंट जै व निम्नीकरणीय हैं । हालां कि, इनका उपयोग
अन्य समस्याएं उत्पन्न कर सकता है । जै व निम्नीकरणीय डिटर्जेंट के निम्नीकरण के लिए
् करते हैं । सं वृदधि
जीवाणु उत्तरदायी होते हैं , वे उन पर पोषित होते हैं और ते जी से वृ दधि ्
के दौरान, वे जल में घु लित समस्त ऑक्सीजन का उपयोग कर सकते हैं । ऑक्सीजन की
कमी जलीय जीवन के अन्य सभी स्वरूपों जै से मछलियों और पादपों को नष्ट कर दे ती है ।
उर्वरकों में फॉस्फेट योज्य के रूप में मौजूद होते हैं । जल में फॉस्फेट मिलाने से शै वाल
् में बढ़ोत्तरी होती है । शै वाल की प्रचु र वृ दधि
वृ दधि ् जल की सतह को आच्छादित कर
ले ती है तथा जल में ऑक्सीजन की सांदर् ता को कम कर दे ती है । यह आमतौर पर
हानिकारक क्षयित पदार्थों तथा मृ त जीवों क सं चय के साथ अवायवीय परिस्थितियों के
लिए अग्रणी है । इस प्रकार, शै वाल प्रस्फुटित-पर्याक् रान्त जल, जल निकाय में अन्य
जीवों के विकास को अवरुद्ध करता है । यह प्रक्रिया जिसमें पोषक तत्वों से जल निकाय
सघन पादपों की समष्टि का समर्थन करते हैं , जो जलीय प्राणी जीवन को ऑक्सीजन से
वं चित दे ते हैं तथा इसके परिणामस्वरूप जै व विविधता के ह्रास को सु पोषण के रूप में
जाना जाता है ।
o इसलिए विकल्प (a) सही उत्तर है ।

29. निम्नलिखित में से किस अधिनियम के अं तर्गत भारतीय जीव-जं तु कल्याण बोर्ड की स्थापना की
गई थी?

(a) वन्य जीव (सं रक्षण) अधिनियम, 1972

(b) जै व विविधता अधिनियम, 2002

(c) पशु ओं के प्रति क् रूरता का निवारण अधिनियम, 1960


(d) पर्यावरण (सं रक्षण) अधिनियम, 1986

Q29.C

 भारतीय जीव-जं तु कल्याण बोर्ड (Animal Welfare of India) दे श में पशु कल्याण से सं बंधित
कानूनों पर एक सां विधिक सलाहकार निकाय है और यह दे श में पशु कल्याण को बढ़ावा दे ता
है । इसका मु ख्यालय हरियाणा राज्य के बल्लभगढ़ तथा डे यरी मं तर् ालय को परामर्श दे ने वाला
एक सां विधिक सलाहकार निकाय है । इसका मु ख्यालय पहले चे न्नई में स्थित था।
 इसकी स्थापना वर्ष 1962 में पशु ओं के प्रति क् रूरता का निवारण अधिनियम (Prevention of
Cruelty to Animals Act), 1960 की धारा4 के अं तर्गत की गई थी।
 भारतीय पशु -कल्याण बोर्ड का आरम्भ प्रसिद्ध मानवतावादी स्वर्गीय श्रीमती रुक्मिणी दे वी
अरं डेल के ने तृत्व में किया गया था। दे श में पशु कल्याण कानूनों का लगन पूर्वक पालन से
ले कर पशु कल्याण सं गठनों को अनु दान प्रदान करने और पशु -कल्याण के मु द्दों पर भारत
सरकार को परामर्श दे ने तक, यह बोर्ड पिछले 50 वर्षों से दे श में पशु -कल्याण आं दोलन का
चे हरा रहा है ।
 इस बोर्ड में 28 सदस्य होते हैं । इन सदस्यों का कार्यकाल 3 वर्ष की अवधि के लिए होता है ।
 पशु ओं के प्रति क् रूरता का निवारण अधिनियम, 1960:
o यह पशु ओं पर अनावश्यक पीड़ा या यातना की प्रवृ त्ति को रोकने हे तु है ।
o यह पशु ओं को अनावश्यक क् रूरता और यातना दे ने पर दं ड का प्रावधान करता है ।
o यह पशु ओं एवं पशु ओं के विभिन्न प्रकारों को परिभाषित करता है ।
o यह क् रूरता के विभिन्न रूपों, अपवादों और एक पीड़ित जानवर की हत्या पर चर्चा करता
है यदि उसके खिलाफ कोई क् रूरता की गई है , ताकि उसे और पीड़ा से मु क्त किया जा सके।
o यह वै ज्ञानिक उद्दे श्यों के लिए, पशु ओं पर प्रयोग से सं बंधित दिशा-निर्दे श का प्रावधान
करता है ।
o यह करतब दिखाने वाले पशु ओं की प्रदर्शनी से सं बंधित प्रावधानों और करतब दिखाने
वाले पशु ओं के विरुद्ध किए जाने वाले अपराधों से सं बंधित प्रावधानों को सु निश्चित
करता है ।
o इस अधिनियम के विरुद्ध किसी अपराध के लिए अभियोजन उस अपराध के किए जाने की
तारीख से तीन मास के अवसान के पश्चात् सं स्थित नहीं किया जाएगा।
o इसलिए विकल्प (c) सही उत्तर है ।

30. महानं दा नदी, जो गं गा नदी की एक महत्वपूर्ण सहायक नदी है , किन राज्य/राज्यों से प्रवाहित
होती है :
1. बिहार से

2. उत्तर प्रदे श से

3. पश्चिम बं गाल से

नीचे दिए गए कू ट का प्रयोग कर सही उत्तर चु निए।

(a) केवल 1

(b) केवल 1 और 3

(d) 1, 2 और 3

Q30.C

 ू ण को ले कर राष्ट् रीय हरित


महानं दा बचाओ समिति ने हाल ही में , महानं दा नदी के प्रदष
अधिकरण (National Green Tribunal :NGT) में एक याचिका दायर की है ।
o ते जी से हो रहे शहरीकरण और अपशिष्ट के निपटान के लिए जगह की कमी ने महानं दा
नदी को कचरे के मै दान (dumpsite) के रूप में परिवर्तित कर दिया है ।
o नदी के किनारे वृ हत पै माने पर अतिक् रमण और भूमि की अवै ध बिक् री ने महानं दा नदी की
गु णवत्ता की और भी क्षति पहुंचाई है ।
 महानं दा नदी एक पारगमन/सीमापारीय (transboundary) नदी है , जो भारतीय राज्य बिहार
एवं पश्चिम बं गाल से होकर प्रवाहित होती है । यह गं गा नदी की एक महत्वपूर्ण सहायक नदी
है ।
 इसका विस्तार बिहार के पूर्णिया, किशनगं ज, अररिया एवं कटिहार जिलों तथा पश्चिम बं गाल
के मालदा, पश्चिम दिनाजपु र और दार्जिलिं ग जिलों में है ।
 महानं दा पश्चिम बं गाल के दार्जिलिं ग जिले में करसियां ग (Kurseong) के निकट पगलाझोरा
जलप्रपात (Paglajhora falls) से निकलती है । उद्गम स्थान से प्रवाहित हुआ स्वच्छ और
ू त हो
शु द्ध जल चं पासारी (Champasari) में सिलीगु ड़ी में प्रवे श करते ही गं दा और प्रदषि
जाता है ।
 महानं दा नदी 360 किलोमीटर प्रवाहित होने के उपरांत, बां ग्लादे श के गोदागरी घाट
(Godagari Ghat) पर गं गा नदी में मिल जाती है ।
 महानं दा नदी की मु ख्य सहायक नदियाँ हैं बालासन (Balason), मे ची (Mechi) इसलिए
विकल्प (c) सही उत्तर है ।
31. निम्नलिखित में से कौन-से कारण मरुस्थलीकरण के लिए उत्तरदायी हैं ?

1. वनस्पति आवरण का अभाव

2. जल-अपरदन

3. कर्तन दहन प्रणाली (slash and burn) कृषि

4. अतिचारण

नीचे दिए गए कू ट का प्रयोग कर सही उत्तर चु निए।

(a) केवल 1, 2 और 3

(b) केवल 2 और 4

(c) केवल 1, 3 और 4

(d) 1, 2, 3 और 4

Q31.D

 मरुस्थलीकरण भूमि की जै विक क्षमता में ह्रास या विनाश को करता है , जिससे मरुस्थल जै सी
स्थितियां उत्पन्न होती हैं ।
 वानस्पतिक आवरण का अभाव, पवन एवं जल-अपरदन, अतिचारण, कर्तन दन प्रणाली (slash
and burn) कृषि मरुस्थलीकरण का कारण बनती हैं ।
 मरुस्थलीय भूमि पर सार्थक कृषि गतिविधियाँ या क्रियाकलाप सं भव नहीं हैं । मरुस्थलीकरण
से भूमि का क्षरण होता है , जिससे मृ दा अनु पजाऊ हो जाती है ।
 इसलिए विकल्प (d) सही उत्तर है ।

32. भारतीय वन अधिनियम, 1927 के सं दर्भ में , निम्नलिखित कथनों पर विचार कीजिए:

1. यह वन-उपज के अभिवहन और वन-उपज पर उद्ग्रहणीय शु ल्क को विनियमित करता है ।

2. यह किसी क्षे तर् को आरक्षित वन घोषित करने के लिए अपनाई जाने वाली प्रक्रिया की व्याख्या
करता है ।

3. राज्य सरकार किसी भी ग्राम समु दाय को आरक्षित वन की किसी भी भूमि पर सरकार के अधिकार
सौंप सकती है ।
उपर्युक्त कथनों में से कौन-सा/से सही है /हैं ?

(a) केवल 3

(b) केवल 1 और 3

(c) केवल 1 और 2

(d) 1, 2 और 3

Q32.D

 भारतीय वन अधिनियम (Indian Forest Act), 1927 का उद्दे श्य वनों की सु रक्षा एवं सं रक्षण
तथा वन उत्पादों का न्यायसं गत उपयोग था। इसका उद्दे श्य वन-उपज के अभिवहन और
वनोपज के उद्हणीय शु ल्क को विनियमित करना था। इसलिए कथन 1 सही है ।
 यह अधिनियम किसी क्षे तर् को आरक्षित वन, सं रक्षित वन या ग्राम वन घोषित करने के लिए
अपनाई जाने वाली प्रक्रिया की व्याख्या करता है । इसलिए कथन 2 सही है ।
o आरक्षित वन: आरक्षित वन सबसे प्रतिबं धित वन होते हैं और राज्य सरकार द्वारा ऐसी
किसी वन भूमि या बं जर-भूमि को, जो सरकार की सं पत्ति है , में गठित किए जाते हैं ।
आरक्षित वनों में , स्थानीय लोगों को किसी वन अधिकारी द्वारा व्यवस्थापन के दौरान,
विशे ष रूप से अनु मति के बिना आवाजाही प्रतिबं धित होती है ।
o सं रक्षित वन: राज्य सरकार किसी वन भूमि, जो आरक्षित वन में सम्मिलित नहीं है किंतु जो
सरकार की सं पत्ति है या जिस पर सरकार का साम्पत्तिक अधिकार है , उन वन भूमि को
सं रक्षित वनों के रूप में गठित करने का अधिकार रखती है । साथ ही, ऐसे वनों के उपयोग
के सं बंध में नियम जारी करने का अधिकार भी रखती है । इस अधिकार का उपयोग वृ क्षों पर
राज्य नियं तर् ण स्थापित करने के लिए किया गया है । ये ऐसे वृ क्ष होते हैं जिनकी लकड़ी,
फल या अन्य गै र-लकड़ी उत्पादों में राजस्व बढ़ाने की क्षमता होती है ।
o ग्राम वन: ग्राम वन ऐसे वन होते हैं जिसमें राज्य सरकार किसी ऐसी भूमि के प्रति या
उस पर, जो आरक्षित वन कर दी गई है , उस वन के राज्य अधिकार किसी ग्राम समु दाय
को समनु दिष्ट कर सकेगी। इसलिए कथन 3 सही है ।
 राज्य सरकार सं रक्षित वन में किन्हीं वृ क्षों या वृ क्षों के वर्ग को आरक्षित घोषित कर सकती है ।
 राज्य सरकार किसी वन में पत्थर के उत्खनन या चूने या लकड़ी के कोयले को जलाने या किसी
वन में वन-उपज का सं गर् हण करने या उसे हटाने से प्रतिषे ध कर सकती है ।
ू कों को विषाक्त कणिकीय पदार्थ के रूप में वर्गीकृत किया जा सकता
33. निम्नलिखित में से किन प्रदष
है ?

1. सीसा

2. कैडमियम

3. पारा

4. एस्बे स्टस

5. निकेल

नीचे दिए गए कू ट का प्रयोग कर सही उत्तर चु निए।

(a) केवल 1, 3 और 5

(b) केवल 1, 2, 3 और 4

(c) केवल 1, 4 और 5

(d) 1, 2, 3, 4 और 5

Q 33.D

 ू त करके
विषाक्त कणिकीय (particulate) पदार्थ वे पदार्थ हैं , जो खाद्य एवं जलापूर्ति को सं दषि
लोक स्वास्थ्य के लिए गं भीर खतरा पै दा करते हैं ।
o सीसा (Lead) एक हानिकारक विषाक्त तत्व है और यह बच्चों के मस्तिष्क के विकास को
प्रभावित कर सकता है ।
o एस्बे स्टस (Asbestos) श्वसन रोग पै दा कर सकता है , जिसे एस्बे स्टॉसिस (Asbestosis) के
रूप में जाना जाता है , साथ ही इससे छाती और फेफड़ों का कैंसर भी हो सकता है ।
o पारा (Mercury) एक अत्यधिक विषाक्त रसायन है जो तं त्रिका तं तर् पर हमला करता है ,
जिससे मस्तिष्क क्षतिग्रस्त (brain damage) हो जाता है और मृ त्यु भी हो सकती है ।
o निकेल (Nickel) से श्वसन सं बंधी लक्षण और फेफड़ों के कैंसर का कारण बन सकता है ।
o कैडमियम (Cadmium) हृदयवाहिका सं बंधी रोग और उच्च रक्तचाप का कारण सकता है ।
o आर्सेनिक (Arsenic) एक और विषाक्त रसायन है जिससे कैंसर होने का दाव किया गया है ।
 इसलिए विकल्प (d) सही उत्तर है ।
34. अम्ल वर्षा के सं दर्भ में , निम्नलिखित कथनों में से कौन-सा सही नहीं है ?

(a) अम्ल वर्षा सल्फर और नाइट् रोजन के ऑक्साइड के निर्गमन के परिणामस्वरूप होती है ।

् के लिए आवश्यकपोषक तत्वों को घोलकर पृ थक कर दे ती है ।


(b) अम्ल वर्षा फसलों के वृ दधि

(c) अम्ल वर्षा का pH मान 6 से 7 के मध्य होता है ।

(d) अम्ल वर्षा जल के पाइपों का सं क्षारण करती है जिसके परिणामस्वरूप भारी धातु एं पे यजल में मिल
जाती हैं ।

Q34.C

 अम्ल वर्षा, या अम्ल निक्षे पण एक व्यापक शब्द है जिसमें घटकों, जै से सल्फ्यूरिक या


नाइट्रिक अम्ल के साथ वर्षण का कोई भी रूप सम्मिलित होता है , जो नम या रूप में
वायु मंडल से भूमि पर गिरते हैं । वायु मंडल में उपस्थित कार्बन के फलस्वरूप उत्पन्न H+ आयन
के कारण वर्षा जल का pH सामान्यतः 5.6 होता है । जब वर्षा जल का pH मान 5.6 से कम हो
जाता है तो उसे अम्लीय वर्षा कहते हैं । इसलिए विकल्प (c) सही उत्तर है ।
 अम्ल वर्षा में वायु मंडल से पृ थ्वी की सतह पर अम्ल निक्षे पित हो जाता है । अम्लीय प्रकृति के
नाइट् रोजन एवं सल्फर के ऑक्साइड वायु मंडल में ठोस कणों के साथ हवा में बहकर या तो
ठोस रूप में अथवा जल में द्रव के रूप में , कुहासे या हिम की भां ति निक्षे पित होते हैं ।
 अम्ल वर्षा मानवीय क्रियाकलापों का उपोत्पाद होती है जो वातावरण में सल्फर और
नाइट् रोजन के ऑक्साइड का निर्गमित करती है । जै सा पूर्व में बताया जा चु का है , जीवाश्म
ईंधन [जै से - शक्ति-सं यंतर् ों एवं भट्टियों में कोयला एवं ते ल तथा मोटर इं जनों में डीजल और
पे ट्रोल (जिसमें सल्फर और नाइट् रोजन यु क्त पदार्थ होते हैं )] के दहन से सल्फर
डाइऑक्साइड और नाइट् रोजन ऑक्साइड उत्पन्न होते हैं । SO2 तथा NO2 के ऑक्सीकरण के
ू त
पश्चात जल के साथ अभिक्रिया करके अम्ल वर्षा में प्रमु ख योगदान दे ते हैं , क्योंकि प्रदषि
वायु में सामान्यतः कणिकीय द्रव्य उपस्थित होते हैं , जो ऑक्सीकरण को उत्प्रेरित करते हैं ।
 इसमें अमोनियम लवणों का भी निर्माण होता है , जो वायु मंडलीय धुं ध (एरोसॉल के सूक्ष्म कण)
के रूप में दृश्यमान होते हैं । वर्षा की बूंदों में ऑक्साइड तथा अमोनियम लवण के एरोसॉल
कणों के परिणामस्वरूप नम निक्षे पण (wet deposition) होता है । ठोस तथा द्रव भूमि-सतहों
द्वारा SO2 सीधे अवशोषित हो जाते हैं । इस प्रकार शु ष्क निक्षे पण (dry deposition) होता है ।
 ् के लिए
अम्ल वर्षा कृषि, पे ड़-पौधों आदि के लिए हानिकारक होती है क्योंकि यह इनकी वृ दधि
आवश्यक पोषक तत्वों को घोलकर पृ थक कर दे ती है । यह मनु ष्यों तथा जानवरों में श्वसन
अवरोध उत्पन्न करती है । जब अम्लीय वर्षा सतही जल के साथ बहकर नदियों एवं झीलों तक
पहुँचती है , तो यह जलीय पारिस्थितिकी तं तर् में पौधों और जं तुओं के जीवन को प्रभावित
करती है । अम्ल वर्षा के कारण जल के पाइपों का सं क्षारण होता है , जिससे भारी धातु जै से
लोहा, सीसा और ताँबा घु लकर पे यजल में मिल जाते हैं ।

35. भारत में निम्नलिखित में से किन राज्यों में पवन ऊर्जा के उत्पादन की सं भावनाएं हैं ?

1. आं धर् प्रदे श

2. गु जरात

3. महाराष्ट् र

4. तमिलनाडु

नीचे दिए गए कू ट का प्रयोग कर सही उत्तर चु निए।

(a) केवल 1, 2 और 3

(b) केवल 2 और 3

(c) केवल 3 और 4

(d) 1, 2, 3 और 4

Q 35.D

 भारत में पवन ऊर्जा के उत्पादन की असीम सं भावनाएं हैं । तमिलनाडु, आं धर् प्रदे श, केरल
और कर्नाटक तथा गु जरात एवं महाराष्ट् र के तटवर्ती क्षे तर् ों में पवन की 20 किमी प्रति घं टे से
अधिक की वार्षिक औसत गति वाले वृ हद क्षे तर् उपलब्ध हैं ।
 पवन ऊर्जा का उपयोग भारतीय प्रौद्योगिकीविदों द्वारा समीरा (Samira) नामक लघु वाटर
पम्पिं ग पवन मिलों के निर्माण के माध्यम से भी किया गया है । इसका उपयोग उथले कुओं से
जल निकालने , पं पों के माध्यम से सिं चाई करने , जल को गर्म करने आदि के लिए प्रभावी
ग्रामीण प्रौद्योगिकी के रूप में किया जा सकता है ।
 इसलिए विकल्प (d) सही उत्तर है ।

36. जै व विविधता हॉटस्पॉट के सं दर्भ में , निम्नलिखित कथनों में से कौन-सा/से सही है /हैं ?

1. यह किसी क्षे तर् में वनस्पति और जं तु सं बंधी स्थानिकता का प्रतिनिधित्व करता है ।


2. इसे प्रकृति के लिए विश्वव्यापी निधि (WWF) द्वारा अं तर्राष्ट् रीय प्रकृति सं रक्षण सं घ (IUCN) के
घोषित किया गया है । नीचे दिए गए कू ट का प्रयोग कर सही उत्तर चु निए।

(a) केवल 1

(b) केवल 2

(c) 1 और 2 दोनों

(d) न तो 1, न ही 2

Q 36.A

 जै वविविधता हॉटस्पॉट (Biodiversity Hotspot) की सं कल्पना नॉर्मन मायर्स (Norman


Myers) द्वारा वर्ष 1988 में प्रस्तु त की गई थी। किसी क्षे तर् को जै व विविधता हॉटस्पॉट क्षे तर्
के रूप में अर्हता प्राप्त करने के लिए, दो कठोर मानदं डों को पूरा करना होता है :
o प्रजाति स्थानिकता (Species Endemism)- यहां कम-से -कम 1500 सं वहनी पादप
स्थानीय रूप से विद्यमान होने चाहिए अर्थात, यहां पृ थ्वी पर कहीं और नहीं पाई जाने
वाली वनस्पति का उच्च प्रतिशत होना चाहिए।
o सं कट की श्रेणी (Degree of Threat)- ऐसा क्षे तर् , जिसका 70% से अधिक प्राकृतिक
पर्यावास नष्ट हो चु का हो अर्थात, यह सं कटग्रस्त (Threatened) हो।
 कथन 1 सही है : प्रत्ये क जै व विविधता हॉट स्पॉट ते जी से सं कुचित होते तं तर् में बने रहने के
लिए सं घर्षरत पादपों एवं जं तुओं दोनों, प्रजातियों की असाधारण स्थानिकता के एक जगत
का प्रतिनिधित्व करता है । विश्व में पादपों की 50% से अधिक प्रजातियां और सभी स्थलीय
जं तुओं की 42% 36 जै व विविधता वाले हॉटस्पॉट के लिए स्थानिक हैं ।
 कथन 2 सही नहीं है : कंजर्वेशन इं टरने शनल (Conservation International) 30 से अधिक वर्षों
से अत्याधु निक विज्ञान, धता हॉटस्पॉट को सं रक्षित करने के लिए कार्य कर रहा है । यह लोगों
को प्रकृति (जिस पर विश्व भोजन, ताजे पानी और आजीविका के लिए निर्भर है ।) की रक्षा
करने के लिए भी सशक्त करता है ।

् पर तु षार के प्रभाव के सं दर्भ में , निम्नलिखित कथनों पर विचार कीजिए:


37. पादपों की वृ दधि

1. यह कोशिका में लवणों की सांदर् ता में वृ दधि


् करता है ।

2. यह कैंकर रोग का कारण बनता है ।

उपर्युक्त कथनों में से कौन-सा/से सही है /हैं ?


(a) केवल 1

(b) केवल 2

(c) 1 और 2 दोनों

(d) न तो 1, न ही 2

Q 37.C

 अजै विक या निर्जीव एवं जै विक या सजीव, पारिस्थितिकी तं तर् के दो घटक हैं । ये घटक
पारिस्थितिकी तं तर् और पर्यावरण दोनों के लिए समान ही होते हैं । ये कारक या घटक किसी
जीव की उत्तरजीविता को निर्धारित करते हैं । इनमें से कोई एक एकल कारक या घटक भी जीव
की विभिन्नता (Range) को भी सीमित कर सकता है ।
 अजै विक कारकों में - ऊर्जा, विकिरण, तापमान एवं ऊष्मा का प्रवाह, जल, तु षार वायु मंडलीय
गै सें और वायु , अग्नि, गु रुत्व, स्थलाकृति, मृ दा, भूगर्भिक आधार आदि सम्मिलित हैं ।
 ् पर प्रभाव:
तु षार का पादपों की वृ दधि
o इसके परिणामस्वरूप मृ दा में उपस्थित नमी जम जाती है ।
o लवणों की सांदर् ता और कोशिकाओं के निर्जलीकरण में वृ दधि
् होती है , क्योंकि पादपों के
अं तरकोशिकीय स्थानों में उपस्थित जल जमकर बर्फ हो जाता है । इसलिए कथन 1 सही
है ।
o यह कैंकर (Canker) रोग का कारण बनता है (जीवाणु , कवक एवं विषाणु ओं के कारण समान
लक्षणों वाले अन्य रोग भी हो सकते हैं )। इसलिए कथन 2 सही है ।

38. राष्ट् रीय हरित अधिकरण (NGT) के पास निम्नलिखित में से किन अधिनियमों से सं बंधित विवादों
के निपटान का अधिकार है ?

1. वन्यजीव सं रक्षण अधिनियम, 1972

2. जै व विविधता अधिनियम, 2002s

3. सार्वजनिक दायित्व बीमा अधिनियम, 1991

4. भारतीय वन अधिनियम, 1927

नीचे दिए गए कू ट का प्रयोग कर सही उत्तर चु निए।

(a) केवल 1, 2 और 3
(b) केवल 1 और 4

(c) केवल 2 और 3

(d) 1,2,3 और 4

Q38.C

 राष्ट् रीय हरित अधिकरण अधिनियम, 2010 को राष्ट् रीय अधिकरण नामक एक विशे ष
अधिकरण के गठन हे तु अधिनियमित किया गया था। इसका गठन पर्यावरणीय मु द्दों से ।
मामलों के त्वरित निपटान हे तु किया गया था।
 यह सं विधान के अनु च्छे द 21 के अं तर्गत प्राण स्वतं तर् ता के सं रक्षण के लिए किए गए
प्रावधान से प्रेरणा प्राप्त करता है । अनु च्छे द 21 भारत के नागरिकों को स्वस्थ पूर्ण पर्यावरण
के अधिकार का आश्वासन प्रदान करता है ।
 अधिकरण का पर्यावरणीय मामलों को समर्पित क्षे तर् ाधिकार त्वरित पर्यावरणीय न्याय प्रदान
करता है और उच्चतर न्यायालयों में मु कदमे बाजी के अतिरिक्त भार को कम करने में सहायता
करता है ।
 यह सिविल प्रक्रिया सं हिता, 1908 के तहत निर्धारित प्रक्रिया से बाध्य नहीं है , बल्कि
प्राकृतिक न्याय के सिद्धांतों द्वारा निर्देशित होता है ।
 अधिकरण का क्षे तर् ाधिकार उन सभी दीवानी मामलों पर विस्तारित है जिनमें पर्यावरण से
सं बंधित महत्वपूर्ण प्रश्न (पर्यावरण से सं बंधित किसी भी कानूनी अधिकार के प्रवर्तन सहित)
शामिल हैं । ऐसे प्रश्न अनु सच
ू ी | में निर्दिष्ट अधिनियमों के अधिनियमन से उत्पन्न होते हैं ।
इनमे निम्न शामिल हैं :
ू ण निवारण तथा नियं तर् ण) अधिनियम, 1974;
o जल (प्रदष
ू ण निवारण तथा नियं तर् ण) उपकर अधिनियम, 1977;
o जल (प्रदष
o वन (सं रक्षण) अधिनियम, 1980;
ू ण निवारण तथा नियं तर् ण) अधिनियम, 1981;
o वायु (प्रदष
o पर्यावरण (सं रक्षण) अधिनियम, 1986;
o सार्वजनिक दायित्व बीमा अधिनियम, 1991;
 जै व विविधता अधिनियम, 2002
 इसलिए विकल्प (c) सही उत्तर है ।

39. जब भी वन भूमि को वने तर प्रयोजनों हे तु प्रयोग किया जाता है , तो निम्नलिखित में से किस
अधिनियम के तहत प्रतिपूर्ति वनीकरण अनिवार्य होता है ?
(a) जै व विधता अधिनियम, 2002

(b) वन (सं रक्षण) अधिनियम, 1980

(c) पर्यावरण (सं रक्षण) अधिनियम, 1986

(d) भारतीय वन अधिनियम, 1927

Q39.B

 भारत में , वन भूमि का वने तर प्रयोजन जै से कि बां धों का निर्माण, खनन और अन्य
विकासात्मक गतिविधियां आदि हे तु प्रयोग किया जा सकता है । हालां कि इसके लिए सरकार
की अनु मति अनिवार्य है ।
 चूंकि वन भूमियों को सरे कार्यों में लगाए जाने के परिणामस्वरूप जै व विविधता का ह्रास होता
है । जिसके परिणामस्वरूप वन्यजीवों के साथ-साथ भौगोलिक मानकों जै से जलवायु और
भूभाग भी प्रभावित होते है । चूंकि वन भूमि के वने तर पर्योजन हे तु प्रयोग के परिणामस्वरूप
जै व विविधता की क्षति होती है जो बदले में वन्यजीवों के साथ-साथ जलवायु और भूभाग जै से
भौगोलिक मानकों को प्रभावित करता है । वन (सं रक्षण) अधिनियम, 1980 के तहत
प्रतिपूरक वनीकरण भी अधिदे शित किया गया है इसके तहत उपबं ध किया गया है कि वने तर
प्रयोजन हे तु प्रयु क्त भूमि के बराबर क्षे तर् को प्रतिपूरक वनरोपण के तहत लाया जाएगा।
इसलिए विकल्प (b) सही उत्तर है ।
 इसके अतिरिक्त, वन के सं वर्धन हे तु धन के प्रबं धन का दायित्व वन भूमि के वने तर प्रयोजन
हे तु प्रयोग करने वाले व्यक्ति पर अधिरोपित किया जाता है । यदि व्यवहार्य हो तो वनीकरण
के लिए चयनित भूमि को आरक्षित या सं रक्षित वन के निकट होना चाहिए ताकि विभाग द्वारा
उनका प्रबं धन सफलतापूर्वक किया जा सके।
 प्रतिपूर्ति वनीकरण कोष अधिनियम, केंद्र और प्रत्ये क राज्य व केंद्र शासित प्रदे श दोनों में
एक उपयु क्त सं स्थागत तं तर् की स्थापना का प्रयास करता है । इसके माध्यम से वने तर
प्रयोजन हे तु प्रयु क्त वन भूमि के प्रतिपूर्ति हे तु प्रदत्त राशि का कुशल व पारदर्शी तरीके से
शीघ्र उपयोग सु निश्चित किया जा सकेगा, जिससे वन भूमि के वने तर प्रयोग का प्रभाव कम
होगा।

40. स्वच्छ सर्वेक्षण, 2021 रिपोर्ट के सं दर्भ में ,

निम्नलिखित कथनों पर विचार कीजिए:

1. यह सर्वेक्षण आवासन और शहरी कार्य मं तर् ालय द्वारा आयोजित किया जाता है ।
2. यह सर्वेक्षण ग्रामीण और शहरी दोनों क्षे तर् ों में प्रतिवर्ष आयोजित किया जाता है ।

उपर्युक्त कथनों में से कौन-सा/से सही है /हैं ?

(a) केवल 1

(b) केवल 2

(c) 1 और 2 दोनों

(d) न तो 1, न ही 2

Q40.A

 हाल ही में , आवासन और शहरी कार्य मं तर् ालय द्वारा स्वच्छ सर्वेक्षण, 2021 रिपोर्ट जारी की
गई है । इसलिए कथन 1 सही है ।
 यह स्वच्छ भारत मिशन-शहरी के तहत भारत के शहरों और कस्बों में स्वच्छता, स्वास्थ्य और
सफाई का एक वार्षिक सर्वे क्षण है । वर्ष 2016 में प्रथम सर्वेक्षण किया गया था। इसमें 73 शहरों
को शामिल किया गया था। वर्ष 2021 तक, इसके छठे सं स्करण में , 4320 शहरों को सर्वेक्षण में
शामिल किया गया था, तब इसे विश्व का सबसे बड़ा स्वच्छता सर्वेक्षण कहा गया था।
 आवासन और शहरी कार्य मं तर् ालय ने स्वच्छ भारत मिशन-शहरी के तहत सभी को रै क दे ने के
लिए सर्वेक्षण का छठा सं स्करण आयोजित किया, जिसमें भारतीय गु णवत्ता परिषद (QCI) को
इसके न्वयन भागीदार के रूप में शामिल किया गया था। स्वच्छ सर्वेक्षण के छठे सं स्करण को
रैं किंग अभ्यास के कवरे ज को के लिए और कस्बों और शहरों को समय पर और अभिनव तरीके
से मिशन पहल को सक्रिय रूप से लागू करने हे तु करने के लिए आयोजित किया गया।
 स्वच्छ सर्वेक्षण एक अखिल भारतीय वार्षिक स्वच्छता सर्वेक्षण है जो च्छता, अपशिष्ट
प्रबं धन और समग्र स्वच्छता के आधार पर भारत के शहरों, कस्बों और राज्यों को रैं क प्रदान
करता है । कथन 2 सही नहीं है ।

41. निम्नलिखित में से कौन-सा कथन से लेक्टिव कैटे लिटिक रिडक्शन (SCR) नामक तकनीक का
सबसे अच्छा वर्णन करता है ?

(a) यह एक ऐसी तकनीक है जो NOx को नाइट् रोजन में परिवर्तित करने के लिए अमोनिया का उपयोग
करती है ।

(b) यह एक ऐसी तकनीक है जो हानिकारक कार्बन मोनोऑक्साइड को कार्बन डाइऑक्साइड में


परिवर्तित के लिए अधिक मात्रा में ऑक्सीजन का उपयोग करती है ।
् यों को दरू करने के लिए विद्यु त आवे श का उपयोग करती
(c) यह एक ऐसी विधि है जो वायु में अशु दधि
है ।

(d) यह एक ऐसी तकनीक है जो सल्फर डाइऑक्साइड को सल्फर में परिवर्तित करती है ।

Q41.A

 से लेक्टिव कैटे लिटिक रिडक्शन (SCR) - यह वर्तमान बहुत व्यापक रूप से प्रयोग की जाने
वाली तकनीक है जिसमें एक कन्वर्टर में उत्प्रेरक की उपस्थिति में NOx को नाइटोजन में
परिवर्तित करने हे तु अमोनिया को अपचयित करने वाले एजें ट के रूप में उपयोग किया जाता
है ।
 यह उत्प्रेरक आमतौर पर टाइटे नियम डाइऑक्साइड, वै नेडियम पें टॉक्साइड और टं गस्टन
ट् राइऑक्साइड का मिश्रण होता है । इसलिए विकल्प (a) सही उत्तर है । SCR फ्लू गै सों
(बिजली स्टे शनों और औद्योगिक सं यंतर् ों में ईंधन या अन्य सामग्रियों के जलने से उत्पन्न
गै सों का मिश्रण) से 60-90% तक NOx हटा सकता है ।
 यह प्रक्रिया बहुत महं गी है और इससे सं बंधित अमोनिया अं तःक्षे पण के परिणामस्वरूप
निकास मार्ग में अमोनिया स्लिपस्ट् रीम (वापस सं चालित वायु प्रवाह) उत्पन्न हो जाती है ।

42. ब्लै क कार्बन के सं दर्भ में , निम्नलिखित कथनों पर विचार कीजिए:

1. यह एक गै सीय पदार्थ है जो जीवाश्म ईंधन और बायोमास के अधूरे दहन के परिणामस्वरूप उत्पन्न


होता है ।

2. यह वातावरण में समय की एक छोटी अवधि के लिए रहता है और जलवायु परिवर्तन में योगदान
करता है ।

् होती है ।
3. यह पृ थ्वी की सतह के एल्बिडो को कम करता है जिससे तापमान में वृ दधि

उपर्युक्त कथनों में से कौन-सा/से सही है /हैं ?

(a) केवल 2

(b) केवल 1 और 3

(c) केवल 2 और 3

(d) 1, 2 और 3

Q42.C
 ब्लै क कार्बन एक गै स नहीं बल्कि एक कण है - यह जीवाश्म ईंधन और बायोमास के अधूरे दहन
से उत्सर्जित कालिख (Soot) का एक घटक है । इसलिए कथन 1 सही नहीं है ।
 मीथे न के साथ, ब्लै क कार्बन दो ग्रीनहाउस योगदानकर्ताओं में से एक है , जिसे वै श्विक
समु दाय द्वारा आर्क टिक के पिघलने से पहले ग्लोबल वार्मिं ग को रोकने हे तु तत्काल कम करने
का लक्ष्य रखा गया है ।
 यह कम समय के लिए वातावरण में रहता है और कुछ समय बाद वर्षा के साथ नीचे आता है ।
ब्लै क कार्बन के छोटे वायु मंडलीय जीवन काल के कारण, इसकी उपस्थिति कम होने से ग्लोबल
वार्मिं ग के प्रभावों को कुछ ही हफ्तों के भीतर शीघ्रता से कम करने का अवसर प्राप्त होता
है । इसलिए कथन 2 सही है ।
 कई वै ज्ञानिकों के अनु सार - विशे ष रूप से जीवाश्म ईंधन स्रोतों से निकलने वाले - ब्लै क
कार्बन का नियं तर् ण निकट भविष्य में वै श्विक तापन (Global Warming) को धीमा करने का
सबसे ते ज तरीका हो सकता है ।
 पृ थ्वी के एल्बिडो प्रभाव का हमारी जलवायु पर महत्वपूर्ण प्रभाव पड़ता है । यह सही है ।
किसी सतह का एल्बिडो आपतित सूर्य के प्रकाश का वह अं श है जो सतह से परावर्तित होता
है । एल्बिडो जितना कम होगा, उतना ही अधिक सूर्य से प्राप्त होने वाला विकिरण ग्रह द्वारा
् होगी। यदि एल्बिडो अधिक है , तो
अवशोषित किया जाएगा और इससे तापमान में वृ दधि
पृ थ्वी अधिक परावर्तक होगी, जिससे अधिक विकिरण अं तरिक्ष में वापस चला जाता है और
ग्रह ठं डा हो जाता है ।
 आमतौर पर ब्लै क कार्बन का प्रत्यक्ष तापन प्रभाव (वायु मंडल में आने वाले सौर विकिरण को
अवशोषित करके और इसे ऊष्मा विकिरण में परिवर्तित करने ) तथा अप्रत्यक्ष तापन प्रभाव
(हिम और बर्फ के परावर्तन (एल्बिडो) को कम करने } दोनों रूपों में माना जाता है । इसलिए
कथन 3 सही है ।

43. समु दर् ी स्नॉट(Sea Snot) के सं दर्भ में , निम्नलिखित कथनों पर विचार कीजिए:

1. यह बड़े पै माने पर प्रवास के दौरान व्हे ल द्वारा छोड़ी गई अपशिष्ट सामग्री को सं दर्भित करता है ।

2. यह समु दर् ी पारिस्थितिकी तं तर् के लिए अत्यधिक लाभदायक है ।

उपर्युक्त कथनों में से कौन-सा/से सही है /हैं ?

(a) केवल 1

(b) केवल 2
(c) 1 और 2 दोनों

(d) न तो 1, न ही 2

Q 43.D

 हालिया सं दर्भ: पिछले कुछ महीनों से तु र्की का मरमरा सागर एक नई चु नौती का सामना कर
रहा है । काला सागर को एजियन सागर से जोड़ने वाला मरमरा सागर 'समु दर् ी स्नॉट' (Sea
Snot) से ढक गया है जिससे नई पर्यावरणीय चिं ताएं उत्पन्न हो गई हैं ।
 समु दर् ी स्नॉट को पहली बार वर्ष 2007 में तु र्की में दे खा गया था। इसे 'समु दर् ी कफ' (Marine
Mucilage) के रूप में भी जाना जाता है । यह एक मोटी, चिपचिपा स्ले टी-भूरे रं ग की चादर है
जो मृ त और जीवित कार्बनिक पदार्थों द्वारा बनता है । यह पं क तब बनता है जब शै वाल गर्म
मौसम में पोषक तत्वों से भरपूर होते हैं । यह गर्म मौसम तापन के कारण होता है । इसलिए
कथन 1 सही नहीं है ।
 पर्यावरण विशे षज्ञों ने कहा है कि जलवायु परिवर्तन के कारण पादप-प्लवक अत्यधिक उत्पादन
तथा घरे लू और औद्योगिक कचरे के समु दर् में अनियं त्रित डंपिंग ने वर्तमान सं कट को जन्म
दिया है ।
 समु दर् की सतह पर भूरे रं ग के कफ की तरह तै रने वाली समु दर् ी स्नॉ बढ़ना इस दे श के समु दर् ी
पारिस्थितिकी तं तर् के लिए गं भीर खतरा उत्पन्न कर रहा है । इसके प्रभाव से बड़ी सं ख्या में
लयों की मृ त्यु हुई हैं और अन्य जलीय जीवों जै से प्रवाल और स्पं ज भी मारे गए है । इसलिए
कथन 2 सही नहीं है ।
 पारिस्थितिकीविदों ने कहा है कि तु र्की के समु दर् ों में भूरा कफ इस बात का सं केत है कि समु दर् ी
पारिस्थितिकी तं तर् को कैसे के दोहरे सं कट से निपटने के लिए गं भीर कदम नहीं उठाए गए तो
इसका प्रभाव समग्र पर्यावरण पर पड़ सकता है ।

44. केमिकल ऑक्सीजन डिमांड (COD) के सं दर्भ में , निम्नलिखित कथनों पर विचार कीजिए:

1. यह एक निर्दिष्ट तापमान पर जल में विद्यमान कार्बनिक पदार्थों के विखं डन हे तु सूक्ष्मजीवों द्वारा


आवश्यक ऑक्सीजन की मात्रा का प्रतिनिधित्व करता है ।

2. COD माप का उपयोग एक विशिष्ट स्थान के लिए आवश्यक अपशिष्ट जल उपचार सं यंतर् के
आकार के सं केतक के रूप में किया जाता है ।

उपर्युक्त कथनों में से कौन-सा/से सही है /हैं ?

(a) केवल 1
(b) केवल 2

(c) 1 और 2 दोनों

(d) न तो 1, न ही 2

Q44.B

 केमिकल ऑक्सीजन डिमांड (Chemical Oxygen Demand : COD) एक परीक्षण है जो जल में


मौजूद कार्बनिक पदार्थों और अमोनिया या नाइट् रेट जै से अकार्बनिक पोषक तत्वों को
रासायनिक रूप से ऑक्सीकरण करने के लिए आवश्यक ऑक्सीज की मात्रा को मापता है ।
इसलिए कथन 1 सही नहीं है ।
 पोटे शियम डाइक् रोमे ट सबसे अधिक इस्ते माल किया जाने वाला ऑक्सीकारक (Oxidant) है ,
जिसे उबलते सल्फ्यूरिक एसिड के साथ उपयोग किया जाता है । यह ध्यान रखना महत्वपूर्ण है
कि कार्बनिक या अकार्बनिक यौगिकों के लिए कोई विशिष्ट रासायनिक ऑक्सीकारक नहीं होते
है , इसलिए ऑक्सीजन की मां ग के इन दोनों स्रोतों को COD की कसौटी पर मापा जाता है ।
 COD जल की गु णवत्ता का एक महत्वपूर्ण मापदं ड है और अनु पर् योगों की एक विस्तृ त
श्रखृं ला में इसका उपयोग किया जाता है , जिसमें शामिल हैं ; (क) अपशिष्ट जल डिस्चार्ज और
अपशिष्ट उपचार प्रक्रिया हे तु नियामकों द्वारा निर्धारित मानदं डों को पूरा करने की पु ष्टि
करने में ; (ख) अपशिष्ट जल प्रवाह के बायोडिग्रेडे बल अं श को मापने में ; (ग) बायोकेमिकल
ऑक्सीजन डिमांड (BOD) और केमिकल ऑक्सीजन डिमांड (COD) के बीच का अनु पात मापने
के लिए; (घ) COD या BOD माप का उपयोग किसी विशिष्ट स्थान के लिए आवश्यक
अपशिष्ट जल उपचार सं यंतर् के आकार के सं केतक के रूप में भी किया जाता है । इसलिए
कथन 2 सही है ।
 COD निगरानी से जु ड़ी चु नौतियां -
o इस परीक्षण के विधान में शामिल होने के बावजूद, परीक्षण के उपयोग से जु ड़ी कई समस्याएं
और चु नौतियां हैं : (क) परिणाम उपलब्ध होने में समय लगता है (प्रयोगशाला तक परिवहन +
परीक्षण हे तु 2 घं टे), इसलिए आं कड़े उपलब्ध होने से पहले ही पर्यावरणीय क्षति हो सकती है ;
(ख) यह परीक्षण समय ले ने वाला और महं गा है ; (ग) इस परीक्षण में खतरनाक रसायन शामिल
हैं जिनका सावधानीपूर्वक निपटान करने की आवश्यकता होती है और सं भावित रूप से
इस्ते माल करने वालों के लिए हानिकारक होते हैं ; (घ) यह प्राकृतिक प्रक्रियाओं को फिर से
स्थापित करने में विफल रहता है (अर्थात् परीक्षण में एक मजबूत ऑक्सीकरण एजें ट के साथ
एक कृत्रिम ऊष्मायन शामिल है ); (ङ) यह सटीक नहीं है और इसकी पहचान करने की उच्च
ू त नदी के नमूनों पर लागू नहीं किया जा सकता
न्यूनतम सीमा है , इसलिए यह स्वच्छ/अदषि
है ।
o यह स्पष्ट है कि पारं परिक प्रयोगशाला परीक्षण के स्वस्थाने (in-situ) (रियल टाइम) निगरानी
की ओर बढ़ने से ऊपर उल्लिखित कुछ समस्याओं को कम करने में सहायता मिले गी। ऐसा
इसलिए है क्योंकि COD एक बाह्य स्थाने (ex-situ) में विधि है क्योंकि प्रयोगशाला में नमूने
का परीक्षण किया जाता है ।
o यह तु रंत चु नौतियों के बिं दु (क) से (ग) को सं बोधित करे गा और निगरानी के स्थानिक-
तात्कालिक सं कल्प में सु धार करने में सहायता करे गा, जो सीधे बे सिन प्रबं धकों, जल कंपनियों
और कानून बनाने वालों के लिए समान रूप से लाभदायक होगा।

ू ण के सं दर्भ में , निम्नलिखित कथनों पर विचार कीजिए:


45. आर्सेनिक प्रदष

1. आर्सेनिक में कोई गं ध या स्वाद नहीं होता है ।

2. आर्सेनिक एक कैंसरजनक अभिकर्ता है ।

ू क है ।
3. आर्सेनिक भारत की नदियों में पाया जाने वाला सर्वाधिक सामान्य भारी धातु प्रदष

उपर्युक्त कथनों में से कौन-सा/से सही है /हैं ?

(a) केवल 1

(b) केवल 1 और 2

(c) केवल 2 और 3

(d) 1, 2 और 3

Q45.B

 आर्सेनिक भूपर्पटी का एक प्राकृतिक घटक है जो वायु , जल और भूमि सहित सम्पूर्ण पर्यावरण


में व्यापक रूप से वितरित होता है । अपने अकार्बनिक रूप में यह अत्यधिक विषै ला होता है ।
 ू त जल पीने , उससे भोजन पकाने और खाद्य फसलों की सिं चाई, औद्योगिक प्रक्रियाओं
सं दषि
ू त जल का उपयोग करने , सं दषि
आदि में सं दषि ू त भोजन खाने और धूमर् पान करने से लोग
अकार्बनिक आर्सेनिक के उच्च स्तर के सं पर्क में आते हैं ।
 ू त पे यजल और भोजन के माध्यम से , लं बे समय तक अकार्बनिक आर्सेनिक के
मु ख्य रूप से सं दषि
सं पर्क में रहने से गं भीर एवं चिरस्थायी आर्सेनिक विषाक्तता हो सकती है । त्वचा के घाव और
त्वचा कैंसर इसके सर्वाधिक विशिष्ट प्रभाव हैं । आर्सेनिक में कोई स्वाद या गं ध नहीं होता है
और कोई व्यक्ति अनजाने में ही इसके सं पर्क में आ सकता है । इसलिए कथन 1 सही है ।
 स्वास्थ्य पर पड़ने वाले प्रभाव:
o अं तर्राष्ट् रीय कैंसर अनु संधान एजें सी (IARC) ने आर्सेनिक और आर्सेनिक के यौगिकों को
मनु ष्य के लिए कैंसरजनक के रूप में वर्गीकृत किया है । इसलिए कथन 2 सही है ।
o गं भीर आर्सेनिक विषाक्तता के तात्कालिक लक्षणों में उल्टी, पे ट दर्द और दस्त शामिल हैं ।
इसके पश्चात हाथों और पै रों में सं वेदनशून्यता एवं सिहरन, मांसपे शियों में ऐंठन तथा
अत्यधिक गं भीर मामलों में मृ त्यु भी हो सकती है ।
o लं बे समय तक अकार्बनिक आर्सेनिक के उच्च स्तर के सं पर्क में रहने (उदाहरण के लिए, पे यजल
और भोजन के माध्यम से ) के आरं भिक लक्षण सामान्यतः त्वचा में दिखाई दे ते हैं । इन लक्षणों
में त्वचा के रं ग में परिवर्तन होना, त्वचा में घाव होना तथा हथे लियों एवं पै रों के तलवों पर गहरे
धब्बे होना (हाइपरकेराटोसिस) शामिल हैं । ये लक्षण न्यूनतम लगभग पांच वर्ष तक आर्सेनिक
के उच्च स्तर के सं पर्क में रहने के पश्चात उत्पन्न होते हैं ।
 केंद्रीय जल आयोग (CWC) के अनु सार भारत की नदियों में पाया जाने वाला सर्वाधिक
ू क लोहा है । इसलिए कथन 3 सही नहीं है ।
सामान्य भारी धातु प्रदष

46. पर्यावरण के सं दर्भ में , 'कार्बन मूल्य निर्धारण' का सर्वोत्तम वर्णन है :

(a) किसी क्षे तर् में उत्सर्जनों की उच्चतम सीमा निर्धारित करने हे तु किए गए नीतिगत उपायों के
् ।
प्रत्यक्ष परिणाम के रूप में उस क्षे तर् के बाहर उत्सर्जन में वृ दधि

(b) कार्बनिक पदार्थों में मौजूद कार्बन की मात्रा के सं दर्भ में उन पदार्थों की पर्यावरणीय लागत का
आकलन करने का अभ्यास।

(c) ग्रीनहाउस गै स उत्सर्जनों के लिए करों का भु गतान करने से बचने हे तु उत्सर्जकों द्वारा उपयोग
किया जाने वाला कर चोरी तं तर् ।

(d) बाह्य लागतों को किसी इकाई द्वारा उत्सर्जित उत्सर्जनों के प्रति निर्दिष्ट करके ग्रीनहाउस गै स
उत्सर्जनों से होने वाले नु कसान के बोझ को स्थानांतरित करना।

Q 46.D

 कार्बन मूल्य निर्धारण (Carbon Pricing) एक ऐसा उपकरण है जो ग्रीनहाउस गै स उत्सर्जनों


की बाह्य लागतों को दर्ज करता है । ये उत्सर्जनों की ऐसी लागतें होती हैं जिनका भु गतान
जनता को करना पड़ता है , जै से फसलों का नष्ट होना, ग्रीष्म लहरों और सूखे के कारण
स्वास्थ्य दे खभाल की लागत, बाढ़ और समु दर् -स्तर में वृ दधि
् के कारण सं पत्ति की हानि आदि।
इसके साथ ही, कार्बन मूल्य निर्धारण इन लागतों को एक मूल्य निर्धारित करके इनके स्रोतों से
जोड़ता है । यह मूल्य सामान्यतः उत्सर्जित कार्बन डाइऑक्साइड (CO2) के लिए निर्धारित
मूल्य के रूप में होता है ।
 कार्बन मूल्य निर्धारण ग्रीनहाउस गै स उत्सर्जनों से होने वाले नु कसान के बोझ को वापस उन
लोगों पर स्थानांतरित करने में मदद करता है जो इन उत्सर्जनों के लिए उत्तरदायी हैं और जो
इस उत्तरदायित्व से बच सकते हैं । यह निर्धारित करने के बजाय कि उत्सर्जनों को किसे , कहां
और कैसे कम करना चाहिए, कार्बन मूल्य उत्सर्जकों को एक आर्थिक सं केत प्रदान करता है
और उन्हें यह निर्णय ले ने की अनु मति दे ता है कि वे या तो अपनी गतिविधियों में बदलाव लाएं
और अपने उत्सर्जनों को कम करें अथवा उत्सर्जन करना और अपने उत्सर्जनों के लिए भु गतान
करना जारी रखें । इसलिए विकल्प (d) सही उत्तर है ।
 अन्य अवधारणाएं :
o कार्बन रिसाव (Carbon leakage) एक ऐसी स्थिति है जो तब उत्पन्न हो सकती है जब
कंपनियां , जलवायु सं बंधी नीतियों से जु ड़ी लागतों के कारण, अपने उत्पादन को शिथिल
उत्सर्जन मानकों वाले अन्य दे शों में स्थानांतरित कर दे ती हैं । इससे उनके कुल उत्सर्जनों में
् हो सकती है ।
वृ दधि
o कार्बन कर (Carbon Tax) व्यवसायों और व्यक्तियों पर लगाया जाने वाला एक रूप में कार्य
करता है । यह कर उन कंपनियों पर लगाया जाने वाला शु ल्क कार्बन आधारित ईंधनों, जिनमें
कोयला, ते ल, गै सोलीन और प्राकृतिक गै स शामिल हैं , का दहन करती हैं ।
o कार्बन व्यापार (Carbon Trading) का तात्पर्य कार्बन क् रे डिट के और विक् रय से है । यह किसी
कंपनी या अन्य सं स्था को कार्बन डाइऑक्साइड (CO2) की एक निश्चित मात्रा का करने की
अनु मति दे ता है । कार्बन व्यापार समझौते एक अं तर्राष्ट् रीय समझौते के भाग के रूप में कार्बन
उत्सर्जित करने के लिए विभिन्न दे शों के बीच कार्बन क् रे डिट के विक् रय की अनु मति दे ते हैं ।
इसका उद्दे श्य कुल को धीरे -धीरे कम करना है ।
o कार्बन व्यापार वर्ष 1997 के क्योटो प्रोटोकाल है (जो वर्ष 2005 में लागू हुआ) के तहत शु रू
हुआ था। इसके अं तर्गत 'प्रमाणित उत्सर्जन कटौतियां ' (CERs) उन कंपनियों को जारी की गई
थीं जिन्होंने उत्सर्जन को कम करने वाली परियोजनाओं जै से पवन, सौर या ऊर्जा दक्षता
सं बंधी परियोजनाओं को स्थापित किया था।
o यह उन दे शों को कार्बन क् रे डिट का क् रय करने की अनु मति प्रदान करता है जो अपने कटौती
लक्ष्यों को पूरा करने में असमर्थ हैं । पे रिस समझौता भी अपने राष्ट् रीय स्तर पर निर्धारित
योगदान (NDC) के लक्ष्यों को पूरा करने के लिए दे शों के बीच स्वै च्छिक व्यापार की अनु मति
दे ता है । यदि कोई दे श ग्रीनहाउस गै स उत्सर्जनों में अपने लक्ष्य से अधिक कटौती करता है ,
तो वह इस उत्सर्जन कटौती को किसी अन्य दे श को "अं तर्राष्ट् रीय स्तर पर स्थानांतरित शमन
परिणामों" के रूप में बे च सकता है ।
o न्यूजीलैं ड का शून्य कार्बन कानून: शून्य कार्बन कानून में सभी ग्रीन हाउस गै सों का उत्सर्जन
घटाने का लक्ष्य निर्धारित किया गया है , जबकि वर्ष 2050 तक कार्बन डाइऑक्साइड और
नाइट् रस ऑक्साइड के उत्सर्जन को निवल शून्य तक लाने का लक्ष्य रखा गया है ।

47. यमु ना नदी में एक अभिनव बबल कर्टे न तकनीकी का प्रयोग किया गया था:

(a) गं गा नदी डॉल्फिन के प्रवास की निगरानी करने के लिए।

(b) यमु ना में प्लास्टिक के प्रवे श को रोकने के नदी के मौसमी झाग को हटाने के लिए। यमु ना नदी के
किनारे स्थित विरासत स्थलों को सु शोभित करने के लिए।

(c) यमु ना नदी के मौसमी झाग को हटाने के लिए।

(d) यमु ना नदी के किनारे स्थित विरासत स्थलों को सु शोभित करने के लिए।

Q47.B

 निर्माण समाधान प्रदाता कंपनी लाफार्ज होल्किम (LafargeHolcim) की वै श्विक अपशिष्ट


प्रबं धन शाखा जियोसाइकिल (Geocycle) यमु ना नदी में प्लास्टिक के प्रवे श को रोकने के
लिए भारत में पहली बार अभिनव बबल कर्टे न तकनीकी का प्रयोग कर रही है । इसके तहत
आगरा शहर के मं टोला नाले पर बबल बै रियर स्थापित किया गया है । यह नाला आगरा शहर
के 40% निस्सरण और अपशिष्ट जल का परिवहन करता है ।
 महासागरों में प्लास्टिक के प्रवे श को रोकने के लिए बबल कर्टन एक गै र-आक् रामक समाधान
है । जलयान एवं मछलियां इस तकनीक द्वारा उत्पन्न वायु के बु लबु लों के बीच से होकर गु जर
सकते हैं , ले किन प्लास्टिक रुक जाता है । बबल स्क् रीन का निर्माण विशे ष रूप से डिजाइन की
गई एक वायु ट्यूब द्वारा किया जाता है । इस वायु ट्यूब को नहर या नदी के अधस्तल पर
तिरछा रखा जाता है । इस बबल स्क् रीन से गु जरने पर अपशिष्ट सतह पर आ जाता है और
प्लास्टिक किनारों पर चला जाता है जहां से उसे बाहर निकाला जा सकता है । इसलिए विकल्प
(b) सही उत्तर है ।
 बबल बै रियर सं पीड़ित वायु का उपयोग करके उत्पन्न किया जाता है । यह सं पीडित वायु नहर
के अधस्तल पर स्थापित की गई नलिकाओं या ट्यूब से होकर गु जरती है । ये नलिकाएं एक
वायु सं पीडक (कंप्रेसर) से जु ड़ी होती हैं जो अक्षय सौर ऊर्जा द्वारा सं चालित होता है । इसके
अतिरिक्त, नहर में वायु सं चारण से अपशिष्ट जल/निस्सरण जल में घु लित ऑक्सीजन के स्तर
् होती है जिससे जल की गु णवत्ता में समग्र रूप से सु धार होता है ।
में वृ दधि
48. ब्लू कार्बन पारितं तर् के सं दर्भ में , निम्नलिखित कथनों पर विचार कीजिए:

1. एक तटीय ब्लू कार्बन पारितं तर् की कार्बन प्रच्छादन क्षमता उष्णकटिबं धीय वनों की कार्बन
प्रच्छादन क्षमता से अधिक होती है ।

2. 'ब्लू कार्बन पहल' ब्लू कार्बन पारितं तर् ों को सु रक्षित करने के लिए कंजर्वेशन इं टरने शनल की एक
पहल है ।

उपर्युक्त कथनों में से कौन-सा/से सही है /हैं ?

(a) केवल 1

(b) केवल 2

(c) 1 और 2 दोनों

(d) न तो 1, न ही 2

Q48.C

 तटीय पारितं तर् , जै से मैं ग्रोव, ज्वारीय कच्छ और समु दर् ी घास के क्षे तर् कई लाभ एवं से वाएं
प्रदान करते हैं जो कि विश्व स्तर पर तटीय क्षे तर् ों में जलवायु परिवर्तन के प्रति अनु कूलन के
लिए अनिवार्य हैं । इन लाभों और से वाओं में तूफानों एवं समु दर् के स्तर में वृ दधि
् के विरुद्ध
सु रक्षा, तटरे खा के क्षरण की रोकथाम, तटीय जल की गु णवत्ता को बनाए रखना, व्यावसायिक
रूप से महत्वपूर्ण मत्स्य पालन एवं सं कटग्रस्त समु दर् ी प्रजातियों के लिए पर्यावास और कई
तटीय समु दायों को खाद्य सु रक्षा प्रदान करना शामिल हैं । इसके अतिरिक्त, ये पारितं तर्
वायु मंडल और महासागर से तटीय ब्लू कार्बन का बड़ी मात्रा में प्रच्छादन एवं भं डारण करते
हैं । इसलिए अब इन्हें जलवायु परिवर्तन का शमन करने में अपनी भूमिका के लिए पहचाना
जाता है ।
 अं टार्क टिका को छोड़कर प्रत्ये क महाद्वीप के तटों पर तटीय ब्लू कार्बन पारितं तर् पाए जाते
हैं । मैं ग्रोव उष्णकटिबं धीय और उपोष्णकटिबं धीय तटों के अं तर-ज्वारीय क्षे तर् में उगते हैं ।
मैं ग्रोव के व्यापक क्षे तर् ों वाले दे शों में इं डोने शिया, ऑस्ट् रेलिया, मै क्सिको, ब्राजील,
नाइजीरिया, मले शिया, म्यांमार, पापु आ न्यू गिनी, क्यूबा, भारत, बां ग्लादे श और मोजाम्बिक
शामिल हैं ।
 तटीय ब्लू कार्बन पारितं तर् कार्बन का प्रच्छादन एवं भं डारण करने में बहुत सक्षम होते हैं । इन
पारितं तर् ों का प्रत्ये क वर्ग मील क्षे तर् किसी उष्णकटिबं धीय वन के प्रत्ये क वर्ग मील क्षे तर्
की तु लना में उच्च दर से वायु मंडल और महासागरों से कार्बन को हटा सकता है । इसलिए
कथन 1 सही है ।
 ब्लू कार्बन पहल ऐसा पहला एकीकृत कार्यक् रम है जो वै श्विक स्तर पर तटीय समु दर् ी के
सं रक्षण, पु नर्स्थापन, और सतत उपयोग को बढ़ावा दे कर जलवायु परिवर्तन का शमन करने पर
केंद्रित है । यह वर्तमान में मैं ग्रोव, ज्वारीय कच्छ और समु दर् ी घास पर केंद्रित है । यह
कंजर्वेशन इं टरने शनल, अं तर्राष्ट् रीय प्रकृति सं रक्ष सं घ (IUCN) और यूनेस्को के अं तर-
सरकारी महासागर आयोग का एक अं तर्राष्ट् रीय सहयोगात्मक प्रयास है । इसलिए 2 सही है ।

49. हाल ही में जलवायु परिवर्तन पर सं युक्त राष्ट् र फ् रे मवर्क कन्वें शन (UNFCCC) के पक्षकारों के
26 वें सम्मे लन (COP26) का आयोजन किया गया। इस सं दर्भ में , पक्षकारों के सम्मे लन (COP) से
सं बंधित निम्नलिखित कथनों पर विचार कीजिए:

1. पक्षकारों का सम्मे लन जलवायु परिवर्तन पर सं युक्त राष्ट् र फ् रे मवर्क कन्वें शन का निर्णय ले ने वाला
सर्वोच्च निकाय है ।

2. COP की अध्यक्षता सं युक्त राष्ट् र सु रक्षा परिषद के पांच स्थाई सदस्यों के बीच चक् रीय क् रम में
स्थानांतरित होती रहती है ।

उपर्युक्त कथनों में से कौन-सा/से सही है /हैं ?

(a) केवल 1

(b) केवल 2

(c) 1 और 2 दोनों

(d) न तो 1,

Q49.A

 हालिया सं दर्भ: वर्ष 2021 का सं युक्त राष्ट् र जलवायु परिवर्तन सम्मे लन, जिसे सामान्य तौर
पर COP26 के रूप में जाना जाता है , सं युक्त राष्ट् र का 26 वां जलवायु परिवर्तन सम्मे लन
(UNFCCC) था। इसका आयोजन यूनाइटे ड किंगडम में स्कॉटलैं ड के ग्लासगो में स्कॉटिश
इवें ट कैंपस केंद्र में किया गया था।
 यह जलवायु परिवर्तन पर सं युक्त राष्ट् र फ् रे मवर्क कन्वें शन (UNFCCC) के पक्षकारों का
26 वां सम्मे लन (COP26) था।
 पक्षकारों का सम्मे लन (COP):
o पक्षकारों का सम्मे लन (COP) जलवायु परिवर्तन पर सं युक्त राष्ट् र फ् रे मवर्क कन्वें शन का
निर्णय ले ने वाला सर्वोच्च निकाय है । इसलिए कथन 1 सही है ।
o कन्वें शन के पक्षकार सभी राज्यों को COP में प्रतिनिधित्व प्राप्त होता है , जहां वे
कन्वें शन तथा COP द्वारा अपनाए गए किसी अन्य कानूनी उपकरण के कार्यान्वयन की
समीक्षा करते हैं । इसके अतिरिक्त वे कन्वें शन के प्रभावी कार्यान्वयन को बढ़ावा दे ने के
लिए आवश्यक निर्णय ले ते हैं जिनमें सं स्थागत और प्रशासनिक व्यवस्थाएं भी शामिल
होती हैं ।
o COP का एक महत्वपूर्ण कार्य पक्षकारों द्वारा प्रस्तु त राष्ट् रीय सं चार एवं उत्सर्जन सूची
की समीक्षा करना है । इस सूचना के आधार पर COP पक्षकारों द्वारा किए गए उपायों के
प्रभावों तथा कन्वें शन के अं तिम उद्दे श्य को प्राप्त करने में हुई प्रगति का आकलन
करता है ।
o जब तक पक्षकार अन्यथा निर्णय न लें तब तक COP की बै ठक प्रतिवर्ष होती है । जब तक
कोई पक्षकार सत्र की मे जबानी
o करने की पे शकश नहीं करता है , तब तक COP की बै ठक बॉन में होती है जहां इसका का
सचिवालय स्थित है । COP की पहली बै ठक वर्ष 1995 में जर्मनी के बर्लिन में हुई थी।
o COP की अध्यक्षता सं युक्त राष्ट् र द्वारा मान्यता प्राप्त पांच क्षे तर् ों के बीच चक् रीय क् रम
में स्थानांतरित होती रहती है । ये क्षे तर् हैं : अफ् रीका, एशिया, लै टिन अमे रिका एवं
कैरिबियन, मध्य एवं पूर्वी यूरोप तथा पश्चिमी यूरोप एवं अन्य। साथ ही, COP का स्थान
इन समूहों के बीच भी स्थानांतरित होता रहता है । इसलिए कथन 2 सही नहीं है ।

50. में स्थापित अक्षय ऊर्जा क्षमता के सं दर्भ में , निम्नलिखित कथनों पर विचार कीजिए:

1. स्थापित अक्षय ऊर्जा क्षमता में भारत विश्व स्तर पर चौथे स्थान पर है ।

2. स्थापित सौर ऊर्जा क्षमता में भारत शीर्ष स्थान पर है ।

3. सरकार की योजना वर्ष 2030 तक 1000 गीगावॉट अक्षय ऊर्जा क्षमता स्थापित करने की है ।

उपर्युक्त कथनों में से कौन-सा/से सही है /हैं ?

(a) केवल 1

(b) केवल 2 और 3

(c) 1, 2 और 3

(d) केवल 2

Q50.A
 विगत कुछ वर्षों में भारत की स्थापित अक्षय ऊर्जा उत्पादन क्षमता में आई है , जिसमें वर्ष
् हुई है । चूंकि सरकार इस क्षे तर् क के लिए
2016 और 2020 के बीच 17.33% की दर से वृ दधि
समर्थन बढ़ा रही है , इसलिए यह क्षे तर् क निवे शकों के लिए आकर्षक हो गया है । सरकार का
लक्ष्य वर्ष 2022 तक गीगावॉट नवीकरणीय ऊर्जा क्षमता (जिसमें 114 गीगावॉट सौर ऊर्जा
क्षमता और 67 गीगावॉट पवन ऊर्जा क्षमता मे ल है ) हासिल करना है । यह पे रिस समझौते के
तहत निर्धारित 175 गीगावॉट के लक्ष्य से अधिक है । सरकार की वर्ष 2030 तक 523 गीगावॉट
अक्षय ऊर्जा क्षमता स्थापित करने की कथन 3 सही नहीं है ।
 भारतीय अक्षय ऊर्जा क्षे तर् क विश्व का चौथा सबसे आकर्षक अक्षय ऊर्जा बाजार है । वर्ष 2020
की स्थिति के अनु सार, स्थापित क्षमता में भारत पवन ऊर्जा में चौथे , सौर ऊर्जा में पांचवें और
अक्षय ऊर्जा में चौथे स्थान पर था। इसलिए कथन 1 सही है और कथन 2 सही नहीं है ।

ू ण के सं दर्भ में , निम्नलिखित कथनों पर विचार कीजिए:


51. भारत में ओजोन प्रदष

1. धरातलीय ओजोन नाइट् रोजन के ऑक्साइडों (NOx) और वाष्पशील कार्बनिक यौगिकों (VOC) के
बीच रासायनिक अभिक्रिया से बनती है ।

2. ओजोन गे हं ू जै सी अनाज वाली फसलों की पै दावार को कम कर सकती है ।

ू ण को राष्ट् रीय वायु गु णवत्ता सूचकांक के माध्यम से मापा जाता है ।


3. ओजोन प्रदष

उपर्युक्त कथनों में से कौन-सा/से सही है /हैं ?

(a) केवल 1

(b) केवल 1 और 3

(c) केवल 2 और 3

(d) 1,2 और 3

Q 51.D

 ओजोन एक गै स है जो ऑक्सीजन के तीन परमाणु ओं से मिलकर बनी होती है । ओजोन पृ थ्वी के


ऊपरी वायु मंडल और धरातल दोनों स्तरों पर विद्यमान होती है । ओजोन अच्छी या खराब हो
सकती है , ले किन यह इस बात पर निर्भर करता है कि यह कहां पाई जाती है ।
 ू क है क्योंकि यह लोगों के स्वास्थ्य और पर्यावरण
धरातलीय ओजोन एक हानिकारक वायु प्रदष
पर हानिकारक प्रभाव डालती है , साथ ही यह "धूम कोहरे " का मु ख्य घटक है ।
 क्षोभमं डलीय या धरातलीय ओजोन सीधे वायु में उत्सर्जित नहीं होती है , बल्कि नाइट् रोजन के
ऑक्साइडों (NOx) और वाष्पशील कार्बनिक यौगिकों (VOC) के बीच रासायनिक अभिक्रिया
से बनती है ।
 यह तब होता है जब कारों, विद्यु त सं यंतर् ों, औद्योगिक बॉयलरों, रिफाइनरियों, रासायनिक
ू क सूर्य के प्रकाश की उपस्थिति में रासायनिक
सं यंतर् ों और अन्य स्रोतों से उत्सर्जित प्रदष
अभिक्रिया करते हैं ।
 भारतीय प्रौद्योगिकी सं स्थान-मद्रास (IIT-M) के ने तृत्व में एक बहु-सं स्थान अध्ययन से ज्ञात
हुआ है कि धरातलीय ओजोनप्रतिवर्ष भारत की गे हं ू की उपज का लगभग 22 मिलियन टन
(21%) और धान की उपज का लगभग 6.5 मिलियन टन (6%)नष्ट कर दे ती है । गे हं ू और धान
की उपज में क् रमशः 16% और 11% की हानि अकेले पं जाब और हरियाणा में हो जाती है ।
o किसी भी अन्य गै स की तरह धरातलीय ओजोन भी पादपों की पत्तियों में उनके रं धर् ों
(स्टोमे टा) के माध्यम से प्रवे श करती है । यह सामान्य वायु मंडलीय गै सीय-विनिमय के भाग
के रूप में होता है । पादपों द्वारा ग्रहण किए जाने पर यह पादपों में विद्यमान जल में घु ल जाती
है और आगे अन्य रसायनों के साथ अभिक्रिया करती है जिससे प्रकाश सं श्ले षण की क्रिया
प्रभावित होती है । इसके फलस्वरूप फसल की पै दावार भी प्रभावित होती है ।
 ू ण के प्रभाव
स्वास्थ्य पर ओजोन प्रदष
o खांसी और गले में खराश उत्पन्न होना
o इसके कारण गहरी और जोर से सांस ले ना कठिन हो जाता है तथा गहरी सांस ले ते समय दर्द
होता है ।
o श्वास मार्ग में सूजन और क्षति होना।
o फेफड़े सं क्रमण के प्रति अधिक सं वेदनशील हो जाते हैं ।
o फेफड़े के रोग जै से अस्थमा, वातस्फीति और क् रोनिक ब्रोंकाइटिस अधिक गं भीर हो जाते हैं ।
o अस्थमा के दौरे अधिक बार आने लगते हैं ।
o पर्यावरण मं तर् ालय द्वारा अप्रैल 2015 में राष्ट् रीय वायु गु णवत्ता सूचकांक (AQI) का
शु भारम्भ किया गया था।
ू त,
o वायु गु णवत्ता सूचकांक (AQI) की छह श्रेणियां हैं : अच्छी, सं तोषजनक, मध्यम प्रदषि
खराब, बहुत खराब और गं भीर।
 ू कों पर विचार किया जाता है (PM10, PM2.5, NO2, CO2, O3, NH3 और
AQI में आठ प्रदष
Pb)। इसके लिए अल्पकालिक (24 घं टे की औसत अवधि तक) राष्ट् रीय परिवे शी गु णवत्ता
मानक निर्धारित किए गए हैं ।
 इसलिए विकल्प (d) सही उत्तर है ।
52. निम्नलिखित में से कौन-सा/से उपकरण कार्बन क् रे डिट बाजारों से सं बंधित है /हैं ?

1. प्रमाणित उत्सर्जन कटौती (CER)

2. उत्सर्जन न्यूनीकरण इकाई (ERU)

3. सत्यापित उत्सर्जन कटौती (VER)

नीचे दिए गए कू ट का प्रयोग कर सही उत्तर चु निए।

(a) केवल 1 और 2

(b) केवल 2 और 3

(c) केवल 3

(d) 1, 2 और 3

Q52.D

कार्बन बाजार दो प्रकार के हैं :

 नियामक अनु पालन बाजार (Regulatory Compilance markets)

o अनु पालन बाजार का उपयोग कंपनियों तथा सर द्वारा किया जाता है जहां उन्हें कानूनी तौर पर
अपने ग्रीनहाउस गै स (GHG) उत्सर्जनों का हिसाब दे ना होता है । अनिवार्य राष्ट् रीय, क्षे तर् ीय या
अं तर्राष्ट् रीय कार्बन कटौती व्यवस्थाओं द्वारा नियं त्रित किया जाता है ।
 स्वै च्छिक बाजार (Voluntary markets)
o स्वै च्छिक बाजार में कार्बन क् रे डिट का व्यापार स्वै च्छिक आधार पर होता है ।
o स्वै च्छिक कार्बन क् रे डिट (VER) मु ख्य रूप से निजी क्षे तर् क द्वारा खरीदे जाते हैं । कार्बन
क् रे डिट खरीदने के लिए कॉर्पोरेट सामाजिक उत्तरदायित्व (CSR) और जनसं पर्क सबसे
सामान्य प्रेरणास्रोत हैं । प्रमाणीकरण, प्रतिष्ठा और पर्यावरणीय एवं सामाजिक लाभ
जै से विचार अन्य कारण हैं ।
o स्वै च्छिक बाजारों में कार्बन क् रे डिट को सत्यापित उत्सर्जन कटौती (Verified Emission
Reduction: VER) कहा जाता है । इसलिए विकल्प 3 सही है ।
 क्योटो प्रोटोकॉल के तहत, विनियमित कार्बन व्यापार बाजारों के दो प्रमु ख तं तर् हैं :
o स्वच्छ विकास तं तर् (Clean Development Mechanism: CDM)
 क्योटो प्रोटोकॉल के तहत विकासशील दे श (गै र अनु लग्नक | के दे श) अपने GHG
उत्सर्जनों को कम करने के लिए बाध्य नहीं हैं , जबकि औद्योगिक दे शों (अनु लग्नक | के
दे श) के लिए निर्दिष्ट लक्ष्यों को पूरा करना अनिवार्य है ।
 ये दे श इन लक्ष्यों को प्राप्त करने के लिए अपने राज्यक्षे तर् में GHG उत्सर्जनों को कम
कर सकते हैं , अन्य दे शों में उत्सर्जनों को कम करने के लिए परियोजनाओं को कार्यान्वित
कर सकते हैं या कार्बन व्यापार कर सकते हैं ।
 कोई औद्योगिक दे श किसी विकासशील दे श में उत्सर्जन कटौती परियोजना का
क्रियान्वयन कर सकता है । यह वनीकरण, ऊर्जा दक्षता या अक्षय ऊर्जा परियोजना हो
सकती है । GHGs के ग्रहण या बचत के कारण कार्बन क् रे डिट उत्पन्न होते हैं ।
 CDM परियोजनाओं से प्राप्त कार्बन क् रे डिट को प्रमाणित उत्सर्जन कटौती (CER) कहा
जाता है । इसलिए विकल्प 1 सही है ।
o सं युक्त कार्यान्वयन (Joint Implementation: JI)
 घरे लू स्तर पर उत्सर्जनों को कम करने के एक विकल्प के रूप में , अनु लग्नक | का कोई
भी दे श अनु लग्नक | के ही किसी अन्य दे श में GHG उत्सर्जनों को कम करने के लिए
किसी परियोजना में निवे श कर सकता है (इसे "सं युक्त कार्यान्वयन परियोजना" के रूप
में सं दर्भित किया जाता है )।
 इस तरीके से दे श ऐसी परियोजनाओं में निवे श करके, जहां उत्सर्जनों को कम करना
सस्ता हो सकता है , अपने क्योटो लक्ष्यों के अनु पालन की लागत को कम कर सकते हैं
और परिणामी उत्सर्जन न्यूनीकरण इकाइयों (Emission Reduction Units: ERU) को
अपने प्रतिबद्धता लक्ष्य की प्राप्ति के लिए शामिल कर सकते हैं । इसलिए विकल्प
2 सही है ।
 उत्सर्जन न्यूनीकरण इकाई (ERU) एक उत्सर्जन इकाई है जिसे सं युक्त कार्यान्वयन
परियोजना के तहत जारी किया जाता है ।

53. वै श्विक तापन से बचने के लिए भू-अभियां त्रिकी तकनीकों के एक भाग के रूप में निम्नलिखित
विधियों में से कौन-सी सं भावित रूप से अपनाई जा सकती हैं ?

1. अं तरिक्ष में दर्पण स्थापित करना

2. बारिश कराने के लिए बादलों में सिल्वर आयोडाइड की बौछार करना

3. समु दर् में लौह चूर्ण का छिड़काव करना

4. आकाश में सल्फर कणों का अं तःक्षे पण करना


नीचे दिए गए कू ट का प्रयोग कर सही उत्तर चु निए।

(a) केवल 2, 3 और 4

(b) केवल 1 और 3

(c) 1,2,3 और 4

(d) केवल 2 और 4

Q 53.C

 भू-अभियां त्रिकी (Geo-Engineering) वै श्विक जलवायु परिवर्तन का मु काबला करने के लिए


जलवायु प्रणाली में जानबूझकर किया गया हे रफेर है । वै ज्ञानिकों ने अने क भू-अभियां त्रिकी
योजनाओं का अनु मान लगाया है जिन्हें दो रणनीतिक समूहों में रखा गया है । एक समूह का
लक्ष्य ग्रीन हाउस प्रभाव को कम करने के लिए वातावरण से CO2 को हटाने के लिए बड़े
पै माने पर प्रयास करना है । इस प्रकार की रणनीति का एक उदाहरण "कृत्रिम वृ क्ष" नामक
मशीनें हैं जो अपने चारों ओर की वायु से कार्बन से 32 को अवशोषित कर ले ते हैं और उसे
ू रे समूह को सौर विकिरण
परित्यक्त ते ल या गै स क्षे तर् ों में मु क्त कर दे ते हैं । रणनीतियों के दस
प्रबं धन (Solar Radiation Management: SRM) कहा गया है । इस सामान्य दृष्टिकोण का
उद्दे श्य आपतित सौर विकिरण को या तो ऊपरी वायु मंडल में या पृ थ्वी की सतह पर अथवा
उसके निकट परावर्तित करके पृ थ्वी को ठं डा करना है । वै श्विक तापन से बचने और जलवायु
परिवर्तन का शमन करने करने के लिए भू-अभियां त्रिकी के कुछ प्रस्तावित उपाय इस प्रकार
हैं :
 सूर्य की किरणों को परावर्तित करने के लिए ऊपरी वायु मंडल में सल्फेट कणों का विस्फोट
करना;
 CO2 को अवशोषित करने वाले प्लवक को पोषित करने के लिए महासागरों में लौह चूर्ण का
छिड़काव करना;
 बारिश कराने के लिए बादलों में सिल्वर आयोडाइड की बौछार करना;
 पृ थ्वी की जलवायु को पूर्व-औद्योगिक स्तर पर वापस लाने के लिए पर्याप्त सूर्य के को विक्षे पित
करने हे तु अं तरिक्ष में दर्पण स्थापित करना;
 परावर्तक पत्तियों के लिए आनु वंशिक रूप से इं जीनियर्ड फसलें उगाना;
 बादलों को सफेद बनाने के लिए बादलों में समु दर् ी जल का
 बड़ी मात्रा में पौधों को समु दर् में डालकर या मिट् टी में दफनाकर चारकोल में बदलना;
 इसलिए विकल्प (C) सही उत्तर है ।
54. रॉटरडै म अभिसमय के सं दर्भ में , निम्नलिखित कथनों पर विचार कीजिये :

1. यह एक कानूनी रूप से बाध्यकारी बहुपक्षीय सं धि है जिसका उद्दे श्य खतरनाक रसायनों के आयात के
सं बंध में साझा उत्तरदायित्व को बढ़ावा दे ना है ।

2. भारत में , रसायन और पे ट्रो रसायन विभाग कीटनाशकों के लिए प्राधिकृत राष्ट् रीय प्राधिकरण
है ।

उपर्युक्त कथनों में से कौन-सा/से सही है /हैं ?

(a) केवल 1

(b) केवल 2

(c) 1 और 2 दोनों

(d) न तो 1, न ही 2

Q 54.A

 रॉटरडै म अभिसमय जिसे औपचारिक रूप से “अं तर्राष्ट् रीय व्यापार में कुछ खतरनाक रसायनों
और कीटनाशकों के लिए पूर्व सूचित सहमति प्रक्रिया पर रॉटरडै म अभिसमय" कहा जाता है ,
एक कानूनी रूप से बाध्यकारी बहुपक्षीय सं धि है । यह अभिसमय का उद्दे श्य मानव स्वास्थ्य
और पर्यावरण को सं भावित नु कसान से बचाने के लिए कुछ खतरनाक रसायनों के अं तर्राष्ट् रीय
व्यापार में पक्षकार दे शों के बीच साझा उत्तरदायित्व और सहयोगात्मक प्रयासों को बढ़ावा
दे ना है । इसलिए कथन 1 सही है ।
 अभिसमय सूचना के खु ले आदान-प्रदान को बढ़ावा दे ता है और खतरनाक रसायनों के
निर्यातकों को, सु रक्षित प्रबं धन के लिए निर्दे शों सहित, उचित ले बलिं ग का उपयोग करने तथा
किसी भी ज्ञात रोक या प्रतिबं ध के बारे क् रे ता को सूचित करने के लिए आह्वान करता है ।
 अभिसमय में कीटनाशकों और औद्योगिक रसायनों दोनों को शामिल किया गया है जिन्हें
पक्षकारों द्वारा स्वास्थ्य या पर्यावरणीय कारणों से निषिद्ध या गं भीर रूप से प्रतिबं धित और
पूर्व-सूचित सहमति (PIC) प्रक्रिया में शामिल करने के लिए अधिसूचित किया गया है ।
 अभिसमय के अं तर्गत आवश्यक प्रशासनिक कार्यों को करने के लिए प्रत्ये क पक्षकार द्वारा
एक राष्ट् रीय प्राधिकरण को प्राधिकृत करना अनिवार्य है । भारत में , औद्योगिक रसायनों के
लिए रसायन और पे ट्रो रसायन विभाग तथा कीटनाशकों के लिए कृषि एवं सहकारिता
विभाग प्राधिकृत राष्ट् रीय प्राधिकरण (Designated National Authority: DNA) हैं ।
इसलिए कथन 2 सही नहीं है ।
55. निम्नलिखित यु ग्मों पर विचार कीजिए:

कन्वें शन/समझौता सं बंधित है

1.बे सल कन्वें शन : खतरनाक अपशिष्ट के सीमापार सं चलन पर नियं तर् ण से

ू कों से
2. स्टॉकहोम समझौता : स्थायी कार्बनिक प्रदष

3. मिनामाता समझौता : पारे के मानवजनित निर्मुक्ति से

उपर्युक्त यु ग्मों में से कौन-सा/से सही सु मेलित है /हैं ?

(a) केवल 1 और 2

(b) 1, 2 और 3

(c) केवल 3

(d) केवल 2 और 3

Q55.B

 पारे पर मिनामाता समझौता मानव स्वास्थ्य और पर्यावरण को पारे के प्रतिकू ल प्रभावों से


सु रक्षा प्रदान करने हे तु एक वै श्विक सं धि है । मिनामाता समझौता एक ऐसी वै श्विक और
सर्वव्यापी धातु की ओर ध्यान आकर्षित करता है जो प्राकृतिक रूप से पाई जाती है और दै निक
उपयोग की वस्तु ओं में इसका व्यापक प्रयोग होता है । साथ ही इस धातु को विभिन्न स्रोतों
से पर्यावरण, मृ दा और जल में निर्मुक्त किया जाता है । पारे के सं पर्ण
ू जीवन चक् र में इसके
मानवजनित विमोचन को नियं त्रित करना इस समझौते के तहत प्रदत्त दायित्वों के निर्धारण
में एक महत्वपूर्ण कारक रहा है । इसलिए यु ग्म 3 सही सु मेलित है ।
 ू पर स्टॉकहोम समझौते को स्टॉकहोम, स्वीडन में 22 मई 2001 को पूर्ण
स्थायी कार्बनिक प्रदष
अधिकार प्राप्त (Plenipotentiaries) सभा द्वारा अपनाया गया था। यह समझौता 17 मई
ू कों पर स्टॉकहोम समझौता मानव स्वास्थ्य
2004 को लागू हुआ था। स्थायी कार्बनिक प्रदष
और पर्यावरण को दीर्घ अवधि तक पर्यावरण में बने रहने वाले रसायनों से सु रक्षा प्रदान करने
हे तु एक वै श्विक सं धि है । ये रसायन भौगोलिक रूप में व्यापक स्तर पर वितरित हो जाते है
तथा मनु ष्यों और वन्यजीवों के वसायु क्त ऊतक में सं चित होकर मानव स्वास्थ्य या पर्यावरण
पर हानिकारक प्रभाव डालते है । इसलिए यु ग्म 2 सही सु मेलित है ।
 22 मार्च 1989 को बे सल, स्विटजरलैं ड में पूर्ण अधिकार प्राप्त (Plenipotentiaries) सभा द्वारा
खतरनाक अपशिष्टों के सीमा पार सं चलन पर नियं तर् ण और उनके निपटान हे तु बे सल कन्वें शन
को अपनाया गया था। इसे 1980 के दशक में , अफ् रीका और विकासशील विश्व के अन्य
हिस्सों में विदे शों से आयातित विषाक्त अपशिष्ट के भं डार की पहचान के बाद सार्वजनिक
आक् रोश की अनु क्रिया में अपनाया गया था। बे सल कन्वें शन का व्यापक उद्दे श्य खतरनाक
अपशिष्ट के प्रतिकू ल प्रभावों के विरुद्ध मानव स्वास्थ्य और पर्यावरण की रक्षा करना है । यह
कन्वें शन के दायरे में अपशिष्ट की एक विस्तृ त शृं खला शामिल है , जिसे उनकी उत्पत्ति/सं रचना
और उनकी विशे षताओं के आधार पर "खतरनाक अपशिष्ट" के रूप में परिभाषित किया गया
है । साथ ही इसके तहत दो प्रकार के अपशिष्ट - घरे लू अपशिष्ट और भट् टी की राख को
"अन्य अपशिष्ट" के रूप में परिभाषित किया गया है । इसलिए यु ग्म 1 सही सु मेलित है ।

56. तटीय विनियमन क्षे तर् (CRZs) के सं दर्भ में , निम्नलिखित कथनों पर विचार कीजिए:

1. CRZs पर्यावरण, वन और जलवायु परिवर्तन मं तर् ालय द्वारा घोषित किए जाते हैं ।

2. CRZs और ती क्षे तर् का भू-भाग शामिल होता है । में से कौन-सा/से सही है /हैं ?

(a) केवल 1

(b) केवल 2

(c) 1 और 2 दोनों

(d) न तो 1, न ही 2

Q56.A

 हाल ही में , तटीय विनियमन क्षे तर् (CRZ) अधिसूचना, 2019 में प्रस्तावित सं शोधनों हे तु एक
मसौदा अधिसूचना जारी की गई है ।
 प्रस्तावित प्रमु ख सं शोधन:
o ते ल और प्राकृतिक गै स के विकास एवं उत्पादन को पूर्व CRZ मं जरू ी से छट
ू प्रदान की
जाएगी। पूर्णतः अस्थायी और मौसमी सं रचनाओं (shacks) को पर्याप्त सावधानियों
मानसून के मौसम के दौरान बनाए रखा जाएगा।
o जब तक सं बंधित राज्यों/केंद्र शासित प्रदे शों द्वारा एकीकृत द्वीप योजना (IIMP) तै यार
नहीं की जाती, तब तक CRZ अधिसूचना 2011 लागू रहे गी।
o अं तर्वारीय क्षे तर् ों में बालू भित्तियों को पारं परिक तटीय द्वारा केवल मै नुअल तरीके से
हटाया जाएगा।
 ये सं शोधन अभी प्रस्तावित हैं जिनकी अभी तक गोषणा नहीं की गई है । और वन मं तर् ालय
(MoEF) द्वारा तटीय क्षे तर् में गतिविधियों के नियमन के लिए फरवरी, 1991 में एक
अधिसूचना जारी गई थी। इसलिए कथन 1 सही है ।
 तटीय विनियमन क्षे तर् :
o CRZ-I, CRZ-IA के रूप में सीमां कित पारिस्थितिकी सं वेदनशील क्षे तर् ों में मैं ग्रोव वॉक, ट् री
हट् स, ने चर ट् रेल्स जै सी ईको- पर्यटन गतिविधियों को अनु मति दे ता है । CRZ-IA में सी लिं क,
नमक सं चयन और विलवणीकरण सं यंतर् एवं स्टिल्ट पर सड़क निर्माण की भी अनु मति दी गई
है । अं तर्ध्वारीय क्षे तर् या CRZ-IB क्षे तर् ों में बं दरगाहों और सी लिं क के लिए विवादास्पद भूमि
पु नर्ग्रहण की अनु मति प्रदान की गई है । विवादास्पद भूमि में महासागरों या झील सं स्तरों से
सृ जित नवीन भूमि और तटीय पारिस्थितिकी पर गहन प्रभाव यु क्त भूमि शामिल है ।
o CRZ-II में पर्याप्त रूप से निर्मित क्षे तर् (built-up area) शामिल है । इन क्षे तर् ों में परियोजना
डे वलपर्स अब फ्लोर एरिया रे शियो या फ्लोर स्पे स इं डेक्स बढ़ा सकते हैं तथा रिसॉर्ट्स और
अन्य पर्यटन सु विधाओं का निर्माण कर सकते हैं ।
o CRZ III क्षे तर् ों में ऐसे भूमि क्षे तर् शामिल हैं जो अपे क्षाकृत अहस्तक्षे पित हैं और जो CRZ II
के अं तर्गत शामिल नहीं हैं । इन्हे जनसं ख्या घनत्व के आधार पर CRZ IIIA और IIIB में
वर्गीकृत किया गया है ।
o CRZ-IV, में 12 समु दर् ी मील तक विस्तारित तटीय जल की उथली पट् टी शामिल है ।
इसलिए कथन 2 सही नहीं है ।

57. निम्नलिखित में से कौन-सा कार्बन इं टेंसिटी इं डिकेटर का सही वर्णन करता है ?

(a) विद्यु त सं यंतर् द्वारा उत्पादित ऊर्जा की प्रति इकाई कार्बन उत्सर्जन की मात्रा।

(b) एक स्वस्थ मनु ष्य द्वारा अपने जीवनकाल में उत्पन्न कार्बन उत्सर्जन की मात्रा।

(c) परिवहन कार्य की एक इकाई को करने में जहाज द्वारा उत्पन्न कार्बन उत्सर्जन की मात्रा।

(d) हिमनदों की परतों के ते जी से पिघलने के कारण कार्बन पृ थक्करण की सं भावित हानि।

Q 57.C

 कार्बन इं टेंसिटी इं डिकेटर (Carbon Intensity Indicator : CII) इस बात का मापक है कि एक


जहाज कितनी कुशलता से माल या यात्रियों का परिवहन करता है । इसे परिवहन कार्य की एक
इकाई को करने में जहाज द्वारा उत्पन्न कार्बन उत्सर्जन की मात्रा के रूप में वर्णित किया जाता
है । यह प्रति कार्गो-वहन क्षमता एवं समु दर् ी मील के दौरान उत्सर्जित CO2 के ग्राम में मापा
जाता है । इसलिए विकल्प (c) सही उत्तर है ।
 इसके बाद जहाज को A से E तक की वार्षिक रे टिंग प्रदान की जाती है । वर्ष 2030 तक रे टिंग
की सीमा अधिक कठोर होती जाएगी।
 CII 5000 GT (गीगा टन) से अधिक की क्षमता वाले सभी कार्गो और क् रूज जहाजों पर लागू
होता है ।
 CII प्रत्यक्ष रूप से ईंधन की खपत पर आधारित है , जो इस बात से प्रभावित होता है कि एक
विशिष्ट जहाज अपनी तकनीकी दक्षता और ईंधन के सं योजन से कैसे सं चालित होता है ।
 इसका मूल्य उपयोग किए गए ईंधन के प्रकार, पोत की दक्षता और पोत की गति, कार्गो
परिवहन, मौसम की स्थिति तथा पोत की सामान्य स्थिति (जै से जै व ईंधन) जै से परिचालन
मानकों से प्रभावित होता है ।
 वार्षिक CII की गणना अं तर्राष्ट् रीय समु दर् ी सं गठन (IMO) द्वारा रिपोर्ट किए गए आं कड़ों के
अधार पर की जाती है ।

58. मे थनॉल के सं दर्भ में , निम्नलिखित कथनों पर विचारकीजिए:

1. इसमें पे ट्रोल और डीजल की तु लना में बहुत अधिक ऊर्जा दक्षता होती है ।

2. यह सभी आं तरिक दहन इं जनों में कुशलता से दहन होता है ।

3. इसका दहन होने पर पार्टिकुले ट मै टर उत्पन्न नहीं होते है ।

उपर्युक्त कथनों में से कौन-सा/से सही है /हैं ?

(a) केवल 1

(b) केवल 1 और 3

(c) केवल 2 और 3

(d) 1,2 और 3

Q 58.C

 मे थनॉल उच्च राख वाले कोयले , कृषि अवशे षों, ताप विद्यु त सं यंतर् ों से प्राप्त कार्बन डाइ
ऑक्साइड और प्राकृतिक गै स से बनने वाला "अल्प कार्बनयु क्त और हाइड्रोजन वाहक
ईंधन" है । यह COP 21 के लिए भारत की प्रतिबद्धता को पूरा करने का सबसे अच्छा मार्ग है ।
नीति आयोग ने वर्ष 2030 तक कच्चे ते ल के आयात के 10% को अकेले मे थनॉल के साथ
बदलने के लिए एक रोडमै प तै यार किया है ।
 कथन 1 सही नहीं है : मे थनॉल(22.7 MJ/Kg) की ऊर्जा दक्षता पे ट्रोल (44-46 MI/Kg) और
डीजल (42-46 MI/Kg) की तु लना में कम होती है । हालां कि यह परिवहन क्षे तर् (सड़क, रे ल
और समु दर् ी), ऊर्जा क्षे तर् (ट् रैक्टर और वाणिज्यिक वाहनों सहित) और रिटे ल कुकिंग (मिट् टी
के ते ल और लकड़ी के चारकोल के स्थान पर) में इन दोनों ईंधनों को प्रतिस्थापित कर सकता
है ।
 कथन 2 सही है : मे थनॉल सभी आं तरिक दहन इं जनों में कुशलता से दहन होता है । मे थनॉल के
गै सीय सं स्करण – DME (डाइमिथाइल ईथर) को LPG के साथ मिश्रित किया जा सकता है
और यह बड़ी बसों एवं ट् रकों में डीजल का एक उत्कृष्ट विकल्प हो सकता है ।
 कथन 3 सही है : मे थनॉल न तो पार्टिकुले ट मै टर और न ही कालिख उत्पन्न नहीं करता है ।
साथ ही यह लगभग शून्य नाइट् रोजन ऑक्साइड (NOx) और सल्फर ऑक्साइड (SOx)
ू ण) करता है । इसका उत्पादन प्राकृतिक गै स, भारतीय उच्च
उत्सर्जन (लगभग शून्य प्रदष
राख वाले कोयले , बायोमास, नगरपालिका ठोस अपशिष्ट (MSW), स्टैं डे ड और फ्ले यर्ड गै सों
आदि से किया जा सकता है ।

59. राष्ट् रीय जै व ईंधन नीति के अनु सार, इथे नॉल उत्पादन के लिए निम्नलिखित में से कौन-से कच्चे
माल का उपयोग किया जा सकता है ?

1. मीठे चु कंदर

2. मीठा ज्वार

3. कसावा

4. जौ

दिए गए कू ट का प्रयोग कर सही उत्तर चु निए।

(a) केवल 1, 2, 3 और 5

(b) केवल 2, 4 और 6

(c) केवल 1, 3,4 और 6

(d) 1, 2, 3, 4,5 और 6
Q 59.A

 राष्ट् रीय जै व ईंधन नीति 2018, जै व ईंधन को "आधारभूत जै व ईंधनों" अर्थात पहली पीढ़ी
ू री पीढ़ी (2G) के इथे नॉल,
(1G) के बायोएथे नॉल, बायोडीजल और "उन्नत जै व ईंधन", दस
नगर निगमों के ठोस अपशिष्ट (MSW) से उत्पादित ड्रॉप-इन ईंधन, तीसरी पीढ़ी (3G) के
जै व ईंधन, जै व-सीएनजी आदि के रूप में वर्गीकृत करती है । इस वर्गीकरण का उद्दे श्य प्रत्ये क
श्रेणी के लिए उपयु क्त वित्तीय और आर्थिक प्रोत्साहन को बढ़ावा दे ना है ।
 यह नीति गन्ने के रस, चीनी यु क्त सामग्री जै से चु कंदर, मीठा ज्वार, स्टार्च यु क्त सामग्री
जै से मक्का, कसावा, गे हं ,ू टू टे चावल, पके आलू, मानव उपभोग के लिए अनु पयु क्त अनाज के
उपयोग की अनु मति दे कर इथे नॉल उत्पादन के लिए कच्चे माल के दायरे का विस्तार करती है ।
 उन्नत जै व ईंधन पर बल दे ने के साथ, नीति में 2G इथे नॉल जै व रिफाइनरी के लिए 1G जै व
ईंधन की तु लना में अतिरिक्त कर प्रोत्साहनों, उच्च खरीद मूल्य के अलावा 6 वर्षों में 5000
करोड़ रूपए व्यवहार्यता अं तराल अनु दान (Viability gap funding) का सं केत दिया गया है ।
 यह नीति गै र-खाद्य वनस्पति ते लों, प्रयु क्त किए जा चु के खाना पकाने के ते ल, कम समय में
तै यार होने वाली फसलों से बायोडीजल उत्पादन के लिए आपूर्ति श्रखृं ला तं तर् की स्थापना को
प्रोत्साहित करती है ।

60. निम्नलिखित में से कौन-से ग्रीन पटाखे के घटक हैं ?

1. लिथियम

2. बे रियम

3. एल्यु मिनियम

4. थर्माइट

नीचे दिए गए कू ट का प्रयोग कर सही उत्तर चु निए।

(a) केवल 1 और 2

(b) केवल 1, 2 और 3

(c) केवल 3 और 4

(d) केवल 1, 3 और 4

Q60.C
 ग्रीन पटाखे पर्यावरण के अनु कूल पटाखों को सं दर्भित करते हैं एवं ये पारं परिक पटाखों से होने
ू ण को भी कम कर सकते हैं । इन्हें राष्ट् रीय पर्यावरण इं जीनियरिं ग अनु संधान
वाले वायु प्रदष
सं स्थान (NEERI) तथा वै ज्ञानिक और औद्योगिक अनु संधान परिषद (CSIR) प्रयोगशाला
द्वारा विकसित किया गया है ।
 ग्रीन पटाखों में लिथियम, आर्सेनिक, बे रियम और ले ड जै से प्रतिबं धित रसायन नहीं होते हैं ।
उन्हें से फ वाटर रिलीजर (SWAS), से फ थर्माइट क् रै कर (STAR) और से फ मिनिमल
एल्यु मिनियम (SAFAL) क् रै कर्स कहा जाता है । इसलिए विकल्प (c) सही उत्तर है ।
 इन ग्रीन पटाखों को तीन अलग-अलग नाम दिए गए हैं :
 SWAS - से फ वाटर रिलीजर- ये वायु में जलवाष्प छोड़ते हैं जो निकलने में एवं धु एं को दबा
दे ता है ।
o इसमें पोटे शियम नाइट् रेट और सल्फर प्रयोग नहीं किया जाता है ।
o गै सीय उत्सर्जन के लिए एक मं दक का प्रयोग किया जाता है ।
o इससे निकलने वाले धूलकणों में लगभग 30 प्रतिशत की कमी
 STAR - से फ थर्माइट क् रै कर- इसमें पोटे शियम नाइट् रेट और सल्फर का प्रयोग नहीं होता
है ।
o इससे कणिकीय पदार्थ का कम उत्सर्जन- कम ध्वनि तीव्रता होती है ।
 SAFAL - से फ मिनिमल एल्यु मीनियम
o एल्यु मिनियम का न्यूनतम उपयोग किया जाता है ।
o एल्यु मिनियम के स्थान पर मै ग्नीशियम का उपयोग होता है ।
ू ण में कमी होगी।
o पारं परिक पटाखों की तु लना में ध्वनि प्रदष

61. पराली जलाने के सं दर्भ में , निम्नलिखित कथनों पर विचार कीजिए:

1. फसल अवशे ष जलाने के दौरान सल्फर के ऑक्साइड निर्मुक्त होते हैं ।

2. धान की पराली जलाने से निकलने वाली ऊष्मा उपजाऊ मृ दा के महत्वपूर्ण जीवाणु और कवक
आबादी को नष्ट कर दे ती है ।

ू ण नियं तर् ण अधिनियम 1981 के तहत फसल अवशे ष


3. भारतीय दं ड सं हिता (IPC) और वायु प्रदष
जलाना एक अपराध है ।

उपर्युक्त कथनों में से कौन-से सही हैं ?

(a) केवल 1 और 2
(b) केवल 2 और 3

(c) केवल 1 और 3

(d) 1, 2 और 3

Q61.D

 एक अध्ययन के माध्यम से यह आकलन किया गया है कि फसल अवशे षों के दहन से 149.24
मिलियन टन कार्बन डाइऑक्साइड (CO2), 9 मिलियन टन से अधिक कार्बन मोनोऑक्साइड
(CO), 0.25 मिलियन टन सल्फर ऑक्साइड (SOX), 1.28 मिलियन टन पार्टिकुले ट मै टर और
0.07 मिलियन टन ब्लै क कार्बन उत्सर्जित होते हैं । इसलिए कथन 1 सही है ।
 ू त करते हैं और दिल्ली में कोहरे की समस्या के साथ-साथ
ये प्रत्यक्ष तौर पर पर्यावरण प्रदषि
हिमालय के ग्ले शियरों के पिघलने के लिए भी उत्तरदायी हैं ।
 पराली को जलाने से निकलने वाली ऊष्मा मृ दा के भीतर 1 सें टीमीटर तक प्रवे श करती है ,
जिससे उसका तापमान 33.8 से 42.2 डिग्री से ल्सियस तक बढ़ जाता है । यह उपजाऊ मृ दा के
लिए महत्वपूर्ण जीवाणु और कवक आबादी को नष्ट कर दे ती है । इसलिए कथन 2 सही है ।
 फसल अवशे षों को जलाने से मृ दा की ऊपरी परत में मौजूद अन्य सूक्ष्म जीवों के साथ-साथ
इसकी जै विक गु णवत्ता को भी क्षति पहुंचती है । 'मित्र' सूक्ष्मजीवों के नष्ट होने से 'शत्रु'
कीटों का प्रकोप बढ़ जाता है जिसके फलस्वरूप फसलों में रोग की सं भावना अधिक होती है ।
मृ दा की ऊपरी परतों की विले यता की क्षमता भी कम हो जाती है ।
 एक रिपोर्ट के अनु सार, एक टन पराली जलाने से आर्गेनिक कार्बन के अतिरिक्त मृ दा के सभी
पोषक तत्व जै से 5.5 किलोग्राम नाइट् रोजन, 2.3 किलोग्राम फास्फोरस, 25 किलोग्राम
पोटे शियम और 1 किलोग्राम से अधिक सल्फर नष्ट हो जाते हैं ।
 ू ण
फसल अवशे षों का दहन भारतीय दं ड सं हिता (IPC) की धारा 188 और वायु तथा प्रदष
नियं तर् ण अधिनियम 1981 के अं तर्गत एक अपराध है । हालां कि, सरकार द्वारा सही ढं ग से
इसका कार्यान्वयन नहीं किया गया है । इसलिए कथन 3 सही है ।

62. किफायती परिवहन के लिए टिकाऊ विकल्प (SATAT) पहल सं बंधित है :

(a) ईंधन से ल से

(b) सं पीडित बायोगै स से

(c) बायोडीजल से
(d) हाइब्रिड और इले क्ट्रिक वाहनों से

Q62.B

 केंद्रीय पे ट्रोलियम और प्राकृतिक गै स तथा कौशल विकास एवं उद्यमिता मं तर् ी ने सं भावित
उद्यमियों की ओर से सार्वजनिक क्षे तर् क की ते ल विपणन कंपनियों (OMC) के साथ मिलकर
किफायती परिवहन के लिए टिकाऊ विकल्प(SATAT) नामक एक अभिनव पहल का शु भारं भ
किया है । इसका उद्दे श्य सं पीडित बायोगै स (CBG) उत्पादन सं यंतर् स्थापित करना और बाजार
में ऑटोमोटिव (मोटर वाहन) ईंधन के रूप में उपयोग के लिए CBG उपलब्ध कराना है । यह
बायोमास और जै विक कचरे के कुशल प्रबं धन के लिए एक वै कल्पिक, हरित परिवहन ईंधन है ।
 बायोगै स प्राकृतिक रूप से कृषि अवशे षों, मवे शियों के गोबर, गन्ना का कम्पोस्ट (sugarcane
press mud), नगरपालिका ठोस अपशिष्ट, सीवे ज उपचार सं यंतर् अपशिष्ट आदि जै से
अपशिष्ट स्रोतों से अवायवीय अपघटन की प्रक्रिया के माध्यम से उत्पन्न होती है ।
् करण के बाद इसे सं पीड़ित किया जाता है और CBG के रूप में वर्णित किया जाता है ।
शु दधि
CBG में शु द्ध मीथे न की मात्रा 95% से अधिक होती है । सं पीडित बायोगै स अपनी सं रचना
और ऊर्जा क्षमता में व्यावसायिक रूप से उपलब्ध प्राकृतिक गै स के समान है । ऊष्मीय मान (~
52,000 KJ/kg) और CNG के समान अन्य गु णों के साथ, सं पीड़ित बायोगै स को एक
वै कल्पिक, नवीकरणीय ऑटोमोटिव ईंधन के रूप में उपयोग किया जा सकता है ।

63. वन्यजीवों और वनस्पतियों की सं कटापन्न प्रजातियों के अं तर्राष्ट् रीय व्यापार पर समझौते


(CITES) के सं दर्भ में , निम्नलिखित कथनों पर विचार कीजिए:

1. यह एकमात्र वै श्विक सं धि है जो उन पौधों और जानवरों के व्यापार को प्रबं धित करती है जिनका


अस्तित्व उनके वन्य पर्यावासों में सं कटग्रस्त नहीं है ।

2. CITES कानूनी रूप से पक्षकारों के लिए बाध्यकारी है और यह राष्ट् रीय कानूनों का स्थान ग्रहण
करती है ।

3. CITES को सं युक्त राष्ट् र पर्यावरण कार्यक् रम (UNEP) के माध्यम से प्रशासित किया जाता है ।

उपर्युक्त कथनों में से कौन-सा/से सही है /हैं ?

(a) केवल 1

(b) केवल 2 और 3

(c) केवल 1 और 3
(d) 1, 2 और 3

Q63.C

 वन्यजीवों और वनस्पतियों की सं कटापन्न प्रजातियों के अं तर्राष्ट् रीय व्यापार पर (CITES),


सरकारों के मध्य सं पन्न और वर्ष 1975 में लागू एक अं तर्राष्ट् रीय समझौता है । यह उन पादप
और जं तु के अं तर्राष्ट् रीय व्यापार को प्रबं धित करने वाली एकमात्र सं धि बन गई है जिनका
अस्तित्व उनके वन्य पर्यावासों सं कटग्रस्त नहीं है । इसलिए कथन 1 सही है ।
 CITES एक अं तर्राष्ट् रीय समझौता है जिसका दे श और क्षे तर् ीय एकीकरण सं गठन स्वे च्छा से
पालन करते हैं । जिन दे शों ने इस समझौते में शामिल होने की पु ष्टि की है , उनके लिए सम
बाध्यकारी है और उन्हें पक्षकारों के रूप में जाना जाता है । हालां कि CITES कानूनी रूप से
ू रे शब्दों में , उनके लिए इस समझौते को लागू करना अनिवार्य
पक्षकारों के लिए बाभ है - दस
होता है फिर भी यह राष्ट् रीय कानूनों का ग्रहण नहीं करता है । इसके बजाय यह प्रत्ये क
पक्षकार द्वारा घरे लू कानूनों के अं तर्गत CITES के राष्ट् रीय स्तर पर को सु निश्चित करने हे तु
एक ढांचा प्रदान करता है । इसलिए कथन 2 सही नहीं है ।
 CITES को सं युक्त राष्ट् र पर्यावरण कार्यक् रम (UNEP) के माध्यम से प्रशासित किया जाता
है । इसका सचिवालय जिने वा, स्विट् जरलैं ड में स्थित है जो सं धि के कार्यान्वयन की दे खरे ख
और दे शों के मध्य सं चार में सहायता करता है । इसलिए कथन 3 में सही है ।
 CITES में तीन परिशिष्टों को शामिल किया गया है । जिन प्रजातियों के लिए व्यापार
नियं त्रित होता है , उन्हें CITES की तीन परिशिष्टों में से एक में सूचीबद्ध किया जाता है ।
प्रत्ये क परिशिष्ट एक भिन्न स्तर का विनियमन प्रदान करती है । प्रजातियों के व्यापार के
लिए CITES परमिट या प्रमाण पत्र की आवश्यकता होती है ।

ू ण के प्रमु ख प्रभाव हैं ?


64. निम्नलिखित में से कौन-से अमोनिया प्रदष

1. अर्ध-प्राकृतिक पारिस्थितिक तं तर् पर सु पोषण और अम्लीकरण प्रभाव

2. जल निकायों में खतरनाक रूप से कम ऑक्सीजन के स्तर वाले मृ त क्षे तर्

3. वायु मंडल में धुं ध जै सी स्थितियों का बनना

नीचे दिए गए कू ट का प्रयोग कर सही उत्तर चु निए।

(a) केवल 1 और 2

(b) केवल 2 और 3
(c) केवल 1 और 3

(d) 1, 2 और 3

Q 64.D

 अमोनिया (NH3) एक रं गहीन अत्यधिक अभिक्रियाशील और घु लनशील क्षारीय गै स है । यह


नाइट् रोजन चक् र का एक प्रमु ख घटक है जो उच्च सांदर् ता पर पारिस्थितिक तं तर् को
प्रतिकू ल रूप में प्रभावित करता है ।
 उत्सर्जन के स्रोत: NH3 के उत्सर्जन का सबसे बड़ा स्रोत कृषि है , जिसमें पशु पालन और NH3
आधारित उर्वरक अनु पर् योग शामिल हैं ।
 NH3 के अन्य स्रोतों में औद्योगिक प्रक्रियाएं , वाहनों से होने वाला उत्सर्जन, मृ दा और
महासागरों से वाष्पीकरण, जै विक अपशिष्ट का अपघटन, वनाग्नि, पशु और मानव अपशिष्ट,
नाइट् रोजन स्थिरीकरण प्रक्रियाएं शामिल हैं ।
 ू ण के प्रभाव:
अमोनिया प्रदष
o पर्यावरण पर - जलवायु परिवर्तन - नाइट् रीकरण और विनाइट् रीकरण द्वारा ग्रीनहाउस
गै स उत्सर्जन में योगदान दे ता है ।
ू ण - वातावरण में अन्य ऑक्साइडों और प्रदष
o वायु प्रदष ू कों के साथ अमोनिया की
अभिक्रिया के कारण धुं ध जै सी स्थितियां बनती हैं । इसलिए विकल्प 3 सही है ।
ू ण - जलीय जीवों के लिए विषै ले शै वाल प्रस्फुटन और खतरनाक रूप से कम
o जल प्रदष
ऑक्सीजन के स्तर वाले हानिकारक और मृ त क्षे तर् ों में योगदान दे ता है । इसलिए विकल्प
2 सही है ।
o पारिस्थितिकी तं तर् को नु कसान- अर्ध-प्राकृतिक पारिस्थितिक तं तर् पर सु पोषण
(यूट्रोफिकेशन) और अम्लीकरण प्रभाव तथा प्रजातियों की सं रचना में परिवर्तन का
कारण बनता है । इसलिए विकल्प 1 सही है ।
 मनु ष्यों में - अमोनिया त्वचा, आं खों, मु ख गु हा, श्वसन नली में मौजूद नमी के साथ अमोनियम
हाइड्रॉक्साइड बनाने के लिए अभिक्रिया करती जो बहुत ही सं क्षारक (caustic) होता है और
कोशिका झिल्ली, वसा को बाधित करता है , जो अं ततः कोशिकीय विनाश का कारण बनता है ।
ू कों के साथ अभिक्रिया करके अमोनिया
o गै सीय अमोनिया वायु में मौजूद अन्य प्रदष
लवण के सूक्ष्म कण बनाती है जो मानव श्वसन को प्रभावित करते हैं और साथ ही
निमोनिया और अस्थमा जै सी बीमारियों का कारण बनते हैं ।

65. ट् रैफिक, वन्यजीव व्यापार निगरानी ने टवर्क के सं दर्भ में निम्नलिखित कथनों पर विचार कीजिए:
1. इसे प्रकृति के सं रक्षण के लिए अं तर्राष्ट् रीय सं घ (IUCN) और सं युक्त राष्ट् र द्वारा स्थापित किया
गया था।

2. यह वर्तमान में एक स्वतं तर् गै र-लाभकारी सं गठन है ।

उपर्युक्त का नो में से कौन-सा/से सही है /हैं ?

(a) केवल 1

(b) केवल 2

(c) 1 और 2 दोनों

(d) न तो 1, न ही 2

Q65.B

 ट् रैफिक (TRAFFIC) एक ऐसा सं गठन है जिसे 1976 में विश्व वन्यजीव कोष (WWF) और
अं तर्राष्ट् रीय प्रकृति सं रक्षण सं घ (IUCN) द्वारा वन्यजीव व्यापार निगरानी ने टवर्क के रूप में
स्थापित किया गया था, जो वन्यजीवों के व्यापार पर डे टा सं गर् ह, विश्ले षण और सूचित
निर्णय निर्माण के लिए अनु शंसाओं का प्रावधान करता है । इसलिए कथन 1 सही नहीं है ।
 40 से अधिक वर्षों से TRAFFIC ने WWF और IUCN के सं युक्त कार्यक् रम के रूप व्यापार
अनु संधान में अग्रणी रूप से इस कार्य को किया है ।
 वर्ष 2017 में TRAFFIC एक स्वतं तर् गै र-लाभकारी सं गठन बन गया, साथ WWF और IUCN
स्वतं तर् बोर्ड सदस्यों के साथ इसके निदे शक मं डल में सम्मिलित हुए। इसलिए कथन 2 सही
 TRAFFIC, उद्दे श्यपूर्ण और विश्वसनीय जानकारी के प्रदाता में , वन्यजीव व्यापार और
सं रक्षण क्षे तर् में अपनी विशे षज्ञता और प्रभाव के लिए विश्व स्तर पर प्रसिद्ध है । शे षज्ञ कर्मी
नवोन्मे षी परियोजनाओं को लागू करते हैं और वन्यजीव व्यापार चु नौतियों का समाधान करके
प्रकृति का एवं सं धारणीय विकास का समर्थन करने के मिशन को पूरा करने के लिए नए
उपकरण बनाते हैं ।
 TRAFFIC का एक अं तरराष्ट् रीय ने टवर्क है , जिसमें (अफ् रीका, एशिया, यूरोप, उत्तरी और
दक्षिण अमे रिका) महाद्वीपों में कार्यालयों के साथ कैम्ब्रिज, यूनाइटे ड किंगडम में स्थित
ट् रैफिक इं टरने शनल शामिल है ।
66. प्रवासी प्रजातियों के सं रक्षण पर कन्वें शन (CMS) के सं बंध में निम्नलिखित कथनों पर विचार
कीजिए:

1. CMS एकमात्र वै श्विक अं तर सरकारी सं गठन है जो विशे ष रूप से प्रवासी प्रजातियों के सं रक्षण
और प्रबं धन के लिए स्थापित किया गया है ।

2. यह सं युक्त राष्ट् र पर्यावरण कार्यक् रम के तत्वावधान में एक पर्यावरण सं धि है ।

उपर्युक्त कथनों में से कौन-सा/से सही है /हैं ?

(a) केवल 1

(b) केवल 2

(c) 1 और 2 दोनों

(d) न तो 1, न ही 2

Q66.C

 वन्य जं तुओं की प्रवासी प्रजातियों के सं रक्षण पर कन्वें शन (बॉन कन्वें शन; CMS) सं युक्त
राष्ट् र पर्यावरण कार्यक् रम के तत्वावधान में एक पर्यावरण सं धि है । इसलिए कथन 2 सही है ।
 यह प्रवासी जं तुओं और उनके आवासों के सं रक्षण और सं धारणीय उपयोग के लिए एक
वै श्विक मं च प्रदान करता है । CMS एकमात्र वै श्विक और सं युक्त राष्ट् र-आधारित अं तर
सरकारी सं गठन है जो विशे ष रूप से स्थलीय जीवों, जलीय जीवों और पक्षियों की प्रवासी
प्रजातियों के सं रक्षण और प्रबं धन के लिए स्थापित किया गया है । इसलिए कथन 1 सही है ।
 विलु प्त होने वाले सं कटग्रस्त प्रवासी प्रजातियों को इस कन्वें शन के परिशिष्ट | में सूचीबद्ध
किया गया है । CMS पक्षकार इन जं तुओं का सख्ती से सं रक्षण करने , उनके रहने वाले स्थलों
को सं रक्षित या पु नर्स्थापित करने , प्रवास में आने वाली बाधाओं को कम करने और उनके
समक्ष सं कट उत्पन्न करने वाले अन्य कारकों को नियं त्रित करने की दिशा में प्रयास करते हैं ।
कन्वें शन में शामिल होने वाले प्रत्ये क राज्य के लिए दायित्वों को स्थापित करने के
अतिरिक्त, CMS इन प्रजातियों में से कई प्रजातियों के रें ज वाले राज्यों (Range States) के
बीच ठोस कार्रवाई को बढ़ावा दे ता है ।
 प्रवासी प्रजातियां जिन्हें अं तर्राष्ट् रीय सहयोग की आवश्यकता है अथवा जो इससे
महत्वपूर्ण रूप से लाभान्वित होंगी, उन्हें कन्वें शन के परिशिष्ट || में सूचीबद्ध किया गया है ।
इस कारण से , कन्वें शन वै श्विक या क्षे तर् ीय समझौतों को समाप्त करने के लिए प्रजातियों के
रें ज वाले राज्यों को प्रोत्साहित करता है ।
67. एक वृ क्ष के दो परस्पर निकटतम द्रमों के बीच की अधिक दरू ी किस बात का सं केत दे ती है ?
(a) अपे क्षतया शीतल जलवायु
(b) अपे क्षतया नम जलवायु
(c) हिमानियों का अग्रसर होना
(d) अपे क्षतया शु ष्क जलवायु
उत्तर-B

68. अग्रसर होती हुई हिमानियों का वृ क्षों के द्रुमों पर क्या प्रभाव पड़ता है ?
(a) ये सकेंद्रीय हो जाते हैं
(b) वृ त्तों की आकृति विकृत हो जाती है
(c) इनके बीच परस्पर दरू ी बढ़ जाती है
(d) कोई परिवर्तन नहीं होता
उत्तर-B

69. पृ थ्वी की धु री के झुकाव का कोण 22.1° से 24.5° के मध्य परिवर्तित होता रहता है । इस चक् रीय प्रक्रिया
की अवधि कितने वर्ष की है ?
(a) लगभग 21000 वर्ष
(b) लगभग 25000 वर्ष
(c) लगभग 36000 वर्ष
(d) लगभग 41000 वर्ष
उत्तर-D

70. निम्नलिखित कथनों पर विचार कीजिए:


1. सभी पादप अपने भोजन की आपूर्ति प्रकाश सं श्ले षण द्वारा करते हैं ।
2. सभी जन्तु अपने भोजन के लिए पादपों तथा अन्य जीवों पर निर्भर करते हैं ।
उपरोक्त कथनों में से कौन-सा/से कथन सही है /हैं ?
(a) केवल 1
(b) केवल 2
(c) 1 तथा 2 दोनों
(d) दोनों में से कोई नहीं
उत्तर-B

71. निम्नलिखित कथनों पर विचार कीजिए:


1. किसी पारितं तर् का जै विक घटक अपने जीवन के लिए अजै व पर्यावरण पर निर्भर करता है ।
2. अजै व पर्यावरण सामान्यतया अपरिवर्तनशील होता है तथा जै विक घटकों से प्रभावित नहीं होता।
उपरोक्त कथनों में से कौन-सा/से कथन सही है /हैं ?
(a) केवल 1
(b) केवल 2
(c) 1 तथा 2 दोनों
(d) दोनों में से कोई नहीं
उत्तर-A

72. निम्नलिखित कथनों पर विचार कीजिए:


ू रे पोषी स्तर पर ऊर्जा का सं चरण एक ही दिशा में होता है ।
1. एक पोषी स्तर से दस
2. ऊर्जा की भां ति जै विक पदार्थ का खाद्य श्रखृं ला के रूप सं चरण एक ही दिशा में होता है ।
उपरोक्त कथनों में से कौन-सा/से कथन सही है /हैं ?
(a) केवल 1
(b) केवल 2
(c) तथा 2 दोनों
(d) दोनों में से कोई नहीं
उत्तर-C

73. निम्नलिखित कथनों पर विचार कीजिए:


1. जब नाइट् रोजन ऑक्साइड हाइड्रोकार्बन्स के साथ सूर्य के प्रकाश की उपस्थिति में प्रतिक्रिया
करती है तो इससे ओजोन की उत्पत्ति होती है ।
2. समताप मं डल में नाइट्रिक ऑक्साइड से प्रतिक्रिया करने पर ओजोन का विखं डन हो जाता है ।
उपरोक्त कथनों में से कौन-सा/से कथन सही है /हैं ?
(a) केवल 1
(b) केवल 2
(c) 1 तथा 2 दोनों
(d) दोनों में से कोई नहीं
उत्तर-C

74. मॉण्ट्रियल प्रोटोकॉल किससे सं बंधित था?


(a) ग्लोबल वॉर्मिं ग पर अं कुश लगाना
(b) ओजोन अवक्षय को रोकना
(c) ग्लोबल वॉर्मिं ग तथा ओजोन के अवक्षय को रोकना
(d) ग्लोबल वॉर्मिं ग और ओजोन के अवक्षय को रोकना तथा जै व विविधता को बचाना
उत्तर-B

75. जीवाश्म ईंधनों के प्रज्ज्वलन के पश्चात् भूमि उपयोग में परिवर्तन वायु मंडल में कार्बन-डाइ-ऑक्साइड के
ू रा सबसे महत्वपूर्ण कारक है । निम्नलिखित क्रियाओं में से कौन-सी क्रिया
बढ़ते हुए अनु पात के लिए दस
इसके लिए सबसे अधिक उत्तरदायी है ?

(a) प्रति हे क्टे यर उत्पादन बढ़ाने के लिए भूमि उपयोग की गहनता में वृ दधि
(b) बहु-फसली कृषि का बढ़ता हुआ स्तर
(c) सिं चित चावल की कृषि का बढ़ता हुआ विस्तार
(d) कृषि तथा अन्य उद्दे श्यों के लिए वनों का ह्रास
उत्तर-D

76. किसी पारितं तर् को प्रकार्यक बनाए रखने के लिए क्या आवश्यक नहीं है ?
(a) सूर्य का प्रकाश
(b) पादप समु दाय
(c) खनिज पदार्थ
(d) मांसाहारी जीव
उत्तर-D

77. किसी खाद्य शृं खला में पोषी स्तरों की सं ख्या को सीमित करने वाला सबसे महत्वपूर्ण कारक कौन-सा है ?
(a) प्रत्ये क स्तर पर खाद्य पदार्थ के स्वरूप में परिवर्तन होना
(b) प्रत्ये क स्तर पर उपलब्ध भोजन की मात्रा का घटते जाना
(c) प्रत्ये क स्तर पर जीवों की सं ख्या का बढ़ना
(d) प्रत्ये क स्तर पर जीवों का भोजन के लिए अधिकाधिक चयनात्मक होना
उत्तर-B

78. सामान्यतया खाद्य श्रखृं लाओं में ऊपरी पोषी स्तरों पर जीवों की सं ख्या घटती जाती है । इसका एक
महत्वपूर्ण अपवाद है :
् होना
(a) खाद्य शृं खला के अं त में एकाएक जै विक पदार्थ की मात्रा में वृ दधि
् होना
(b) खाद्य शृं खला के अं त में ऊर्जा की आपूर्ति में वृ दधि
(c) खाद्य शृं खला के अं त में अपघटकों की सं ख्या का अत्यधिक होना
(d) खाद्य श्रखृं ला का खाद्य जाल में परिवर्तित होना
उत्तर-C

79. पारिस्थितिकी अनु दान का एक उदाहरण है :


(a) पौधों की उत्पादकता बढ़ाने के लिए सिं चाई तथा उर्वरकों का उपयोग करना
(b) पौधों को काटना
(c) किसी पारितं तर् में नई प्रजातियों को लाना
(d) किसी पारितं तर् से किसी प्रजाति को हटाना
उत्तर-A

80. किसी खाद्य श्रखृं ला में ऊपर के पोषी स्तरों पर जाते हुए ऊर्जा दक्षता में क्या परिवर्तन दे खने को मिलते हैं ?
(a) ऊर्जा दक्षता के घटने की प्रवृ त्ति पाई जाती है
(b) ऊर्जा दक्षता के बढ़ने की प्रवृ त्ति पाई जाती है
(c) ऊर्जा दक्षता लगभग स्थिर रहती है ।
(d) प्राथमिक स्तरों पर दक्षता बढ़ने की तथा अन्तिम चरणों में घटने की प्रवृ त्ति पाई जाती है
उत्तर-B

81. अने क जलीय पारितं तर् ों में जै व भार पिरामिड उल्टे हो जाते हैं । क्यों?
् की तथा खाए जाने की दर लगभग समान होती है , जिससे इस स्तर पर जै व भार की
(a) पादप प्लवकों की वृ दधि
मात्रा कम रहती है ।
(b) पादप प्लवकों को खाने वाली मछलियों की जीवन प्रत्याशा प्लवकों से अधिक होती है ।
(c) जलीय पारितं तर् ों में उत्पादकों की सं ख्या कम होती है ।
(d) ये पिरामिड उल्टे कभी नहीं होते
उत्तर-A

82. किसी खाद्य श्रख ् का प्रमु ख कारण क्या है ?


ृं ला के अन्तिम पोषी स्तरों पर ऊर्जा दक्षता में वृ दधि
(a) श्वसन में होने वाले ऊर्जा उपयोग में कमी

(b) स्वां गीकरण दक्षता में वृ दधि
(c) प्रजनन क्रिया में उपयोग होने वाली ऊर्जा में कमी
(d) उपलब्ध भोजन की मात्रा में कमी
उत्तर-B

83. एक कृषि पारितं तर् में दलहन जै से पौधों का क्या महत्व है ?


(a) अधिक पत्तों के कारण ये जै विक खाद का अच्छा स्रोत होते हैं
(b) इनकी जड़ों में नाइट् रोजन को मिट् टी में स्थापित करने वाले बै क्टीरिया पाए जाते हैं ।
(c) इनकी जै विक उत्पादकता अधिक होती है
(d) ये पौधे कम नमी में उग सकते हैं
उत्तर-B

84. पं जाब तथा हरियाणा में भूजल में नाइट् रेट का अनु पात काफी ऊंचा पाया में जाता है । इसका प्रमु ख कारण
है :
(a) जलोढ़ मिट् टी में नाइट् रेट की अधिकता
(b) औद्योगीकरण तथा शहरीकरण का ऊँचा स्तर
(c) गहरी चट् टानों से नाइट् रेट का भू-जल में घु लना
(d) मिट् टी से नाइट् रेट का सतत् निक्षालन
उत्तर-D

85. जलराशियों की जै विक ऑक्सीजन आवश्यकता से क्या तात्पर्य है ?


(a) जै विक पदार्थों के अपघटन के लिए ऑक्सीजन की आवश्यकता
(b) पानी की मात्रा को बनाए रखने के लिए ऑक्सीजन की आवश्यकता
् के लिए ऑक्सीजन की आवश्यकता
(c) जलीय वनस्पति की वृ दधि
(d) जल के वाष्पन के लिए ऑक्सीजन की आवश्यकता
उत्तर-A

ू त जल राशियों से निकलने वाली दुर्गंध का क्या कारण होता है ?


86. अत्यधिक जै विक पदार्थ से प्रदषि
(a) जै विक पदार्थ के अपघटन के लिए ऑक्सीजन की समु चित मात्रा का न होना
(b) अत्यधिक जै विक पदार्थ का ते जी से अपघटित होना
(c) जलीय जीवों के शरीर से निकलने वाली दुर्गंध
(d) अत्यधिक मात्रा में कार्बन-डाइ-ऑक्साइड का उत्सर्जन
उत्तर-A

87. कृषि पारितं तर् ों में कीटनाशक दवाओं के उपयोग से निम्नलिखित में से क्या नहीं होता?
ू ण
(a) भू-जल प्रदष
(b) मृ दा विकास प्रक्रिया में विघ्न
(c) पक्षियों की सं ख्या में कमी
(d) पादप प्रजातियों में कमी
उत्तर-D

ू ण से निम्नलिखित में से कौन-सा परिणाम होता है ?


88. जल के रासायनिक प्रदष

(a) जै विक ऑक्सीजन आवश्यकता में वृ दधि
(b) खाद्य श्रख ू कों का केंद्रण
ृं ला में रासायनिक प्रदष
ू ण
(c) जल का तापीय प्रदष
(d) उपरोक्त में से कोई भी नहीं
उत्तर-B

ू ण का प्रमु ख स्रोत क्या है ?


89. जल के तापीय प्रदष
(a) जलीय वनस्पति का अपघटन
(b) उद्योगों में जल का शीतलन के लिए उपयोग
(c) प्लवक के द्वारा सौर ऊर्जा का सं चयन
(d) जलराशियों की सौर ऊर्जा को अधिक अवशोषित करने की प्रवृ त्ति
उत्तर-B

ू ण की समस्या उत्पन्न नहीं होती?


90. किस मानवीय क्रिया से भू-प्रदष
(a) कृषि
(b) वानिकी
(c) खनन
(d) औद्योगीकरण
उत्तर-B

ू ण का प्रमु ख स्रोत क्या है ?


91. समु दर् ी पारितं तर् ों में रे डियोधर्मी प्रदष
(a) वायु मंडलीय धूल
(b) ज्वालामु खी क्रिया
(c) परमाणु ऊर्जा से चलने वाली पनडु ब्बियाँ
(d) समु दर् की सतह पर पे ट्रोलियम पदार्थों का फैलना
उत्तर-C

92. समु दर् की सतह पर ते ल फैलने से समु दर् ी पक्षी क्यों मर जाते हैं ?
(a) ते ल लगी मछलियाँ खाने से
(b) जल में ऑक्सीजन की कमी होने से
(c) पीने के लिए स्वच्छ पानी की कमी से
(d) पं खों में ते ल भर जाने से शरीर का तापमान घटने से
उत्तर-D

93. यह माना जाता है कि सी एफ सी का उत्पादन बं द करने के पश्चात् भी ओजोन परत का ह्रास जारी है ।
क्यों?
(a) उत्पादन बं द होने के बाद भी पहले से उत्पादित सी एफ सी का उपयोग जारी है
(b) सी एफ सी काफी दीर्घ अवधि तक क्रियाशील बने रहते हैं
(c) सी एफ सी की प्रतिस्थापक गै सें भी ओजोन का ह्रास करती हैं
(d) सभी दे शों ने सी एफ सी के उत्पादन पर रोक नहीं लगाई है
उत्तर-B

94. पशु पालन का जै व विविधता पर क्या प्रभाव पड़ा है ?


् हुई है
(a) इससे जै व विविधता में वृ दधि
(b) इससे जै व विविधता में कमी हुई है
(c) इस क्रिया से जै व विविधता पर कोई प्रभाव नहीं पड़ता
(d) पालतू पशु ओं को जै व विविधता में सम्मिलित नहीं किया जाता
उत्तर-B

95. वन क्षे तर् ों के अलग-थलग खं डों में बं ट जाने का क्या परिणाम हो सकता
(a) इससे वन्य जीवों पर विशे ष प्रभाव नहीं पड़ता जब तक वनों के अं तर्गत क्षे तर् फल में महत्वपूर्ण कमी न
आए
(b) वनों के इस पर विभाजित होने से वन्य जीवों की सु रक्षा बढ़ जाती
(c) किसी प्रकार की आपदा की परिस्थिति में वन्य जीव स्थानांतरण नहीं कर पाते और मर जाते हैं
् होती है
(d) इस प्रकार के विभाजन से जै व विविधता में वृ दधि
उत्तर-C

96. किसी पारितं तर् में जै व विविधता के अधिक होने का क्या परिणाम होता
(a) अधिक जै व विविधता पारितं तर् को मजबूत बनाती है ।
(b) जै व विविधता की अधिकता जीव-जं तुओं की सं ख्या पर अं कुश लगाती है
(c) अधिक जै व विविधता वाले पारितं तर् अधिक भं गुर होते हैं
(d) अधिक जै व विविधता पारितं तर् में ऊर्जा की मां ग को कम करती है
उत्तर-A

97. वनस्पति के अनु क्रमण से क्या तात्पर्य है ?


(a) वनस्पति का जलवायु परिवर्तन से प्रभावित होना
(b) अनु कूलन की प्रक्रिया तथा पर्यावरण से अन्योन्य क्रिया के माध्यम से एक पूर्णतया अनु कूलित तथा
स्थाई समु दाय का विकसित होना
ू रे को प्रतिस्थापित करना
(c) वनस्पति के एक समु दाय द्वारा दस
(d) पादप प्रजातियों में विशिष्ट लक्षणों का विकास होना
उत्तर-B

98. वर्ष 1883 के पश्चात् कराकटोआ द्वीप पर आरं भ होने वाला जै विक अनु क्रमण किस प्रकार का अनु क्रमण
था?
(a) प्राथमिक
(b) द्वितीयक
(c) मृ दीय
(d) मरुद्भिद
उत्तर-B

99. जै विक अनु क्रमण में प्रजातीय विविधता किस क् रमक में सर्वाधिक होती है ?
(a) पायनियर
(b) जलवायविक चरम
(c) जलवायविक चरम से कुछ पहले
(d) प्रजातीय विविधता तथा क् रमकों में कोई सं बंध नहीं होता
उत्तर-C

100. पारिस्थितिकी अध्ययन में सबसे छोटी इकाई को क्या कहा जाता है ?
(a) जीवोम
(b) बायोम
(c) पारिनिकेतन
(d) पारितं तर्
उत्तर-C

101. एक प्रकार्यक इकाई को, जिसमें विभिन्न जीव-जं तुओं के बीच अन्योन्य क्रिया के परिणामस्वरूप ऊर्जा
सं चरण होता है , क्या कहा जाता है ?
(a) जीवोम
(b) बायोम
(c) फॉर्मेशन क्लास
(d) पारितं तर्
उत्तर-D

102. एक ऐसी बृ हत इकाई को, जिसमें पादप तथा जं तु एक विशिष्ट प्रकार के पर्यावरण के साथ अनु कूलित है
तथा जिसमें वनस्पति का एक विशिष्ट जीवन स्वरूप होता है , क्या कहा जाता है ?
(a) पारिनिकेतन
(b) पारितं तर्
(c) जीवोम
(d) जलवायविक समु दाय
उत्तर-C

103. टै गा बायोम में मृ दा में जै विक पदार्थ की काफी बड़ी मात्रा पाई जाती है । इसका सबसे प्रमु ख कारण क्या
है ?
(a) वनों की अति ऊँची उत्पादकता
(b) वृ क्षों का बड़ी मात्रा में पत्ते गिराना
(c) निक्षालन की कमी
(d) निम्न तापमान के कारण जै विक अपघटन की दर का नीचा होना
उत्तर-D

104. शीत प्रदे शों में पाए गए जीव शीत निद्रा में क्यों जाते हैं ?
(a) ठं ड से बचने के लिए
(b) भोजन की आपूर्ति न होने के समय ऊर्जा बचाने के लिए
(c) प्रजनन के लिए
(d) उपरोक्त सभी उद्दे श्यों के लिए
उत्तर-B

105. निम्नलिखित कथनों पर विचार कीजिए:


1. स्टे पी क्षे तर् गे हं ू के उत्पादन के लिए प्रसिद्ध हैं ।
2. ये क्षे तर् अने क बार सूखा ग्रस्त होते हैं ।
उपरोक्त कथनों में से कौन-सा/से कथन सही है /हैं ?
(a) केवल 1
(b) केवल 2
(c) 1 तथा 2 दोनों
(d) दोनों में से कोई नहीं
उत्तर-C

106. ध्रुवीय भालू तथा ध्रुवीय लोमड़ी किस बायोम के जन्तु हैं ?
(a) टै गा वन
(b) टु ं ड्रा
(c) बोरियल वन
(d) शं कुधारी वन
उत्तर-B

107. सर्वाधिक पादप विविधता किस जीवोम में पाई जाती है ?


(a) उष्ण कटिबं धीय वर्षा वन
(b) उष्ण कटिबं धीय पर्णपाती वन
(c) सवाना
(d) मानसून वन
उत्तर-A

108. भारत के किस क्षे तर् को जाति उद्भवन का स्रोत क्षे तर् कहा जाता है ?
(a) पश्चिमी घाट
(b) पूर्वी हिमालय
(c) पश्चिमी हिमालय
(d) सुं दरवन
उत्तर-B

109. समु दर् ी वर्षा वन की सं ज्ञा किस बायोम को दी जाती है ?


(a) विस्तृ त शै वाल प्रस्फुटन के क्षे तर्
(b) मैं ग्रोव (कच्छ) वनस्पति जीवोम
(c) प्रवाल भित्ति जीवोम
(d) मैं ग्रोव तथा प्रवाल भित्तियों दोनों को
उत्तर-C

110. निम्नलिखित कथनों पर विचार कीजिए:


1. मैं ग्रोव पादपों में लगभग 110 प्रजातियों को सम्मिलित किया जाता है ।
2. इन जीवोमों में पादप विविधता काफी ऊँची होती है ।
उपरोक्त कथनों में से कौन-सा/से कथन सही है /हैं ?
(a) केवल 1
(b) केवल 2
(c) 1 तथा 2 दोनों
(d) दोनों में से कोई नहीं
उत्तर-A

111. प्रवाल भित्तियों के 50 मी. से अधिक गहराई पर न पनपने का प्रमु ख कारण क्या है ?
(a) इससे अधिक गहराई पर तापमान कम होना
(b) अधिक गहराई पर लवणता का अधिक होना
(c) अधिक गहराई पर सूर्य के प्रकाश का न पहुँचना
(d) अधिक गहराई पर जल में गतिशीलता अधिक होना
उत्तर-C

112. बड़ी नदियों के मु हानों के आस-पास प्रवाल भित्तियाँ नहीं पाई जाती अथवा उनकी निरं तरता भं ग हुई
पाई जाती है । इसका प्रमु ख कारण है :
(a) नदियों द्वारा अवसादों का लाया जाना
(b) इन क्षे तर् ों में लवणता का कम होना
(c) इन क्षे तर् ों में जल के बहाव का समु दर् की ओर होना
(d) इन क्षे तर् ों में तापमान का बढ़ना
उत्तर-A

113. भारत के किस प्रदे श को पांच प्रकार के पामों की उत्पत्ति का क्षे तर् माना जाता है ?
(a) पूर्वी हिमालय
(b) पश्चिमी घाट
(c) अं डमान-निकोबार द्वीप समूह
(d) पश्चिम हिमालय
उत्तर-A

114. नामदाफा क्षे तर् में पाया जाने वाला एक स्थानिक प्राणी है :
(a) उड़ने वाली गिलहरी
(b) रिडलीज टर्टल
(c) नदियों में रहने वाली डॉल्फिन
(d) लायन-टे ल मै कॉक
उत्तर-A
115. पूर्वी हिमालय से बाहर ऑर्कि ड की ऊँची स्थानिकता वाला एक प्रमु ख क्षे तर् हैं :
(a) पश्चिमी हिमालय
(b) मध्य भारत के मानसून वन
(c) पश्चिमी घाट
(d) पूर्वी घाट
उत्तर-C

116. भारत में पाए जाने वाले जं गली गधे की आश्रय स्थली किस प्रकार की है ?
(a) शीतोष्ण घास भूमि
(b) अर्द्ध-मरुस्थलीय वन
(c) लवणीय घास भूमि
(d) लवणीय दलदली क्षे तर्
उत्तर-C

117. जं गली गधे की आश्रय स्थली किस मानवीय क्रिया से सर्वाधिक प्रभावित हो रही है , जिससे इस
प्रजाति का अस्तित्व खतरे में है ?
(a) पशु चारण
(b) कृषि
(c) नमक उत्पादन
(d) परिवहन तं तर् का विकास
उत्तर-C

118. भारत की कौन-सी नदियों में डॉल्फिन पाई जाती है ।


(a) गं गा - ब्रह्मपु तर्
(b) गं गा - यमु ना
(c) गं गा - गोदावरी
(d) गं गा - कृष्णा
उत्तर-A

 Q1. पीएम मोदी ने किस राज्य में सावरा-कुड्डू जलविद्यु त परियोजना का उद्घाटन किया है ?
(a) गुजरात
(b) उत्तर प्रदे श
(c) हिमाचल प्रदे श
(d) महाराष्ट्र
उत्तर (सी)
Q2. किस राज्य के मु ख्यमंतर् ी ने 'कौशल रोजगार निगम' पोर्टल लॉन्च किया है ?
(a) हरियाणा
(b) बिहार
(c) राजस्थान
(d) असम
उत्तर (ए)

Q3. श्यामा प्रसाद मुखर्जी रूर्बन मिशन के कार्यान्वयन में किस राज्य ने प्रथम स्थान प्राप्त किया
है ?
(a) बिहार
(b) असम
(c) तमिलनाडु
(d) तेलंगाना 
उत्तर (d)

Q4. किस दे श ने मिनीबस की तरह दिखने वाला दु निया का पहला डुअल-मोड वाहन (DMV) पेश
किया है जो सड़कों और पटरियों पर चल सकता है ?
(a) वियतनाम
(b) चीन
(c) मले शिया
(d) जापान
उत्तर (डी)

Q5. प्रधानमंतर् ी नरें द्र मोदी ने ________ पर ब्लॉकचे न-आधारित डिजिटल डिग्री लॉन्च की।
(a) आईआईटी दिल्ली
(b) आईआईटी कानपुर
(c) आईआईटी बॉम्बे
(d) आईआईटी रुड़की
Ans (बी)

Q6. वर्ष 2019–20 के लिए राज्य स्वास्थ्य सूचकांक के NITI Aayog के चौथे संस्करण में बड़े राज्यों
में कौन सा राज्य शीर्ष पर है ?
(a) तमिलनाडु
(b) कर्नाटक
(c) केरल
(d) आंध्र प्रदे श
उत्तर (सी)

Q7. निम्नलिखित में से किस दे श ने हाल ही में "ज़ियु आन-1 02 ई" या "पांच मीटर ऑप्टिकल
उपग्रह 02" लॉन्च किया है ?
(a) इंडोने शिया
(b) दक्षिण कोरिया
(c) मंगोलिया
(d) चीन
उत्तर (डी)

Q8. पे टा इंडिया के 2021 पर्सन ऑफ द ईयर किसे चुना गया है ?


(a) आलिया भट्ट
(b) अनु ष्का शर्मा
(c) जै कलिन फर्नांडीज
(d) आर माधवन
उत्तर (A)

Q9. हाल ही में , हॉकी इंडिया जूनियर पु रुष राष्ट्रीय चै म्पियनशिप के किस संस्करण को ________
द्वारा ताज पहनाया गया था।
(a) हरियाणा
(b) चंडीगढ़
(c) उत्तर प्रदे श
(d) ओडिशा
उत्तर (सी)

Q10. भु वने श्वर, ओडिशा में संपन्न हुई चौथी पैरा-बै डमिंटन राष्ट्रीय चैं पियनशिप में डबल गोल्ड
किसने जीता है ?
(a) मनोज सरकार
(b) प्रमोद भगत
(c) तरुण ढिल्लों
(d) नितेश कुमार 
उत्तर (d)
Q11. 255,700 पंजीकृत EVs के साथ कौन सा राज्य शीर्ष स्थान पर है ?
(a) गु जरात
(b) कर्नाटक
(c) तमिलनाडु
(d) उत्तर प्रदे श
Ans.(d)

Q12. वै श्विक स्वास्थ्य सुरक्षा (GHS) सूचकांक 2021 में भारत की रैं क क्या था? 
(a) 49
(b) 55
(c) 66 
(d)74
Ans.(c)

Q13. भारत को 2022-2023 के लिए अंतर्राष्ट्रीय समुद ्री संगठन परिषद के लिए फिर से चुना गया है ।
अंतर्राष्ट्रीय समुद ्री संगठन (IMO) का मु ख्यालय कहाँ है ?
(a) मले शिया, कुआलालं पुर
(b) जकार्ता, इं डोने शिया
(c) शं घाई, चीन
(d) लं दन, यूनाइटे ड किंगडम
Ans.(d)

Q14. निम्नलिखित में से किसने 2021 मिस यूनिवर्स का ताज जीता है ?


(a) एडलाइन कैस्टे लिनो
(b) सं जना विज
(c) वर्तिका सिं ह
(d) हरनाज सं ध ू
Ans.(d)

Q15. अबू धाबी जीपी 2021 में अपना पहला फॉर्मूला वन ड्राइवर्स चैं पियनशिप खिताब किसने जीता है ?
(a) सर्जियो पे रेज़
(b) एस्टे बन ओकोन
(c) मै क्स वे रस्टै पे न
(d) वाल्टे री बोटास
Ans.(c)
Q16. भारत ने एशियाई रोइंग चैं पियनशिप में दो स्वर्ण और 4 रजत पदक सहित कुल छह पदक जीते हैं ।
एशियाई रोइंग चै म्पियनशिप 2021 _________ में आयोजित की गई थी।
(a) वियतनाम
(b) थाईलैं ड
(c) इं डोने शिया
(d) मले शिया
Ans.(b)

Q17. " Watershed: How We Destroyed India’s Water And How We Can Save It " नामक नई
पु स्तक किसने लिखी है ?
(a) मृ दुला रमे श
(b) अमित रं जन
(c) सु भद्रा से न गु प्ता
(d) भास्कर चट् टोपाध्याय:
Ans.(a) 

Q18. हाल ही में श्री काशी विश्वनाथ धाम परियोजना के चरण 1 के हिस्से के रूप में कितने भवनों का
उद्घाटन किया गया? 
(a) 16
(b) 19
(c) 28
(d) 23 
Ans.(d)

Q19. ऐन राइस कौन थीं, जिनका हाल ही में निधन हो गया?


(a) अभिने तर् ी
(b) ले खक
(c) निदे शक
(d) डांसर
Ans.(b)

Q20. वै श्विक स्वास्थ्य सुरक्षा सूचकांक 2021 में कौन सा दे श शीर्ष पर है ?


(a) फिनलैं ड
(b) थाईलैं ड
(c) कनाडा
(d) यूएसए
Ans.(d)

Q21. सरयू नहर राष्ट्रीय परियोजना का उद्घाटन भारत के प्रधानमंतर् ी नरें द्र मोदी ने निम्नलिखित में से
किस राज्य में किया है ?
(a) गु जरात
(b) कर्नाटक
(c) तमिलनाडु
(d) उत्तर प्रदे श 
Ans.(d)

Q22. किस केंद्र शासित प्रदे श की पु लिस ने प्रमुख योजना 'यु वा' के तहत स्कू ल छोड़ने वालों के कौशल
के लिए एक ई-लर्निंग प्ले टफॉर्म "उन्नति" लॉन्च किया है ?
(a) जम्मू और कश्मीर
(b) दिल्ली
(c) लक्षद्वीप
(d) पु डुचे री
Ans.(b)

Q23. आत्मानिर्भर भारत रोजगार योजना (ABRY) के तहत सबसे अधिक लाभार्थियों वाले राज्य की
सूची में कौन सा राज्य शीर्ष पर है ?
(a) पश्चिम बं गाल
(b) झारखं ड
(c) महाराष्ट् र
(d) छत्तीसगढ़
Ans.(c)

Q24.  वाणिज्य और उद्योग मंतर् ालय के तहत भौगोलिक संकेत रजिस्ट्री (GIR) ने किस राज्य के
मिथिला मखाना की भौगोलिक पहचान (GI) टै ग को बनाए रखने के प्रस्ताव को स्वीकार कर लिया है ?
(a) राजस्थान
(b) उत्तराखं ड
(c) उत्तर प्रदे श
(d) बिहार
Ans.(d)
Q25. निम्नलिखित में से कौन 100% पे परलेस होने वाली दु निया की पहली सरकार बन गई है ?
(a) दुबई, सं युक्त अरब अमीरात
(b) लॉज़े न, स्विट् ज़रलैं ड
(c) मॉन्ट्रियल, कनाडा
(d) कुआलालं पुर, मले शिया
Ans.(a)

Q26. #Care4Hockey’अभियान का चे हरा (face) किसे नियु क्त किया गया है ?


(a) सविता पु निया
(b) वं दना कटारिया
(c) मनप्रीत सिं ह
(d) रानी रामपाल
Ans.(d)

Q27. ET BFSI एक्सीलें स अवार्ड्स 2021 में किस बैं क ने दो पुरस्कार जीते हैं ?
(a) यस बैं क
(b) आरबीएल बैं क
(c) एचएसबीसी बैं क
(d) डीबीएस बैं क
Ans.(d)

Q28. टाइम पत्रिका द्वारा 2021 के लिए 'पर्सन ऑफ द ईयर' किसे चुना गया है ?
(a) जो बिडे न
(b) इलॉन मस्क 
(c) व्लादिमीर पु तिन
(d) जे फ बे जोस
Ans.(b)

Q29. किस संगठन ने हे लीकॉप्टर से प्रक्षे पित स्टैं ड-ऑफ एंटी टैं क (SANT) मिसाइल का परीक्षण किया
है ?
(a) भारतीय से ना
(b) हिं दुस्तान एयरोनॉटिक्स लिमिटे ड
(c) डीआरडीओ
(d) रक्षा अनु संधान और विकास प्रयोगशाला
Ans.(c)

Q30. ग्रीन हाइड्रोजन उत्पादन के लिए क्षारीय इले क्ट्रोलाइज तकनीक को बढ़ाने के लिए भाभा परमाणु
अनुसंधान केंद्र (BARC) के साथ किस कं पनी ने समझौता किया है ?
(a) इं डियन ऑयल कॉर्पोरेशन लिमिटे ड
(b) रिलायं स इं डस्ट् रीज
(c) ते ल और प्राकृतिक गै स निगम
(d) भारत पे ट्रोलियम कॉर्पोरेशन लिमिटे ड 
Ans.(d)

Q31. लंबी दूरी की सु परसोनिक मिसाइल असिस्टे ड टॉरपीडो (स्मार्ट ) का ओडिशा के बालासोर तट से
सफल परीक्षण किया गया। स्मार्ट हथियार प्रणाली किस दे श द्वारा विकसित की गई है ?
(a) रूस
(b) भारत
(c) यूएसए
(d) इज़राइल
Ans.(b) 

Q32. BCCI ने _________ की अध्यक्षता में अलग-अलग-एबल्ड क्रिकेट समिति के गठन की घोषणा
की।
(a) सु नील गावस्कर
(b) रोहित शर्मा
(c) सौरव गां गुली
(d) जय शाह
Ans.(c)

Q33. किस दे श ने उपग्रहों का एक नया समूह "शिजियन -6 05" अंतरिक्ष में लॉन्च किया है ?
(a) चीन
(b) जापान
(c) दक्षिण कोरिया
(d) वियतनाम
Ans.(a)
Q34. किस परियोजना ने सांस्कृतिक विरासत संरक्षण 2021 के लिए 2 यूनेस्को एशिया-प्रशांत पुरस्कार
जीते हैं ?
(a) बान खु न फिथक राया, पट् टानी, थाईलैं ड
(b) डोले श्वर हनाफिया जामे मस्जिद, ढाका, बां ग्लादे श
(c) मृ गदायवन पै लेस वु डशॉप, फेचबु री, थाईलैं ड
(d) निजामु द्दीन बस्ती, नई दिल्ली, भारत
Ans.(d)

Q35. न्यूजीलैं ड के एजाज पटे ल ने एक पारी में सभी 10 विकेट लेने के लिए टे स्ट क्रिकेट में ______
गें दबाज बनकर इतिहास रच दिया।
(a) तीसरे
(b) चौथे
(c) सातवें
ू रे
(d) दस
Ans.(a)

Q36. कैम्ब्रिज डिक्शनरी ने हाल ही में निम्नलिखित में से किसे वर्ड ऑफ द ईयर 2021 नामित किया
है ?
(a) Perseverance 
(b) Clarity
(c) Vision
(d) NFT
 Ans.(a)

Q37. भारत और यूरोपीय संघ (EU) अपनी स्वच्छ ऊर्जा और जलवायु साझे दारी को बढ़ाने पर सहमत हुए
हैं । वे संयुक्त रूप से ______ तक विस्तृ त कार्य कार्यक् रम पर सहमत हुए।
(a) 2022
(b) 2023 
(c) 2024
(d) 2025
Ans.(b)

Q38. विश्व मृ दा दिवस 2021 की थीम क्या थी?


(a) Stop Soil Erosion, Save our Future!
(b) Be the Solution to Soil Pollution
(c) Halt soil salinization, boost soil productivity 
(d) Keep Soil Alive, Protect Soil Biodiversity
Ans.(c)

Q39. भारतीय रिजर्व बैं क (RBI) के आंकड़ों के अनुसार, किस राज्य ने महाराष्ट्र को पीछे छोड़ते हुए दे श
का प्रमुख विनिर्माण केंद्र बना दिया है ?
(a) मध्य प्रदे श
(b) गु जरात
(c) उत्तर प्रदे श
(d) तमिलनाडु
Ans.(b)

Q40. 'द मिडवे बैटल: मोदीज रोलर-कोस्टर से केंड टर्म' नामक पु स्तक किसके द्वारा लिखित है ।
(a) गौतम चिं तामणि
(b) विपु ल वर्मा
(c) विपिन अरोड़ा
(d) शिखर सूद
Ans.(a)

You might also like